Integrated Endocrine Pharmacology Ppt. Book

Page 1

Photos: The surgeon in this photo is transfusing donor islet cells into a diabetic patient. The islet cells may take residence in the pancreas and secrete insulin for the patient. Note the new islet cells in the right-hand photo. They are now functioning normally. This patient will never again need to inject insulin. From: Seeley’s Anatomy & Physiology 10th ed. New York, NY: McGraw-Hill 2010.


Overall Goals this of Sequence With exception of endocrine pharmacology of the reproductive organs, kidneys, GIT and adrenal medulla (covered in other modules), by the end of this sequence the learner will:  Understand the functional anatomy, biochemistry and physiology of the endocrine system, including negative feedback inhibition and endocrine axes.  Understand the principles governing disease states that result from over- or under-production of key hormones.  Know the biologic and therapeutic effects of the most commonly used endocrine agonists and antagonists.  Know the indications, contraindications, drug interactions and adverse effects of the most commonly used endocrine agents. Marc Imhotep Cray, MD

2


Topical Outline:  Lect. 1: Overview of Endocrine Pharmacology: Some of major applications of endocrine drugs, including:  Lect. 2: Hypothalamic and Pituitary Disorders  Lect. 3: Thyroid Disorders  Lect.4: Parathyroid Disorders & Calcium Homeostasis  Lect. 5: Corticosteroids & Adrenocortical Dysfunction  Lect. 6: Diabetes Mellitus Marc Imhotep Cray, MD

3


Endocrine Pharmacology Focus of Study Focus of study for each drug: (as applicable)  Classification and class prototype/s  Mechanisms of action  Indications [diagnostic and (or) therapeutic use]  Adverse effects  Drug-drug interactions, cautions and contraindications  Pharmacokinetic properties, drug-disease interactions and other patient-specific considerations  Toxicities and antidotes (or) treatment See companion eNotes: Endocrine and Reproductive System Pharmacology Marc Imhotep Cray, MD

4


Some Key Abbreviations ACTH, Adrenocorticotropic hormone AVP, Arginine vasopressin, antidiuretic hormone cAMP, Cyclic adenosine monophosphate CRH, Corticotropin-releasing hormone DHEA, Dehydroepiandrosterone DHT, Dihydrotestosterone FSH, Follicle-stimulating hormone LH, Luteinizing hormone TSH, Thyroid-stimulating hormone TRH, Thyrotropin-releasing hormone GH, Growth hormone GHRH, Growth hormone-releasing hormone

GnRH, Hypothalamic gonadotropin-releasing hormone GIP, gastric inhibitory peptide GLP-1, glucagon-like peptide-1 hCG, Human chorionic gonadotropin hGH, Human growth hormone hMG, Human menopausal gonadotropin JAK2 janus kinase 2 IRS-1, insulin receptor substrate-1 PI3K, phosphatidyl inositol- 3 kinase STAT, signal transducer and activator of transcription MAPK, mitogen-activated protein kinase SHC, Src homology containing 5


Lect. 1 of 6

Overview of Endocrine Pharmacology: Functional Anatomy, Hormone-Receptor Interactions & Basic Pathophysiologic and Pharmacologic Concepts The aim of this lecture is to present a detailed overview of endocrinology—that is, a structural and functional analysis of general features of hormones. Major emphasis will be placed on how the endocrine system uses its chemical messengers (hormones) to communication between cells by way of discussing the principles of hormone-receptor interactions and cell signaling, as these principles form the basis for the mechanisms by which hormones exert their actions and thus, serve as the foundation for understanding the pharmacology of important endocrinehormonal systems presented in subsequent lectures.

Marc Imhotep Cray, MD

6


Learning Objectives 1. Describe the four classes of chemical messengers and how they signal cells. 2. Understand the similarities and differences between the autonomic nervous system and endocrine system in maintaining homeostasis. 3. Describe the chemical nature and various classifications of hormones. 4. Describe the four major families of cell receptors. 5. Describe hormone-receptor interactions and signal transduction mechanisms. 6. Explain the molecular & cellular mechanism of action of peptide/protein and catecholamine hormones and how they exert their effects on target cells. 7. Explain the molecular & cellular mechanisms of action of steroid and thyroid hormones and how they exert their effects on target cells. 8. Define binding protein, bound hormone, and free hormone and the effects of binding proteins on circulating hormone levels and activity 9. Describe the organizational and functional anatomy of the endocrine system. 10. Understand basic endocrine pathophysiologic and pharmacologic concepts Marc Imhotep Cray, MD

7


Organization & Responsibilities of Endocrine System  Hormones are secreted into blood by endocrine organs throughout body, affecting physiological function at various target sites  Endocrine, or hormonal, system is responsible for regulating most of systems of body, including:  energy metabolism  bone metabolism  cardiovascular system  bone marrow and hematopoiesis  renal system  gastrointestinal system  regulation of food intake and Mulroney SE & Myers AK. Netter's Essential Physiology 2  regulating reproductive processes

nd

Philadelphia: Elsevier, 2016.

Ed.

8


Schematic Overview of Endocrine System Neurohormones, often called “hypothalamic-releasing hormones�

Pituitary hormones (regulatory hormones) (tropic hormones)

Second-tier hormones

Marc Imhotep Cray, MD

Costanzo LS. Physiology. 5th ed., (Board review series). New York: Elsevier; 2009

9


Feed-forward and Feed-back mechanisms Feed-forward

Feed-backward

Hypothalamic Releasing Hormone Pituitary Tropic (Signaling) Hormone Target Glands Second-tier Hormone Tissue/Organ-System Effect Negative Feedback (by second-tier hormone)

NB: Key to understanding endocrine pharmacology are feed-forward and feedback mechanisms that govern how “releasing” factors in hypothalamus control release of hormones in pituitary (regulatory /tropic hormones) that in turn cause release of second-tier hormones that target multiple organs within body Marc Imhotep Cray, MD


Hormones and Their Sites of Production Hormones Synthesized and Secreted by Dedicated Endocrine Glands Pituitary Gland Growth hormone (GH) Prolactin Adrenocorticotropic hormone (ACTH) Thyroid-stimulating hormone (TSH) Follicle-stimulating hormone (FSH) Luteinizing hormone (LH) Thyroid Gland Tetraiodothyronine (T4; thyroxine) Triiodothyronine (T3) Calcitonin Parathyroid Glands Parathyroid hormone (PTH)

Islets of Langerhans Hormones Synthesized by Gonads (Endocrine Pancreas) Ovaries Insulin Estradiol-17β Glucagon Progesterone Somatostatin Inhibin Adrenal Gland Testes Epinephrine Testosterone Norepinephrine Antimullerian hormone (AMH) Cortisol Inhibin Aldosterone Dehydroepiandrosterone sulfate (DHEAS)

White BA & Porterfield SP. Endocrine and Reproductive Physiology, 4th ed. (Mosby physiology monograph series). Mosby, 2013. Marc Imhotep Cray, MD


Hormones and Their Sites of Production (2) Hormones Synthesized in Organs with a Primary Function Other Than Adipose Tissue Endocrine Brain (Hypothalamus) Antidiuretic hormone (ADH; vasopressin) Oxytocin Corticotropin-releasing hormone (CRH) Thyrotropin-releasing hormone Gonadotropin-releasing hormone (GnRH) Growth hormone–releasing hormone (GHRH) Somatostatin Dopamine Brain (Pineal Gland) Melatonin Heart Atrial natriuretic peptide (ANP) Kidney Erythropoietin WhiteImhotep BA & Porterfield Marc Cray, MD SP. Endocrine and Reproductive Physiology, 4th ed. (Mosby physiology monograph series). Mosby, 2013.

Leptin Adiponectin Stomach Gastrin Somatostatin Ghrelin Intestines Secretin Cholecystokinin Glucagon-like peptide-1 (GLP-1) Glucagon-like peptide-2 (GLP-2) Glucose-dependent insulinotropic peptide (GIP; gastrin inhibitory peptide) Motilin Liver Insulin-like growth factor-1 (IGF-I)


Hormones and Their Sites of Production (3) Hormones Produced to a Significant Degree by Peripheral Conversion Lungs Angiotensin II Kidney 1ι,25-dihydroxyvitamin D Adipose, Mammary Glands, Other Organs Estradiol-17β Liver, Sebaceous Gland, Other Organs Testosterone Genital Skin, Prostate, Other Organs 5-Dihydrotestosterone (DHT) Many Organs T3 White BA & Porterfield SP. Endocrine and Reproductive Physiology, 4th ed. (Mosby physiology monograph series). Mosby, 2013. Marc Imhotep Cray, MD


Some disorders often requiring applications of endocrine and metabolic drugs:

Kibble J , Cannarozzi ML. Pathophysiology Flash Cards. New York: McGraw-Hill, 2013 Marc Imhotep Cray, MD

14


Overview of Endocrine System  Endocrine system uses hormones to transfer information between different tissues  uses feedback loops and sensors to ensure constant homeostasis within body  It plays some form of regulatory role in almost all physiologic processes  It has effects on development, growth, metabolism and reproduction and works with almost every organ system, including the nervous and immune system  Control is mediated by a combination of neural and endocrine systems located in hypothalamus and pituitary gland (“The Master Gland”)

 In contrast to neurotransmitters, which work in synapse between neuron endplate and receptors they act on, hormones are secreted into circulation and can work on tissues far away from source of origin Marc Imhotep Cray, MD


Nervous system vs Endocrine system  Two major regulatory systems make important contributions to homeostasis:  the nervous system and  the endocrine system  Common properties:  maintain homeostasis  extensive use of negative feedback  high-level integration in brain  ability to influence processes in distant regions of body  both systems use chemicals for transmission of information  both systems use receptors

Nervous

versus

Endocrine

Wired

Wireless

Neurotransmitters

Hormones

Short Distance

Long Distance

Closeness

Receptor Specificity

Rapid Onset

Delayed Onset

Short Duration

Prolonged Duration

Rapid Response

Regulation 16


Pathways by which nervous system influences hormone secretion

Marc Imhotep Cray, MD

Widmaier EP, Raff H & Strang KT. Vander’s Human Physiology : The Mechanisms of Body Function, 11th ed. New York, NY: McGraw-Hill, 2008.


Overview of Endocrine System Nature of Hormones ď ą Hormones can be divided into five major classes: 1. amino acid derivatives such as dopamine, catecholamine, and thyroid hormone 2. small neuropeptides such as gonadotropin-releasing hormone (GnRH), thyrotropin-releasing hormone (TRH), somatostatin, and vasopressin 3. large proteins such as insulin, luteinizing hormone (LH), and PTH produced by classic endocrine glands 4. steroid hormones such as cortisol, estrogen, progesterone and testosterone that are synthesized from cholesterol-based precursors and 5. vitamin derivatives such as retinoids (vitamin A) and vitamin D Jameson JL. Principles of Endocrinology (Ch.338). In: Longo DL, Fauci AS, et al. Harrison's Principles of Internal Medicine,18th Ed. New York: McGraw-Hill, 2012.

Marc Imhotep Cray, MD

NB: As a rule amino acid derivatives and peptide/protein hormones are water-soluble and interact with cell-surface membrane receptors Steroids, thyroid hormones, vitamin D, and retinoids are lipid-soluble and interact with intracellular nuclear receptors

18


Overview of Endocrine System Biochemical classification of hormones Amino acid derivative Epinephrine (adrenaline) Thyroid hormones (T3, T4) Peptides Thyrotropin-releasing hormone (TRH) Gonadotropin-releasing hormone (GnRH) Vasopressin Oxytocin (OT) Vasoactive intes tinal peptide (VIP) Glucagon Adrenocorticotropic hormone (ACTH) Somatostatin Glycoproteins Thyroid-stimulating hormone (TSH) Follicle-stimulating hormone (FSH) Luteinizing hormone (LH) Chorionic gonadotropin (CG)

NB: Chemical nature of a hormone determines: 1. How it is synthesized, stored, and released 2. How it is carried in blood 3. Its biologic half-life (t1/2) and mode of clearance 4. Its cellular mechanism of action

Proteins Insulin Insulin-like growth actors (IGFs ) Growth hormone (GH) Prolactin (PRL) Placental lactogen (PL) Parathyroid hormone (PTH) Steroid Estrogens (e.g. estradiol) Androgens (e.g. testosterone) Progesterone Cortisol Aldosterone Vitamin derivatives vitamin A vitamin D

White BA & Porterfield SP. Endocrine and Reproductive Physiology, 4th ed. (Mosby physiology monograph series). Mosby, 2013.


Overview of Endocrine System cont. Functional classification of hormones Tropic hormone vs Non-tropic hormone vs Trophic hormone  Tropic hormones are hormones that have other endocrine glands as their target (endocrine target tissues)  Most tropic hormones are produced and secreted by anterior pituitary  For example: Hypothalamus secretes tropic hormones that target anterior pituitary, and thyroid gland secretes thyroxine, which targets hypothalamus and therefore can be considered a tropic hormone (Other examples: TSH, FSH, LH, ACTH)

 Non-tropic hormones are hormones that directly stimulate target cells to induce effects (nonendocrine target tissues)  Non-tropic hormones are those that act directly on targeted tissues or cells, and not on other endocrine gland to stimulate release of other hormones (Ex. GH, PTH, prolactin, oxytocin, vasopressin, aldosterone and MSH)

 Trophic hormones are hormones that have a growth effect, hyperplasia or hypertrophy, on tissue they are stimulating. (Ex. TSH, GH, ACTH) Marc Imhotep Cray, MD

20


Overview of Endocrine System cont. Hypothalamus  One of most important function of hypothalamus is to link the nervous system to the endocrine system via pituitary gland (hypophysis)  Hypothalamus is also responsible for certain metabolic processes and other activities of autonomic nervous system (See Lect. 2)  Hypothalamus synthesizes and secretes neurohormones, often called “hypothalamic-releasing hormones” which in turn, stimulate or inhibit secretion of pituitary hormones (regulatory hormones) in turn, stimulate or inhibit second-tier hormones Marc Imhotep Cray, MD

21


Overview of Endocrine System cont. Chemical nature of hypothalamic factors Hypothalamus, which is a part of CNS and not a gland, produces many releasing and inhibitory hormones (neuropeptides) which control secretion of anterior pituitary hormones Hypothalamic hormone/factor (neurohormones) Chemical nature 1. Thyrotropin releasing hormone (TRH) 2. Corticotropin releasing hormone (CRH) 3. Gonadotropin releasing hormone (GnRH), (LH-RH/FSH-RH) 4. Prolactin release inhibitory hormone (PRIH) 5. Growth hormone releasing hormone (GHRH) 6. Somatostatin (Growth hormone release inhibitory hormone) Marc Imhotep Cray, MD

Tripeptide Peptide (41 AAs) Decapeptide Dopamine Peptide (40, 44 AAs) Peptide (14 AA) 22


Overview of Endocrine System cont. Overall Function  A hormone is a substance secreted into bloodstream by one tissue but has actions at remote tissues  widespread delivery of hormones in bld makes endocrine system ideal for functional coordination of multiple organs and cell types NB: Signaling mechanisms which use enzymes, neurotransmitters, hormones, and receptors are similar (aside from distance)  Nervous System chemical mediator (neurotransmitter)  Endocrine system chemical mediator (hormone)  Physiologic, biochemical and pharmacologic principles are same Marc Imhotep Cray, MD

23


Routes by which chemical signals are delivered to cells 1. Autocrine chemical messengers stimulates the cell that originally secreted it (e.g. WBCs) 2. Paracrine chemical messengers act locally on nearby cells (e.g. cytokines) 3. Neurotransmitters secreted by neurons that activate an adjacent cellďƒ another neuron, a muscle cell, or a glandular cell (e.g. acetylcholine) 4. Endocrine chemical messengers are hormones secreted into bloodstream by certain glands and cells– and act at a distant site (e.g. insulin) Marc Imhotep Cray, MD Medical Sciences 2nd Edn. Naish J & Court DS. Eds. Elsevier, 2015.


Overview of Endocrine System cont.  Classically, hormones are released into bloodstream and act on tissues distant from site of hormone production an endocrine effect however,  Some hormones act locally within tissue where they are produced called “local hormones” or paracrine effects  Some hormones have both local and systemic effects  act in a paracrine and endocrine manner  Example is testosterone, has local actions in testes and hormonal effects on muscle

 Some hormones, particularly growth factors, exert their actions on cells which secrete them called autocrine effects Marc Imhotep Cray, MD

25


Overview of Endocrine System cont. Hormone Action  True hormones (endocrine secretions) are released by “ductless glands” and are carried by bloodstream to their sites of action  Hormones are part of a larger group of substances that includes autocrine, paracrine, and neuroendocrine secretions

Mulroney SE & Myers AK. Netter's Essential Physiology 2nd Ed. Philadelphia: Elsevier, 2016. Marc Imhotep Cray, MD

26


Overview of Endocrine System cont. Sites and mechanisms of hormone action  Body releases a wide range of endogenous substances, including: neurotransmitters from neuronal cells (e.g. acetylcholine), hormones (e.g. insulin) or cytokines (e.g. interferon), that alter function of target cells  Hormones act on their specific receptors located on or within their target cells  Receptor activation by hormones is translated into response in a variety of ways 1. At cell membrane receptors 2. At cytoplasmic receptors 3. At nuclear receptor Marc Imhotep Cray, MD

27


Overview of Endocrine System cont. Sites and mechanisms of hormone action (2)  Binding of a hormone to its receptor initiates intracellular events that direct hormone’s action  Ultimately, all hormones produce their effects by altering intracellular protein activity  mechanism by which this occurs depends on location of hormone receptor  Receptors are typically located on cell surface or in cell nucleus  As a result most hormones carry out their effects by means of two general mechanisms: 1. Signal transduction and second messenger systems 2. Gene activation, respectively Marc Imhotep Cray, MD

28


Overview of Endocrine System cont. Sites and mechanisms of hormone action (3)  To function, hormones must bind to specific receptors expressed by specific target cell types within target organs  Hormones are also referred to as ligands, in context of ligandreceptor binding, and as agonists, in that their binding to receptor is transduced into a cellular response  Constitutive activation of a receptor leads to unregulated, hormone independent activation of cellular processes

 Receptor antagonists typically bind to a receptor and lock it in an inactive state unable to induce a cellular response  Loss or inactivation of a receptor leads to hormonal resistance Marc Imhotep Cray, MD


Overview of Endocrine System cont. Cellular Responses to Hormones  A single hormone controls a subset of cellular functions in only cell types that express receptors for that hormone (i.e., target cell)  Specificity of hormonal responses resides in  The structure of hormone itself  The receptor for the hormone, and  The cell type in which the receptor is expressed  Serum hormone concentrations are extremely low (10 -11 to 10-9 M) thus, a receptor must have a high affinity, as well as specificity, for its cognate hormone Marc Imhotep Cray, MD


Overview of Endocrine System cont. Cellular Responses to Hormones (2)  Hormone receptors fall into two general classes: transmembrane receptors and intracellular receptors (belong to nuclear hormone receptor family) Transmembrane Receptors  Most hormones are proteins, peptides, or catecholamines that cannot pass through cell membrane must interact with transmembrane protein receptors  Transmembrane receptors are proteins that contain three domains: (1)an extracellular domain that harbors a high-affinity binding site for a specific hormone (2) one to seven hydrophobic, transmembrane domains that span cell membrane, and (3)a cytosolic domain that is linked to signaling proteins Marc Imhotep Cray, MD


Overview of Endocrine System cont. Cellular Responses to Hormones (3) Hormone binding to a transmembrane receptor induces a conformational shift in all three domains of receptor protein  hormone receptor binding–induced conformational change is referred to as a signal  The signal is transduced into activation of one or more intracellular signaling molecules (protein) Intracellular signaling molecules then act on effector proteins, which, in turn, modify specific cellular functions Marc Imhotep Cray, MD


Overview of Endocrine System cont. Cellular Responses to Hormones (4) Example of hormone induced conformational change in transmembrane receptor.

ď ą This often promotes dimerization of receptors as well as conformational changes in cytosolic domain that unmasks a specific activity (e.g., tyrosine kinase activity) White BA & Porterfield SP. Endocrine and Reproductive Physiology, 4th ed. (Mosby physiology monograph series). Mosby, 2013. Marc Imhotep Cray, MD


Overview of Endocrine System cont. Cellular Responses to Hormones (5) Transmembrane receptor cont.

The combination of…  hormone receptor binding (signal)  activation of signaling molecules (transduction), and  the regulation of one or more effector proteins …is referred to as a signal transduction pathway and the final integrated outcome is referred to as the cellular response Marc Imhotep Cray, MD


Covalent phosphorylation of proteins & lipids Enzymes that phosphorylate proteins or lipids are called kinases, whereas Enzymes that catalyze dephosphorylation are called phosphatases Protein kinases and phosphatases can be classified as either  tyrosine-specific kinases and phosphatases or  serine/threonine-specific kinases and phosphatases

Phosphorylated state of a signaling component alters activity  Phosphorylation can activate or deactivate a substrate  proteins often have multiple sites of phosphorylation that induce quantitative and (or) qualitative changes in protein’s activity Marc Imhotep Cray, MD


Covalent phosphorylation of proteins & lipids (2)  Protein, peptide, and catecholamine hormones signal through transmembrane receptors and use several common forms of informational transfer:  Conformational change  Binding by activated G proteins  Binding by Ca2+ or Ca2+ -calmodulin  IP3 is a major lipid messenger that increases cytosolic Ca2+ levels through binding to IP3 receptor  Phosphorylation and dephosphorylation, using kinases and phosphatases, respectively  Phosphorylation state of a protein affects 

activity, stability, subcellular localization, and recruitment binding of other proteins

Marc Imhotep Cray, MD


Phosphorylation/ dephosphorylation in signal transduction pathways

Phosphotyrosine is shown. White BA & Porterfield SP. Endocrine and Reproductive Physiology, 4th ed. (Mosby physiology monograph series). Mosby, 2013.

Marc Imhotep Cray, MD


“A general principles refresher.”

Re of: Mechanisms of Drug Action (MOA)  Drugs act at four different levels: 1) Molecular: protein molecules are the immediate targets for most drugs. Action here translates into actions at next level. 2) Cellular: biochemical and other components of cells participate in the process of transduction. 3) Tissue: the function of heart, skin, lungs, etc., is then altered. 4) System: the function of the cardiovascular, nervous, gastrointestinal system, etc., is then altered.

Marc Imhotep Cray, MD

38


“A general principles refresher.”

Mechanisms of Drug Action (2)  To most clearly understand pharmacologic actions of drugs (e.g., hormone analogs, agonist and antagonist) it is necessary to know:  which molecular targets are affected by the drug,  the nature of this molecular interaction,  the nature of the transduction system (the cellular response),  the types of tissue that express the molecular target and  the mechanisms by which the tissue influences the body system NB: You are learning pharmacology in a format that integrates the actions of drugs  from the level of molecular targets (biologic effects)  to the level of whole-organism/clinical patient (therapeutic and adverse effects), using kindred sciences (biochem, physio and pathophys. ) as the scaffolding. Marc Imhotep Cray, MD

39


Signal Transduction Pathways Capsule: Question: What are the sequences of events by which binding of a chemical messenger (hormone, neurotransmitter, or paracrine/ autocrine agent) to a receptor causes the cell to respond? Answer: Combination of messenger with receptor causes a change in conformation (three-dimensional shape) of receptor = receptor activation always initial step leading to cell’s responses to messenger  responses (five types of ultimate responses): 1. permeability, transport properties, or electrical state of plasma membrane 2. cell’s metabolism 3. cell’s secretory activity 4. cell’s rate of proliferation and differentiation, or 5. cell’s contractile activity  Despite five different types of responses, there is a common denominator They are all directly due to alterations of particular cell proteins Marc Imhotep Cray, MD


Endocrine Pharmacology Basic Concepts Receptors A receptor is a cell macromolecule either on surface cell or within cytoplasm or nucleus of a cell that is recognized by endogenous or exogenous substances (ligands) with specificity Receptors account for a majority of chemical signalling that occurs within body and are fundamental to ability of chemical messengers to alter function of living cells There are four major families of receptors:  (LGICs) ligand-gated ion channels (e.g. nicotinic ion channel)  (GPCRs) G-protein-coupled receptors (e.g. β-adrenoceptor)  (RTKs) tyrosine kinase receptors (e.g. insulin receptor)  (NHRs) Marc Imhotep Cray, MD intracellular receptors (e.g. glucocorticosteroid receptor)

41


Endocrine Pharmacology Basic Concepts (2) Hormone-Receptor Interactions  There are three major biochemical classes of hormones: 1. Proteins/peptides 2. modified amino acids (catecholamines and TH) 3. steroids

 All known hormones, and drugs that mimic hormones, act via one of two basic receptor systems:  

membrane-associated receptors and intracellular receptors

1. Membrane-associated receptors: (peptide & protein hormones) Membrane-associated receptors bind hydrophilic hormones (which penetrate plasma membrane poorly), such as  Insulin  Adrenocorticotropic hormone (ACTH), and Marc Imhotep MD  Cray,Epinephrine, outside the cell

42


Endocrine Pharmacology Basic Concepts (3) Hormone-Receptor Interactions cont. 1. Membrane-associated receptors transmit signals into cell by a variety of “second messenger” mechanisms, including:  Changes in cyclic adenosine monophosphate (cAMP) or cyclic guanosine monophosphate (cGMP) caused by changes in activity of cyclases  Increased phosphoinositide turnover via increased phospholipase activity  Increased intracellular Ca2+ by action on Ca2+ channels  Increased tyrosine phosphorylation on specific proteins by action of tyrosine kinases (TKR)

Marc Imhotep Cray, MD

43


What are the four primary classes of membrane-spanning receptors to which peptide hormones & NTs bind?  The four primary classes of membrane-spanning receptors to which peptide hormones & neurotransmitters bind are (Illust. next slide): 1) tyrosine and serine kinase receptors 2) receptor-linked kinases 3) G protein–coupled receptors, and 4) ligand-gated ion channels  “Prototypical” agonists (respectively) for the above receptor types: 1) insulin, growth factors (IGF-1, PDGF, EPO etc. ) 2) growth hormones (GHs), prolactin, cytokines (activate receptors of JAK/STAT superfamily) 3) peptide hormones, neurotransmitters and prostaglandins 4) neurotransmitters, amino acids Marc Imhotep Cray, MD

44


The four major classes of membrane receptors for peptide hormones and neurotransmitters

Marc Imhotep Cray, MD

Brown TA, Brown D. USMLE Step 1 Secrets, 3rd Ed. Saunders, 2013

45


Hormone 2nd messenger systems (signal transduction)

McInnis M., Mehta S. Step-up to USMLE Step 1 2015 Edition. Wolters Kluwer, 2015

*Mechanism using a G protein, as shown in D (Next slide)


Transmembrane receptor families: GPCRs  G-protein–coupled receptors, compose largest class of receptors, mediate effects of NTs, hormones, and drugs  All receptors have seven transmembrane segments, three intracellular loops, and an intracellular carboxy-terminal tail  The biologic activity of receptors is mediated via interaction with a number of G (GTP binding) proteins Raff RB, Rawls SM, Beyzarov EP. Netter's Illustrated Pharmacology, Updated Edition. Saunders, 2014


Transmembrane receptor families: GPCRs (2)  G-protein-coupled receptors (GPCRs) act as guanine nucleotide exchange factors (GEFs) to activate Gα subunit of heterotrimeric α/β/Ƴ G-protein complex  Depending on type of Gα subunit that is activated, this will increase cAMP levels, decrease cAMP levels, or increase protein kinase C activity and Ca2+ levels  All catecholamine receptors (adrenergic receptors) are GPCRs  Endocytosis results in lysosomal clearance of hormone  Receptor digested in lysosome or recycled to cell membrane Marc Imhotep Cray, MD


Hormone

nd 2

messenger systems, GPCRs

G-protein mechanism. 1. Messenger system before hormone binding 2. After hormone binding, GTP replaces GDP on G protein 3. GTP, attached to α subunit, dissociates from β−γ complex and converts ATP to cAMP

4. Hormone is released from binding site and complex returns to inactive state when GTPase cleaves GTP to GDP

Marc Imhotep Cray, MD

McInnis M., Mehta S. Step-up to USMLE Step 1 2015 Edition. Wolters Kluwer, 2015

49


Hormone 2nd messenger system: G-Protein Classes G Protein Class

Action

Examples

ATP, adenosine triphosphate; cAMP, cyclic adenosine monophosphate; DAG, diacylglycerol; IP3, inositol triphosphate; PIP2, phosphatidylinositol 4,5-bisphosphate. Modified from: McInnis M., Mehta S. Step-up to USMLE Step 1 2015 Edition. Wolters Kluwer, 2015 50


Basic Concepts (4) Hormone Receptors cont. 2. Intracellular receptors: (steroid hormones, TH, retinol & vitamin D) Intracellular receptors bind hydrophobic (lipophilic) hormones (which penetrate plasma membrane easily) such as        

Cortisol Aldosterone Estrogen Progesterone Testosterone T3/T4 Retinol vitamin D

inside cell-either in cytoplasm or nucleus  Intracellular receptors modulate transcription rate of specific Marc Imhoteptarget Cray, MD genes to change levels of cellular proteins

51


Summary: molecular-cellular mechanisms of hormone action (illustrations next 5 slides)

ď ą Hormones act on their specific receptors located on or within their target cells ď ą Receptor activation by hormones is translated into responses in a variety of ways 1. At cell membrane receptors (proteins and peptides hormones) 2. At cytoplasmic receptors (steroids, T4/T3, Vit. D, retinol) 3. At nuclear receptor

Marc Imhotep Cray, MD

52


Summary: Sites and mechanisms of hormone action 1. At cell membrane receptors

a. Through alteration of intracellular cAMP concentration alteration of protein kinase A regulation of cell function: Ca2+ acting as third messenger in some situations 

Epinephrine, Glucagon, TSH, FSH, LH , PTH, Calcitonin, ACTH, some hypothalamic releasing hormones, Vasopressin (V2) Raff RB, Rawls SM, Beyzarov EP. Netter's Illustrated Pharmacology, Updated Edition. Saunders, 2014

Marc Imhotep Cray, MD

53


Summary: Sites and mechanisms of hormone action 1. At cell membrane receptors

b. Through IP3 DAG generation: release of intracellular Ca2+ and protein kinase C activation  Vasopressin (V1) ,Oxytocin Raff RB, Rawls SM, Beyzarov EP. Netter's Illustrated Pharmacology, Updated Edition. Saunders, 2014

Marc Imhotep Cray, MD

54


Summary: Sites and mechanisms of hormone action 1. At cell membrane receptors

c. Direct transmembrane activation of tyrosine protein kinase phosphorylation cascade regulation of various enzymes  Insulin, Growth hormone , Prolactin Raff RB, Rawls SM, Beyzarov EP. Netter's Illustrated Pharmacology, Updated Edition. Saunders, 2014

Marc Imhotep Cray, MD

NB: Cytoplasmic (Nonreceptor) tyrosine kinase: Prolactin, Immunomodulators (eg, cytokines IL-2, IL-6, IFN), GH, GCSF, Erythropoietin, Thrombopoietin JAK/STAT pathway Think acidophils and cytokines Receptor tyrosine kinase Insulin, IGF-1, FGF, PDGF, EGF MAP kinase pathway Think growth factors 55


Summary: Sites and mechanisms of hormone action 2. At cytoplasmic receptors Penetrating cell membrane, hormone combines with a cytoplasmic receptor  exposes its DNA binding domain migrates to nucleus and binds to specific genes DNA mediated mRNA synthesis synthesis of functional proteins  Steroidal hormones: Glucocorticoids, Mineralocorticoid, Androgens, Estrogens , Progestins  Calcitriol (also called 1,25dihydroxycholecalciferol or 1,25dihydroxyvitamin D) Marc Imhotep Cray, MD

Brunton LL, Chabner BA , Knollmann BC (Eds.). Goodman and Gilman’s The Pharmacological Basis of Therapeutics. 12th ed. McGraw-Hill, 2011

56


Summary: Sites and mechanisms of hormone action 3. At nuclear receptor

Marc Imhotep Cray, MD

Raff RB, Rawls SM, Beyzarov EP. Netter's Illustrated Pharmacology, Updated Edition. Saunders, 2014

Hormone penetrates nucleus, combines with its receptor alters DNA- RNA mediated protein synthesis  Thyroid hormones: Triiodothyronine, Thyroxine  Estrogen, testosterone, glucocorticoids, vitamin D, aldosterone, progesterone

57


Receptor Families and Signaling Pathways: Class of Receptors Used by Various Hormones Receptor Class

Hormones and Related Substances

cAMP

LH, FSH, ACTH, TSH, PTH, hCG, CRH, glucagon, ADH (V2)

cGMP

NO, ANP

IP3

GnRH, GHRH, oxytocin, TRH, ADH (V1)

Steroid receptor (intracellular)

Estrogen, testosterone, glucocorticoids, vitamin D, aldosterone, progesterone, T3/T4

Tyrosine kinase

Insulin, growth factors (e.g., IGF, PDGF), GH, prolactin

Redrawn after: Brown TA, Brown D. USMLE Step 1 Secrets, 3rd Ed. Saunders, 2013

Ledger: ACTH, adrenocorticotropic hormone; ANP, atrial natriuretic peptide; cAMP, cyclic adenosine monophosphate; cGMP, cyclic guanosine monophosphate; CRH, corticotropin-releasing hormone; FSH, follicle-stimulating hormone; GH, growth hormone; GHRH, growth hormone–releasing hormone; GnRH, gonadotropin-releasing hormone; hCG, human chorionic gonadotropin; IGF, insulin-like growth factor; IP3, inositol triphosphate; NO, nitric oxide; PDGF, platelet-derived growth factor; PTH, parathyroid hormone; T3, triiodothyronine; T4, thyroxine; TRH, thyrotropin-releasing hormone; TSH, thyroid stimulating hormone. Marc Imhotep Cray, MD

58


Hormone-Receptor Interactions cont. ď ą We have just completed a discussion of hormonereceptor interactions from the vantage point of the receptor. ď ą In the next 9 slides we will view the interaction from the hormone (ligand) side of the interaction.

Marc Imhotep Cray, MD

59


MOA of peptide hormones & catecholamines  Peptide hormones and catecholamines are not highly lipid-diffusible and thus cannot cross plasma membrane  They bind to cell surface membrane receptors which initiate a variety of biochemical events, including: o activation or inhibition of enzymes o alteration of membrane proteins, and o mediation of cellular trafficking  These processes can occur within seconds to minutes o Nevertheless, peptide hormones can stimulate gene expression as well, and this effect can be delayed as it is with steroid hormones  Examples of peptide hormones are insulin, parathyroid hormone (PTH), vasopressin (antidiuretic hormone), and oxytocin  Examples of catecholamines norepinephrine, epinephrine, and dopamine Marc Imhotep Cray, MD

60


Characteristics of Protein/Peptide Hormones  Synthesized as prehormones or preprohormones  Stored in membrane-bound secretory vesicles (sometimes called secretory granules)  Regulated at level of secretion (regulated exocytosis) and synthesis  Often circulate in blood unbound  Usually administered by injection

 Hydrophilic and signal through transmembrane receptors Marc Imhotep Cray, MD


Characteristics of Catecholamines  Derived from enzymatic modification of tyrosine  Stored in membrane-bound secretory vesicles  Regulated at level of secretion (regulated exocytosis) and through regulation of enzymatic pathway required for their synthesis

 Transported in blood free or only loosely associated with proteins  Often administered as an aerosol, and several specific analogs (agonists and antagonists) can be taken orally  Hydrophilic and signal through transmembrane G-protein-coupled receptors Marc Imhotep Cray, MD called adrenergic receptors


Cellular MOA of steroid & TH hormones  Steroid hormones are lipophilic they diffuse across plasma membrane and form complexes with cytosolic or nuclear receptors bound complexes then activate transcription of various genes  Because steroid hormones rely on the intermediary process of gene expression and protein translation it can take hours to days for their effects to manifest  Examples of steroid hormones are testosterone, estrogen, progesterone, cortisol, and aldosterone  Cholesterol is precursor to all steroid hormones  Although thyroid hormone is not a steroid hormone TH, nonetheless uses same cellular mechanism as steroids Marc Imhotep Cray, MD

63


Transport of steroid and thyroid hormones Why are total serum steroid hormone & TH levels not an accurate reflection of hormone activity?  Most of steroid hormones & TH in serum are inactive because they are attached to serum binding proteins  Only free hormone is biologically active Free hormone is in equilibrium with bound hormone: [Free hormone] + [Binding protein] [Hormone-binding protein complex] For example: In circulation, T3/T4 exist in both active free and inactive protein-bound forms  T4 is 99.98% bound, with only 0.02% circulating free.  T3 is slightly less protein bound (99.8%), resulting in a considerably higher circulating free fraction (0.2%) Marc Imhotep Cray, MD

64


Transport of steroid and thyroid hormones (2) Binding of steroid and thyroid hormones to plasma proteins has several beneficial effects, including:  Facilitation of transport  Prolonged half-life  Hormone reservoir  Steroid and thyroid hormones are minimally soluble in blood binding to plasma proteins renders them water soluble and facilitates their transport  Protein binding prolongs circulating half-life of these hormones e.g., not filtered/excreted by kidney

 Protein-bound form of hormone serves as a “reservoir” of hormone that minimizes changes in free hormone concentration when hormone secretion from Cray, its MD endocrine gland changes abruptly Marc Imhotep

65


Characteristics of Steroid Hormones  Derived from enzymatic modification of cholesterol  Cannot be stored in secretory vesicles because of lipophilic nature  Regulated at level of enzymatic pathway required for their synthesis  Transported in blood bound to transport proteins (binding globulins)  Signal through intracellular receptors (nuclear hormone receptor family)  Can be administered orally

Marc Imhotep Cray, MD


Characteristics of Thyroid Hormones  Derived from iodination of thyronines  Lipophilic, but stored in thyroid follicle by covalent attachment to thyroglobulin

 Regulated at level of synthesis, iodination, and secretion  Transported in blood tightly bound to proteins  Signal through intracellular receptors (nuclear hormone receptor family)  Can be administered orally

Marc Imhotep Cray, MD


To summarize: Mechanisms by which peptide/amine and steroid hormones signal Remember: ď ą amino acid derivatives and peptide/protein hormones are water-soluble and interact with cell-surface membrane receptors ď ą Steroids, thyroid hormones, vitamin D, and retinoids are lipidsoluble and interact with intracellular(cytoplasmic & nuclear) receptors Marc Imhotep Cray, MD

Brown TA, Brown D. USMLE Step 1 Secrets, 3rd Ed. Saunders, 2013

68


Summary of distinguishing features of steroid, protein/peptide, and amine hormones

Kelly LJ. Essentials of Human Physiology for Pharmacy. Boca Raton: CRC Press, 2004.

69


Functional anatomy of endocrine and metabolic systems  Endocrine and metabolic systems regulate seven major bodily functions (detail slides follow)  For each target tissue effect, endocrine glands release hormones in response to regulating factors, which include  physiologic (e.g. sleep and stress),  biochemical (e.g. glucose and Ca2+) and  hormonal (e.g. hypothalamic and enteric hormones) stimuli

Marc Imhotep Cray, MD

70


Functional anatomy of endocrine and metabolic systems (2)  Endocrine and metabolic system consists of a variety of organs (glands) that secrete substances (hormones) into blood which affect function of target tissues elsewhere in body  Glands include hypothalamus, pituitary, thyroid, adrenals, gonads, pancreatic islets of Langerhans and parathyroids  Endocrine system regulates seven major physiologic functions: 1) Availability of metabolic energy (fuel), 2) Metabolic rate, 3) Circulatory volume, 4) Somatic growth, 5) Calcium homeostasis , 6) Reproductive function 7) Adaptation to stress

 A cardinal feature of drug therapy of endocrine diseases is interaction between exogenously administered drugs and endogenous biochemistry, physiology and (pathophysiology) of hormones Marc Imhotep Cray, MD

71


Functional anatomy of endocrine and metabolic systems (3) Endocrine function 1. Availability of metabolic energy (fuel) Regulatory factors Serum glucose, amino acids, enteric hormones (somatostatin, cholecystokinin, gastrin, secretin), vagal reflex, sympathetic nervous system Endocrine organ / hormone Pancreatic islets of Langerhans/insulin, glucagon Target tissues All tissues, especially liver, skeletal muscle, adipose tissue, indirect effects on brain and red blood cells Marc Imhotep Cray, MD

72


Functional anatomy of endocrine and metabolic systems (4) Endocrine function 2. Metabolic rate Regulatory factors Hypothalamic thyrotropin-releasing hormone (TRH), pituitary thyrotropin (TSH) Endocrine organ / hormone Thyroid gland/triiodothyronine (T3) Target tissues All tissues Marc Imhotep Cray, MD

73


Functional anatomy of endocrine and metabolic systems (5) Endocrine function 3. Circulatory volume Regulatory factors Renin, angiotensin II, hypothalamic osmoreceptors Endocrine organ / hormone Adrenals /aldosterone, Pituitary/vasopressin Target tissues Kidney, blood vessels, CNS

Marc Imhotep Cray, MD

74


Functional anatomy of endocrine and metabolic systems (6) Endocrine function 4. Somatic growth Regulatory factors Hypothalamic growth hormone-releasing hormone (GHRH), somatostatin, sleep, exercise, stress, hypoglycemia Endocrine organ / hormone Pituitary/growth hormone, Liver/insulin-like growth factors (IGFs) Target tissues All tissues Marc Imhotep Cray, MD

75


Functional anatomy of endocrine and metabolic systems (7) Endocrine function 5. Calcium homeostasis Regulatory factors Serum Ca+ + and Mg++ concentration Endocrine organ / hormone Parathyroid glands/parathyroid hormone, calcitonin, vitamin D Target tissues Kidney, intestines, bone

Marc Imhotep Cray, MD

76


Functional anatomy of endocrine and metabolic systems (8) Endocrine function 6. Reproductive function Regulatory factors Hypothalamic gonadotropin- releasing hormone (GnRH), pituitary, follicle stimulating hormone (FSH) and luteinizing hormone (LH), inhibins Endocrine organ / hormone Gonads / sex steroids, Adrenals/ androgens Target tissues Reproductive organs, CNS, various tissues Marc Imhotep Cray, MD

77


Functional anatomy of endocrine and metabolic systems (9) Endocrine function 7. Adaptation to stress Regulatory factors Hypothalamic corticotropin- releasing hormone (CRH), pituitary adrenocorticotropic hormone (ACTH), hypoglycemia, stress Endocrine organ / hormone Adrenals/glucocorticosteroids, epinephrine Target tissues Many tissues: CNS, liver, skeletal muscle, adipose tissue, lymphocytes, fibroblasts, cardiovascular system Marc Imhotep Cray, MD

78


Hormones of hypothalamic-pituitary axis Individual Axes: (Hormonal Feedback Regulatory Systems) Anterior Pituitary Gland Hypothalamic-Pituitary–Growth Hormone Axis Hypothalamic-Pituitary–Prolactin Axis Hypothalamic-Pituitary–Thyroid Axis Hypothalamic-Pituitary–Adrenal Axis Hypothalamic-Pituitary–Gonadal Axis Posterior Pituitary Gland Antidiuretic Hormone (ADH) Oxytocin

Marc Imhotep Cray, MD

McInnis M., Mehta S. Step-up to USMLE Step 1 2015 Edition. Wolters Kluwer, 2015

79


Hypothalamic-pituitary signaling pathways Response of an anterior pituitary gland cell to a hypothalamic factor (neurohormone) is initiated when hypothalamic factor (a peptide) binds to specific G protein-coupled receptors located on plasma membrane of appropriate anterior pituitary cell type  Most of these receptors alter levels of intracellular cAMP or IP3 and calcium  Molecular details of receptor signaling provide a biochemical basis for understanding hypothalamic factor action (example in next slide) Marc Imhotep Cray, MD

80


Hypothalamic-pituitary signaling pathways (2) For example:  Growth Hormone-Releasing Hormone (GHRH) binding to its receptors on somatotrophs increases intracellular cAMP and Ca2+ levels, whereas  Somatostatin (Somatotropin ReleaseInhibiting Hormone, SRIH) binding to its receptors on somatotrophs decreases intracellular cAMP and Ca+2  These signaling pathways provide a biochemical explanation for opposing activities GHRH and somatostatin on somatotroph release of GH

Costanzo LS. Physiology (Basic Review Series), 5th ed. New York: 81 Elsevier, 2009.


Hormonal Feedback Regulatory Systems  Feedback control , both negative and positive, is a fundamental feature of endocrine systems  Each of major hypothalamic-pituitary- hormone axes is governed by negative feedback, a process that maintains hormone levels within a relatively narrow range (set-points or set ranges)

Examples of hypothalamic-pituitary negative feedback include (1) thyroid hormones on TRH-TSH axis (2) cortisol on CRH-ACTH axis (3) gonadal steroids on GnRH-LH/FSH axis, and (4) IGF-1 on GHRH-GH axis  These regulatory loops include both positive (e.g., TRH, TSH) and negative (e.g., T 4 , T 3 ) components, allowing for precise control of hormone levels Marc Imhotep Cray, MD

82


Feed-forward and Feed-back Mechanisms  As discussed previously, key to Hypothalamic Releasing Hormone understanding endocrine pharmacology is to be clear on Pituitary Tropic (Signal) Hormone feed-forward and feed-back mechanisms  Feedback regulation is particularly Target Glands critical to physiologic control of thyroid, adrenal cortical, and Second-tier Hormone gonadal function Negative  and is also important in Feedback pharmacologic treatments that Organ-System Effect affect these systems Marc Imhotep Cray, MD

83


Negative and Positive Feedback Regulation  In most cases, a hypothalamic–pituitary–target gland axis is regulated by negative feedback, whereby tropic hormone of anterior pituitary gland has negative feedback effects on hypothalamus and target gland hormone has negative feedback effects on both hypothalamus and anterior pituitary  By way of these mechanisms levels of target gland hormone are maintained within normal physiological range NB: Positive Feedback Although negative feedback is the primary homeostatic mechanism in endocrine system, rare examples of positive feedback exist (e.g., menstrual cycle). Marc Imhotep Cray, MD

84


Example of positive feedback  Prime example of positive feedback occurs during menstrual cycle  In late follicular phase of cycle, estradiol levels rise above a critical point, above which positive feedback occurs  High estradiol concentration results in a surge in hypothalamic secretion of GnRH and pituitary secretion of LH and FSH, inducing ovulation  Ovulation and transformation of ovarian follicular cells into corpus luteum signals end of positive feedback Marc Imhotep Cray, MD

85


Concept of Feedback Loop What is a feedback loop? Hormone synthesis and release are governed at multiple levels  Hormone synthesis and release (secretion) from an organ of interest typically involves regulation by a pituitary hormone, which itself is regulated by a hypothalamic hormone This general pathway structure is commonly referred to as a hypothalamic-pituitary-(organ) axis  e.g., HPO axis refers to ovary, HPA axis refers to adrenal gland These relationships are often depicted using feedback loops (next slide)

Marc Imhotep Cray, MD

86


Regulation of hormone synthesis and secretion cont.

 A negative feedback mechanism is an example of a negative effect

Solid lines = positive effect Dashed lines = negative effect

 It is essential to understand “the negative feedback principle” of hypothalamic /pituitary/ target organ axis

 Negative feedback occurs when a product downstream of an axis inhibits production of a reactant by which it is regulated  for example, TH inhibition TSH Marc Imhotep Cray, MD

Pazdernik TL, Kerecsen L. Rapid Review Pharmacology, 3rd Ed. Mosby, 2010

87


Basic Pathophysiologic and Pharmacologic Concepts  Endocrine systems regulating    

metabolic rate (thyroid hormone) reproductive function (sex steroids) adaptation to physiologic stress (glucocorticosteroids) and somatic growth (growth hormone-IGF axis)

share common disease patterns affecting each level of regulation  While disease at any level in regulatory system may produce a similar effect (i.e. hypo- or hyperstimulation of end-organ effects) different approaches to drug therapy are preferred depending on site of pathology 

Example: hypogonadism due to failure of pituitary gonadotrophs responds therapy with exogenous gonadotropins, but gonadal failure will not Marc Imhotep Cray, MD


Basic Pathophysiologic and Pharmacologic Concepts (2) Hypopituitarism may be partial or complete and may result from hypothalamic disease (leading to deficiency of hypothalamic-releasing hormones) or intrinsic pituitary disease(causing pituitary hormone deficiency)

Hypopituitarism may affect any of these pituitary hormones:  thyrotropin (TSH)  growth hormone(GH)  luteinizing hormone (LH)  follicle stimulating hormone (FSH) and  corticotropin (ACTH) Marc Imhotep Cray, MD

89


Basic Pathophysiologic and Pharmacologic Concepts (3)  In targeting one of these hormones of hypopituitarism  therapy for GH deficiency aims to restore normal body composition, as well as, in children, to promote linear growth  Therapy for acromegaly, caused by excessive GH secretion, includes  surgery and (or) radiation, or  use of a GH inhibitor o Octreotide o Lanreotide o Pegvisomant Marc Imhotep Cray, MD

90


Basic Pathophysiologic and Pharmacologic Concepts (4)

Hypothyroidism can result from either thyroid (high TSH, low T3 &T4) or hypothalamic (or) pituitary dysfunction (low T3, T4, TSH)  Treatment of choice is hormone substitution by using synthetic thyroid hormone Hyperthyroidism (thyrotoxicosis) is characterized by increased metabolism, and primary treatment options include  surgery  radioactive iodine or  drugs that inhibit formation of T3 &T4 by blocking utilization of iodine (Thioamides)=Methimazole, PTU Marc Imhotep Cray, MD

91


Basic Pathophysiologic and Pharmacologic Concepts (5) Principal functions of glucocorticoids involve regulation of carbohydrate metabolism and a variety of other physiologic actions Synthetic corticosteroids (eg, hydrocortisone, prednisone, and dexamethasone) are widely used as therapeutic agents in Tx of cancer and autoimmune or inflammatory-type disorders Pharmacologic treatment is also available for  insufficient adrenal function manifested as Addison disease  excess glucocorticoid exposure results in Cushing syndrome Marc Imhotep Cray, MD

92


Basic Pathophysiologic and Pharmacologic Concepts (6) Diabetes mellitus (DM) is a syndrome caused by a relative or absolute deficiency of insulin, with hyperglycemia being hallmark medical finding  DM can occur as either an early onset form (type 1) or a gradual-onset form (type 2)  In T1DM, insulin-producing β cells of pancreas are destroyed or insufficiently active, and patients require lifelong treatment with exogenous insulin  In T2DM, adequate control of disease may be achieved by means of diet and exercise if these methods fail, patients take oral hypoglycemic agents, which cause o lower plasma glucose levels o improve insulin resistance, and o reduce long-term complications (microvascular and macrovascular such as neuropathy, nephropathy, retinopathy and CVD) Marc Imhotep Cray,problems MD

93


Basic Pathophysiologic and Pharmacologic Concepts (7)  For type 1 DM Insulin is sole treatment and is sometimes also used for type 2 DM  For type 2 DM, drugs (oral hypoglycemic agents) include  sulfonylureas, which stimulate insulin secretion from pancreatic β cells  metformin, a biguanide that decreases blood glucose levels by reducing hepatic glucose production and glycogen metabolism in liver and improving insulin resistance  meglitinides, which increase insulin secretion from pancreatic β cells  α-glucosidase inhibitors, which delay carbohydrate digestion and glucose absorption and  thiazolidinedione (TZD) derivatives (eg, rosiglitazone and pioglitazone), which reduce insulin resistance Marc Imhotep Cray, MD

94


Basic Pathophysiologic and Pharmacologic Concepts (8) ď ą Diagnostic strategies in endocrine disease attempt to identify site of pathology by identifying pattern of hormonal responses characteristic for different diseases ď ą Primary alterations and compensatory responses of regulatory hormones accompanying different patterns of endocrine disease must be understood to allow both, accurate diagnosis and treatment

Marc Imhotep Cray, MD


Basic Pathophysiologic and Pharmacologic Concepts (9) Strategies to Manage the Levels and Action of Hormones

Mechanisms to Increase Hormone Levels and Activity  Increase endogenous hormone synthesis, release, and transport  Reduce endogenous hormone metabolism and excretion  Increase peripheral activation of circulating hormone (if required)  Hormone replacement therapy Mechanisms to Decrease Hormone Levels and Activity  Lower endogenous hormone synthesis, release, or both  Reduce peripheral conversion to activated forms  Promote hepatic/renal metabolism/excretion  Decrease receptor activity by reducing receptor number or affinity for hormone or use competitive receptor antagonists  Suppress response of target tissue to receptor-hormone interaction by interfering with generation of second messengers  Cray, Modify tissue metabolism to blunt the effects of hormone excess Marc Imhotep MD


Thumbnail: Some Drugs Known to Affect Hormonal Balance Effectors of Hormone Release/Reuptake Bromocriptine—antagonizes release of GH and prolactin Octreotide—inhibits selective release of GH Analogs of GnRH—elevated levels desensitize anterior pituitary; pulsatile exposure to physiological levels simulates GnH release Sulfonylureas, Meglitinides and Incretins—promote insulin release from pancreatic beta cells Pramlintide—inhibits glucagon secretion Alteration of Peripheral Conversion of Hormones Finesteride—blocks conversion of testosterone to 5α dihydrotestosterone Aromatase inhibitors—antagonize interconversion of estrogen and androgens Propylthiouracil—blocks conversion of thyroxine to triiodothyronine in tissues Dipeptidyl peptidase-IV inhibitors—block digestion of incretins

Competitive Receptor Antagonists Spironolactone—aldosterone receptor antagonist Raloxifene—estrogen receptor tissue-specific agonist/ antagonist Tamoxifen/Clomiphene—estrogen receptor agonist/ antagonist Mifprostone—progesterone receptor antagonist Danazol/ Cyproterone acetate/Flutamide—androgen receptor antagonists Alteration of Metabolism Metformin—decreases hepatic glucose production Thiazolidinediones—improves insulin-facilitated metabolic effects in patients with insulin resistance Bisphosphonates—cytotoxic effects on osteoclasts Effectors of Hormone Synthesis Thioamides—inhibit synthesis of thyroid hormones Metyrapone—inhibits synthesis of cortisol

Lynn Wecker et.al. Brody’s Human Pharmacology: Molecular to Clinical, 5th Ed. Philadelphia: Mosby, 2010 97


Lect. 2 of 6

Hypothalamic and Pituitary Disorders Hormones produced by hypothalamus and pituitary gland are key regulators of metabolism, growth, and reproduction. Preparations, including products made by recombinant DNA technology and drugs that mimic or block their effects, are used in treatment of a variety of endocrine disorders. Three concepts are of special importance in this presentation: (1) hypothalamic control of pituitary hormone release (2) negative feedback inhibition and (3) endocrine axes Marc Imhotep Cray, MD

98


Learning Objectives Neuroendocrine Pharmacology: Hypothalamic and Pituitary Hormones 1. The physiology of neuroendocrine hormonal regulation, including a) Hypothalamus-Pituitary-Growth Hormone Axis, b) Hypothalamus-Pituitary-Reproductive Axis, c) Hypothalamus-Pituitary-Prolactin Axis 2. The use of specific neuroendocrine agents in treatment of following neuroendocrine disorders: a) growth hormone deficiency b) growth hormone excess c) infertility d) hyperprolactinemia 3. Indications, mechanism of action, adverse effects, contraindications and therapeutic considerations for major neuroendocrine hormones and pharmacological agents. Marc Imhotep Cray, MD

99


Baron SJ and Lee CI. Lange Pathology Flash Cards. New York: McGraw-Hill, 2009 Marc Imhotep Cray, MD

100


Some hormones and drugs affecting the hypothalamus and pituitary glands HYPOTHALAMIC AND ANTERIOR PITUITARY HORMONES

HCG Corticotropin Cosyntropin Urofollitropin Follitropin alfa Follitropin beta Goserelin Histrelin Leuprolide Menotropins Nafarelin Pegvisomant Somatropin Marc Imhotep Cray, MD

Corticorelin (CRH) Gonadorelin (GnRH) Octreotide Somatostatin Triptorelin GnRH antagonists Ganirelix Cetrorelix

HORMONES OF THE POSTERIOR PITUITARY

Desmopressin DDAVP Oxytocin Vasopressin (ADH)

NB (note well [Lat. nota bene])  The ending –relin indicates a hypothalamus-related hormone.  Drugs that end in –tropin are related to the pituitary hormones.

101


Case 39 Drugs Active on the Hypothalamus and Pituitary Gland A 67-year-old man complains of pain in his right hip for the past few weeks. He has had no injury to the area and describes the pain as a “bone ache� that does not radiate. Review of systems is positive only for some weakness of urinary stream and having to get up twice a night to go to the bathroom. His general physical examination is normal. His hip examination is normal with a full range of motion and no tenderness. Examination of his prostate reveals it to be firm, enlarged, and nodular. Blood tests show a markedly elevated prostate-specific antigen (PSA), and biopsy of the prostate shows carcinoma. A bone scan confirms the presence of metastatic disease in the right hip. Along with other adjuvant therapies, a decision is made to start depot leuprolide acetate. _ Leuprolide acetate is an analog of which hypothalamic hormone? _ What is the mechanism of action of leuprolide acetate? _ Which pituitary hormones are affected by leuprolide acetate, and how are they affected? Marc Imhotep Cray, MD

102


Relationship between Hypothalamus and Pituitary Gland  Hypothalamus and pituitary gland function cooperatively as master regulators of endocrine system  Together, hormones secreted by hypothalamus & pituitary control important homeostatic and metabolic functions, including:  reproduction  growth  lactation  thyroid gland physiology  adrenal gland physiology and  water homeostasis Marc Imhotep Cray, MD

103


Relationships among hypothalamic and pituitary hormones, and target organs (2)  Posterior pituitary hormones, formed in supraoptic and paraventricular nuclei, are transported by nerve axons to posterior lobe, where they are released by physiologic stimuli  Oxytocin induces milk ejection by breast and stimulates uterine contractions during labor  Vasopressin increases water and sodium reabsorption by the kidneys

Marc Imhotep Cray, MD

104


Relationship between Hypothalamus and Pituitary Gland (3) Although anterior and posterior pituitary glands derive from different embryologic origins, hypothalamus controls activity of both lobes Connection between hypothalamus and pituitary gland is one of most important points of interaction between nervous and endocrine systems  Hypothalamus acts as a neuroendocrine transducer by integrating neural signals from brain and converting those signals into chemical messages (largely peptides) that in turn regulate secretion of pituitary hormones which in turn, alter activities of peripheral endocrine organs Marc Imhotep Cray, MD

105


Relationships among hypothalamic hormones, pituitary hormones, and target organs  Numerous hormone-releasing and hormone-inhibiting factors (neurohormones) formed in arcuate and other hypothalamic nuclei are transported to anterior pituitary by hypophyseal portal system  In response to hypothalamic hormones, anterior pituitary secretes following:  Corticotropin (ACTH) evokes corticosteroid secretion by adrenal cortex  Growth hormone (GH) elicits production of insulin-like growth factors by liver  Follicle-stimulating hormone (FSH) stimulates spermatogenesis and facilitates ovarian follicle development  Luteinizing hormone (LH) elicits testosterone secretion by testes, facilitates ovarian follicle development, and induces ovulation  Thyroid-stimulating hormone (TSH) stimulates thyroxin secretion by thyroid gland  Prolactin (PRL) induces breast tissue growth and lactation 106


Relationships among hypothalamic and pituitary hormones, and target organs (3) Remember: Hypothalamic Releasing Hormone Pituitary Tropic (Signal) Hormone Target Glands Second-tier Hormone Organ-System Effect Negative Feedback

Marc Imhotep Cray, MD

Brenner GM & CW Stevens. Pharmacology , 4th ed. Philadelphia: Saunders, 2013.

107


Anterior Pituitary Gland Cell Types, Hypothalamic Control Factors, and Hormonal Targets

Golan DE et.al. Principles of Pharmacology: The Pathophysiologic Basis of Drug Therapy 3rd Ed. Lippincott Williams & Wilkins, 2012

108


Pharmacologic applications of hypothalamic and pituitary hormones Drugs that mimic or block effects of hypothalamic and pituitary hormones have pharmacologic applications in three primary areas: (1) as replacement therapy for hormone deficiency states (2) as antagonists for diseases caused by excess production of pituitary hormones (3) as diagnostic tools for identifying several endocrine abnormalities

Marc Imhotep Cray, MD

109


Pharmacologic applications of hypothalamic and pituitary hormones (2)  Due to greater ease of administration of target endocrine gland hormones or their synthetic analogs, related hypothalamic and pituitary hormones (i.e., TRH, TSH, CRH, ACTH, GHRH) are used infrequently as treatments  Some, such as CRH and ACTH, are used for specialized diagnostic testing as tools for stimulation to diagnose hypofunctioning or hyperfunctioning endocrine states  In contrast, GH, SST, LH, FSH, GnRH, and dopamine or analogs of these hormones are commonly used therapeutic agents Marc Imhotep Cray, MD

110


Hypothalamus Hypothalamus is a small area, weighing about 4g of the total 1,400g of adult brain weight only 4g of brain without which life itself is impossible Hypothalamus is so critical for life because it contains integrative circuitry that coordinates autonomic, endocrine, and behavioral responses that are necessary for basic life functions, such as  thermoregulation  control of electrolyte and fluid balance  feeding and metabolism  responses to stress, and NB: Hypothalamic regulatory factors are  reproduction peptides (neuropeptides) with exception of Marc Imhotep Cray, MD

dopamine (an amine/tyrosine derivative).

111


Hypothalamus (2) Hypothalamus is not an endocrine gland, but a part of the brain  Nonetheless, the hypothalamus is vital part of endocrine system o This is b/c chemical messengers released by certain neuron terminals in both hypothalamus and its extension, posterior pituitary, do not function as neurotransmitters affecting adjacent cells rather enter blood as neurohormones Bld then carries them to their sites of action

Marc Imhotep Cray, MD


Hypothalamus (3) Hypothalamus is responsive to:        

Light and day length for regulating circadian and seasonal rhythms Olfactory stimuli, including pheromones Steroids including gonadal steroids and corticosteroids Neurally transmitted information arising in heart, stomach & reproductive tract Autonomic inputs Blood-borne stimuli including leptin, ghrelin, angiotensin, insulin, pituitary hormones, cytokines, plasma [glucose] and osmolarity Stress Invading microorganisms by increasing body temperature and resetting body’s thermostat upward

Marc Imhotep Cray, MD

113


Hypothalamus (4)

Marc Imhotep Cray, MD

Felten, DL. Netter's Atlas of Neuroscience, 2nd Ed. Philadelphia, PA: Saunders, 2010

114


Hypothalamus (5)  One of most important functions of hypothalamus is to link nervous system to the endocrine system via pituitary gland (hypophysis)  Hypothalamus is also responsible for certain metabolic processes and other activities of autonomic nervous system  It synthesizes and secretes neurohormones, often called “hypothalamicreleasing hormones” which in turn stimulate or inhibit secretion of pituitary hormones  Hypothalamus also controls body temperature, hunger, thirst, fatigue and circadian rhythm cycles  Hypothalamus synthesizes & secretes the posterior pituitary hormones Marcvasopressin Imhotep Cray, MD [antidiuretic hormone (ADH)] and oxytocin

115


Hypothalamus produces many releasing and inhibitory hormones which control secretion of anterior pituitary hormones

Hypothalamic hormone/factor

Chemical nature

1. Thyrotropin releasing hormone (TRH) 2. Corticotropin releasing hormone (CRH) 3. Gonadotropin releasing hormone (GnRH) 4. Prolactin release inhibitory hormone (PRIH=DA) 5. Growth hormone releasing hormone (GHRH) 6. Somatostatin (Growth hormone release inhibitory hormone/GHRIH)

Tripeptide Peptide (41 AAs) Decapeptide Dopamine Peptide (40, 44 AAs) Peptide (14 AA)

Note: Nearly all hypothalamic hormones stimulate release of pituitary hormones; dopamine and somatostatin are exceptions.  Dopamine (DA) acts as an inhibitory factor, preventing release of prolactin  Somatostatin (SST) prevents release of growth hormone  With exception of dopamine, all hypothalamic releasing factors are peptides

Marc Imhotep Cray, MD

116


Hypothalamic hormonal control of Growth Hormone (GH) and Prolactin (PRL) Hypothalamic hormonal control of GH and PRL (structurally homologous) differs from regulatory systems for TSH, FSH, LH and ACTH (activate G protein-coupled receptors )  GH and PRL are single-chain protein hormones both activate receptors of JAK/STAT superfamily

Growth Hormone (GH) Hypothalamus secretes two hormones that regulate GH  growth hormone-releasing hormone (GHRH) stimulates GH production, whereas  peptide somatostatin (SST) inhibits GH production  GH and its primary peripheral mediator, insulin- like growth factor-I (IGF-I) , also provide feedback to inhibit GH release Marc Imhotep Cray, MD

117


Hypothalamic hormonal control of Growth Hormone (GH) and Prolactin (PRL) cont. Prolactin structurally homologous to growth hormone  Prolactin production is tonically inhibited by catecholamine dopamine acting through D2 subtype of DA receptors  Hypothalamus does not produce a hormone that specifically stimulates prolactin secretion, although TRH can stimulate prolactin release particularly when TRH concentrations are high in setting of primary hypothyroidism

 PRL decreases GnRH, thus In pts w pituitary prolactinoma amenorrhea, osteoporosis, hypogonadism, galactorrhea  DA agonists (eg, bromocriptine) inhibit PRL secretion and can be used in treatment of prolactinoma  DA antagonists (eg, most antipsychotics) and estrogens (eg, OCPs, pregnancy) stimulate prolactin secretion See: Anterior Pituitary Case-based Tutorial 3 Marc Imhotep Cray, MD


Hypothalamic Control of ANS

 Hypothalamus is highest level of neuraxis that provides input to ANS  It regulates virtually all autonomic functions and coordinates them with each other, and with ongoing behavioral, metabolic, and emotional activity  Hypothalamus contains several sets of neurons, using different NTs, that provide innervation to sympathetic and parasympathetic preganglionic neurons, as well as brainstem areas that regulate autonomic nervous system 119


Pituitary gland (hypophysis) Pituitary weighs about 0.6 g and rests at base of brain in bony sella turcica near optic chiasm and cavernous sinuses Pituitary is composed of two lobes:  Anterior pituitary (adenohypophysis)  Posterior pituitary (neurohypophysis)

Pituitary is functionally linked to hypothalamus by pituitary stalk

Neuroendocrine neurons in hypothalamus project axons to median eminence at base of brain  neurons release substances (releasing hormones) into special capillary system, called “hypothalamic-hypophyseal portal system”,  travel to anterior pituitary gland releasing hormones in turn stimulate release ofCray, pituitary hormones from anterior lobe Marc Imhotep MD

120


Pituitary gland (2) From a developmental perspective, pituitary gland consists of two closely associated organs  Anterior pituitary (adenohypophysis) is derived from ectodermal tissue  Rathke’s pouch

 Posterior pituitary (neurohypophysis) is a neural structure derived from ventral surface of diencephalon  a ventral outgrowth of the primitive hypothalamus

 Prefixes adeno- and neuro- denote oral ectodermal and neural ectodermal origin of anterior and posterior pituitary gland components, respectively Marc Imhotep Cray, MD

Raff RB, Rawls SM, Beyzarov EP. Netter's Illustrated Pharmacology, Updated Edition. Saunders, 2014

121


Normal pituitary gland, gross

Klatt EC. Robbins and Cotran Atlas of Pathology, 3rd Ed. Philadelphia: Saunders, 2015.

122


Normal pituitary, microscopic

Klatt EC. Robbins and Cotran Atlas of Pathology, 3rd Ed. Philadelphia: Saunders, 2015.

123


Normal pituitary, microscopic (2)

Klatt EC. Robbins and Cotran Atlas of Pathology, 3rd Ed. Philadelphia: Saunders, 2015.

124


The hypothalamic-pituitary portal system  As stated, neurons in hypothalamus release regulatory factors that are carried by hypothalamicpituitary portal system to anterior pituitary gland, where they control release of anterior pituitary hormones

Note the separate vascular supplies to anterior and posterior lobes of pituitary gland.

 Posterior pituitary hormones are synthesized in cell bodies of supraoptic and paraventricular neurons in hypothalamus then transported down axonal pathways to terminals in posterior pituitary gland  These hormones are stored in posterior pituitary gland, from which they are released into systemic circulation Golan DE et.al. Principles of Pharmacology: The Pathophysiologic Basis of Drug Therapy 3rd Ed. Lippincott Williams & Wilkins, 2012


Regulation of Hypothalamic & Pituitary Hormones  Hormones secreted by hypothalamus and pituitary are all peptides or low molecular weight proteins that act by binding to specific receptor sites on their target tissues

 Hypothalamic hormones trigger release of anterior pituitary hormones which are sent to target organs  where they induce hormone synthesis  Hormones of anterior pituitary are regulated by neuropeptides called either “releasing” or “inhibiting” factors or hormones produced in hypothalamus, reaching pituitary by hypophyseal portal system Marc Imhotep Cray, MD

126


Regulation of Hypothalamic & Pituitary Hormones cont. Interaction of releasing hormones with their receptors results in activation of genes that promote synthesis of protein precursors protein precursors then undergo posttranslational modification to produce hormones which are released into circulation Each hypothalamic regulatory hormone controls release of a specific hormone from anterior pituitary  Endocrine-organ systems function via negative feedback o e.g., hypothalamic CRH stimulates  pituitary ACTH secretion stimulates adrenal cortisol secretion which in turn inhibits CRH and ACTH secretion Marc Imhotep Cray, MD

127


Relationships Among Hypothalamic, Pituitary, and Target Gland Hormones HYPOTHALAMIC

PITUITARY

TARGET ORGAN

TARGET ORGAN HORMONES

GHRH (+), SRIH (–)

GH (+)

Liver

Somatomedins

CRH (+)

ACTH (+)

Adrenal cortex

Glucocorticoids Mineralocorticoids Androgens

TRH (+)

TSH (+)

Thyroid

T4, T3

GnRH or LHRH (+)

FSH (+), LH (+)

Gonads

Estrogen Progesterone Testosterone

Dopamine (–), PRH=TRH (+)

Prolactin (+)

Breast

Redrawn after: Pazdernik TL, Kerecsen L. Rapid Review Pharmacology, 3rd Ed. Mosby, 2010

+, stimulant; –, inhibitor; ACTH, adrenocorticotropic hormone; CRH, corticotropin-releasing hormone; FSH, folliclestimulating hormone; GH, growth hormone; GHRH, growth hormone–releasing hormone; GnRH, gonadotropinreleasing hormone; LHRH, luteinizing hormone-releasing hormone; LH, luteinizing hormone; PRH, prolactin-releasing hormone; SRIH, somatotropin-releasing inhibiting hormone; TRH, thyrotropin releasing hormone; TSH, thyroidstimulating Marc Imhotephormone. Cray, MD

128


Hormones secreted by anterior pituitary include: FSH, LH, ACTH, TSH, Prolactin, and GH FLAT P(i)G is a useful mnemonic to remember these hormones

Marc Imhotep Cray, MD

Modified from: Whalen K. Lippincott Illustrated Reviews: Pharmacology 6th Ed. Wolters Kluwer, 2015

129


Anterior Pituitary & Hypothalamic Hormone Receptors  Anterior pituitary hormones can be classified according to hormone structure and types of receptors that they activate  Anterior pituitary gland hormones are proteins and glycoproteins Anterior pituitary gland hormones fall into three groups: 1. Somatotropic hormones, consisting of growth hormone (GH) and prolactin activate receptors of JAK/STAT superfamily 2. Glycoprotein hormones, consisting of luteinizing hormone (LH), follicle-stimulating hormone (FSH), and thyroidstimulating hormone (TSH) (also shared by hCG) activate G protein-coupled receptors 3. Adrenocorticotropin (ACTH) a separate class, as it is processed by proteolysis from a larger precursor protein (proetal. Basic and Clinical opiomelanocortin activate G protein-coupled receptors Katzung, Pharmacology, 12th ed. McGraw-Hill, 2012


Anterior pituitary (adenohypophysis)  Secretes FSH, LH, ACTH, TSH, prolactin, GH  FLAT PiG: FSH, LH, ACTH, TSH, PRL, GH  Melanotropin (MSH) secreted from intermediate lobe of pituitary  Adenohypophysis is derived from oral ectoderm (Rathke pouch)  α subunit—hormone subunit common to TSH, LH, FSH, and hCG  β subunit—determines hormone specificity  ACTH and MSH are derivatives of proopiomelanocortin (POMC) Histology: Each anterior pituitary hormone is produced by a separate group of cells, which according to their staining characteristic are either  Basophils: FSH, LH, ACTH, TSH (B-FLAT)  Acidophils: GH, PRL Marc Imhotep Cray, MD

131


Hypopituitarism Hypopituitarism may be partial or complete and may result from hypothalamic disease (leading to deficiency of hypothalamic releasing hormones) or intrinsic pituitary disease (causing pituitary hormone deficiency)  Patients may present with, for example, adrenal insufficiency or hypothyroidism

Clinical signs depend on degree and rapidity of onset of deficiency  For example, basal cortisol secretion is normal in partial ACTH deficiency, but during an illness, adrenal insufficiency may occur  In complete ACTH deficiency, cortisol secretion is always subnormal Marc Imhotep Cray, MD

132


Hypopituitarism (2)  Diagnosis of complete deficiency is relatively easy:  most patients have symptoms, and  serum levels of target-organ hormone (eg, cortisol, thyroxine, and testosterone in men) and pituitary hormone (eg, ACTH, thyrotropin, and luteinizing hormone, respectively) are low  Causes of hypopituitarism include  pituitary tumor (most common)  hypothalamic tumor or cyst  infiltrative and vascular disorders  pituitary or cranial radiotherapy  pituitary necrosis caused by ischemia

 Treatment: hormone replacement therapy (corticosteroids, thyroxine, sex steroids, human growth hormone) Marc Imhotep Cray, MD

133


Hypopituitarism (3)

Raff RB, Rawls SM, Beyzarov EP. Netter's Illustrated Pharmacology, Updated Edition. Saunders, 2014

134


Hypopituitarism (4)

Raff RB, Rawls SM, Beyzarov EP. Netter's Illustrated Pharmacology, Updated Edition. Saunders, 2014

135


Sheehan syndrome:  In this condition, ischemic necrosis of pituitary causes hypopituitarism caused by severe hypotension from postpartum hemorrhage  Pituitary is particularly vulnerable during pregnancy because of reduced blood flow associated with its enlargement at this time  Result of damage to gland is permanent underproduction of essential pituitary hormones (hypopituitarism)  Agalactia, amenorrhea, hypothyroidism and adrenocortical insufficiency are important consequences  Treatment of Sheehan syndrome is hormone replacement therapy

Rubin R , Strayer DS Eds. Rubin’s Pathology: Clinicopathologic Foundations of Medicine, 6th Ed. Baltimore: Lippincott Williams & Wilkins, 2012.


Sheehan syndrome (2)  The specific association with postpartum shock or hemorrhage was described in 1937 by British pathologist Harold Leeming Sheehan (1900–1988), whereas Simmond's disease occurs in either sex due to causes unrelated to pregnancy  Characterized clinically by asthenia, loss of weight and body hair, arterial hypotension, and manifestations of thyroid, adrenal, and gonadal hypofunction (See Endocrine Tutorial 1) Marc Imhotep Cray, MD

Young WF. The Netter Collection of Medical Illustrations Vol 2The Endocrine System 2nd Edn. Philadelphia: Saunders, 2011.

137


Question A 53-year-old woman is diagnosed with hypopituitarism. Which of the following hormones is most likely to be affected first? A. Follicle stimulating hormone (FSH) and luteinizing hormone (LH) B. Thyroid stimulating hormone (TSH) C. Adrenocorticotropic hormone (ACTH) D. Prolactin E. Growth hormone

Marc Imhotep Cray, MD

138


Question A 25-year-old woman is diagnosed with hypopituitarism. Which of the following hormones is essential to replace first when managing her condition? A. Thyroid hormone B. Estrogen C. Growth hormone D. Luteinizing hormone E. Follicle stimulating hormone

Marc Imhotep Cray, MD

139


Growth hormone-releasing hormone (GHRH) GHRH is an active peptide of 44 amino acids produced by hypothalamus within arcuate nucleus  GHRH binds to specific membrane GHRH receptors on pituitary somatotrophs  GHRH rapidly elevates serum growth hormone (somatotropin) levels with high specificity

GHRH release is also modulated by “GH secretagogues” via a unique GH secretagogue receptor which is actually ghrelin receptor  Ghrelin is a peptide secreted by stomach in response to fasting  Ghrelin also stimulates appetite Marc Imhotep Cray, MD

140


Growth Hormone (somatotropin) normal physiologic functions and regulation  GH secretion occurs primarily at night and in response to various stressors such as starvation and hypoglycemia  When released during a good night of sleep, its anabolic actions on muscle and bone are of primary importance  When released in response to physiologic stressors such as starvation and hypoglycemia, its metabolic actions to conserve carbohydrate fuels (for use by central nervous system and other glucose-dependent tissues) and maintain protein stores (to preserve muscle strength needed for mobility) take center stage  GH secretion is inhibited by elevated somatostatin, somatomedins increase glucose levels, emotional stress, illness, malnutrition, obesity, glucocorticoids, and TH and pregnancy Marc Imhotep Cray, MD

N.B. Triiodothyronine (T3) is required for normal function of GH.

141


Actions of growth hormone (STH)  In liver, growth hormone generates production of somatomedins [insulinlike growth factors (IGF)] serve as intermediaries of several physiologic actions  IGF receptor has tyrosine kinase activity, similar to insulin receptor

(1) Direct actions of growth hormone (a) ↓ glucose uptake into cells (diabetogenic) (b) ↑ lipolysis (c) ↑ protein synthesis in muscle and ↑ lean body mass (d) ↑ production of IGF

Note: In both children and adults, GH has anabolic effects in muscle and catabolic effects in lipid cells. Shift balance of body mass to an increase in muscle mass and a reduction in adiposity

(2) Actions of growth hormone via IGF (a) ↑ protein synthesis in chondrocytes and ↑ linear growth (pubertal growth spurt) (b) ↑ protein synthesis in muscle and ↑ lean body mass (c) ↑ protein synthesis in most organs and ↑ organ size Marc Imhotep Cray, MD

142


Physiologic actions of growth hormone

Marc Imhotep Cray, MD

Brown TA. Rapid Review Physiology 2nd Ed. Philadelphia: Mosby, 2012

143


Somatomedins  Somatomedins are produced, predominantly by liver, when growth hormones act on target tissue  Somatomedins inhibit release of growth hormones by acting directly on anterior pituitary and by stimulating secretion of somatostatin from hypothalamus  Somatomedins are a group of hormones that promote cell growth and division in response to stimulation by growth hormone (GH) also known as somatotropin (STH)  Somatomedins have similar biological effects to somatotropin Marc Imhotep Cray, MD

144


Somatomedins cont.  In addition to actions that stimulate growth somatomedins also stimulate production of somatostatin which suppresses growth hormone release  Thus, levels & GH effects of somatomedins are controlled via negative feedback through intermediates of somatostatin and growth hormone  Somatomedins are produced in many other tissues (besides liver) and have autocrine and paracrine actions in addition to their endocrine action  Three forms of somatomedin include:  somatomedin A, another name for insulin-like growth factor 2 (IGF-2)  somatomedin B, derived from vitronectin  somatomedin C, another name for insulin-like growth factor 1 (IGF-1)

Marc Imhotep Cray, MD

145


Mecasermin  Mecasermin is a recombinant form of human IGF-1  Produced by Escherichia coli bacteria that have been stably transfected with human gene for IGF-1  Uses mecasermin is indicated for   

treatment of growth failure in children with severe primary IGF-1 deficiency those with a growth hormone receptor mutation, and those who have developed neutralizing antibodies to growth hormone

 Adverse Effect main adverse effect is hypoglycemia controlled by eating a meal or snack before or soon after time of injection Marc Imhotep Cray, MD

146


Somatostatin (SST)

Synonyms of somatostatin: • growth hormone–inhibiting hormone (GHIH) • growth hormone release–inhibiting hormone (GHRIH) • somatotropin release–inhibiting factor (SRIF) • somatotropin release–inhibiting hormone (SRIH)

 Somatostatin is a peptide hormone that regulates endocrine system and affects neurotransmission and cell proliferation via interaction with G protein-coupled somatostatin receptors and inhibition of release of numerous secondary hormones  Secreted by hypothalamus, GIT and δ-cells of pancreas  SST receptors= five subtypes, SR1 through SR5  Octreotide (SST analog) acts primarily on SR2 and SR5

 Classified as an inhibitory hormone whose actions are spread to different parts of body In anterior pituitary effects of somatostatin are:  Inhibit release of GH (thus opposing effects of GHRH) o decreases sensitivity of anterior pituitary to GHRH

 Inhibit release of TSH  Inhibit adenylyl cyclase in parietal cells  Inhibits release of prolactin (PRL)  Also inhibits secretion of insulin, glucagon, gastrin, and HCl

147


Somatostatin (2) Indicated for management of  acromegaly  islet cell tumors  bleeding due to esophageal varices and  secretory diarrhea disadvantages short duration of action and multiple effects in many secretory systems  A series of longer-acting SST analogs have been developed  Octreotide is a somatostatin analog having high potency and long duration of action o Preferred over somatostatin for all indications Marc Imhotep Cray, MD

148


Somatostatin (3)

Marc Imhotep Cray, MD

Bardal SK et.al. Applied Pharmacology. St. Louis: Saunders, 2011.

149


Hypothalamic-pituitary GH signaling pathway  Growth Hormone-Releasing Hormone (GHRH) binding to its receptors on somatotrophs increases intracellular cAMP and Ca2+ levels, whereas  Somatostatin (Somatotropin ReleaseInhibiting Hormone, SRIH) binding to its receptors on somatotrophs decreases intracellular cAMP and Ca+2  These signaling pathways provide a biochemical explanation for opposing activities GHRH and somatostatin on somatotroph release of GH Costanzo LS. Physiology (Basic Review Series), 5th ed. New York: 150 Elsevier, 2009.


Growth Hormone Deficiency and Treatment Growth hormone promotes linear growth by regulating endocrine and paracrine production of IGF-1 (insulin-like growth factor 1)  Besides disruption in growth, GH deficiency also causes    

increased subcutaneous visceral fat reduced muscle mass reduced bone density and Reduced exercise performance

 Children have short stature and low growth velocity for age and pubertal stage  Adults, who usually have had pituitary tumors or head trauma, show      

low energy reduced strength weight gain anxiety reduced libido and impaired sleep

Marc Imhotep Cray, MD

151


GH Deficiency and Treatment (2)  A GH deficiency before puberty will result in pituitary dwarfism 

Somatrem (Protropin) and somatropin (Humatrope) are human growth hormone produced by recombinant DNA technology (rhGH) Replacement therapy will increase growth o However, replacement therapy cannot induce linear growth after epiphyseal closure has occurred in the long bones

 Androgens and estrogens also increase growth however, they are less effective than GH and can induce epiphyseal closure which limits further growth

Marc Imhotep Cray, MD


GH Deficiency and Treatment (3)  GH therapy goals differ in children and adults  In adults, they are to improve conditioning and strength, restore normal body composition, and improve quality of life  In children, therapy promotes linear growth and restores body composition  rhGH (synthetic growth hormone) is effective for children with GH deficiency as long as epiphyses are not closed Other Therapeutic Uses  GH stimulates growth in patients with Turner syndrome  long-term replacement of GH deficiency in adults  treatment of cachexia and AIDS wasting  patients with severe burns  Prader-Willi syndrome in children  short bowel syndrome Marc Imhotep Cray, MD

153


Molecular and Cellular Bases of Somatotropic Hormone Action  All of effects of GH (and prolactin) result from their interactions with specific membrane receptors on target tissues 

i.e., receptors associated with cytoplasmic Tyrosine Kinases

 GH (and prolactin) receptors are widely distributed cell surface receptors that belong to cytokine receptor superfamily  share structural similarity with receptors for leptin, erythropoietin, granulocyte-macrophage colony-stimulating factor (GM-CSF), and several interleukins

 Like other cytokine receptor family, GH receptors contain   

an extracellular hormone-binding domain a single membrane-spanning region, and an intracellular domain that mediates signal transduction

Marc Imhotep Cray, MD

154


Molecular and Cellular Bases of Somatotropic Hormone Action (2)  These receptors (cytokine receptor superfamily) have no intrinsic enzymatic activity, rather intracellular domain binds a separate, intracellular tyrosine kinase termed a Janus kinase (JAK)  Upon dimerization induced by ligand binding JAKs phosphorylate other proteins termed signal transducers and activators of transcription (STATs) which translocate to nucleus and regulate transcription  Entire pathway is termed the JAK-STAT pathway  There are four JAKs and six STATs in mammals, depending on cell typeCray,and Marc Imhotep MD signal, combine differently to activate gene transcription

155


Signaling from cytokine receptor family JAK-STAT Receptor Pathway  Growth Hormone (and prolactin) bind to transmembrane receptors that belong to cytokine receptor family  These are constitutively dimerized receptors bound by janus kinases (JAKs)  Hormone binding interacts with both extracellular domains and induces JAK-JAK crossphosphorylation followed by recruitment and binding of STAT proteins  Phosphorylation of STATs activates them and induces their translocation to nucleus, where they act as transcription factors White BA & Porterfield SP. Endocrine and Reproductive Physiology, 4th ed. (Mosby physiology monograph series). Mosby, 2013.


Growth Hormone -JAK-STAT Receptor Pathway  Binding of GH to a homodimer of growth hormone receptor (GHR) induces autophosphorylation of JAK2  JAK2 then phosphorylates cytoplasmic proteins that activate downstream signaling pathways, including o STAT5 and mediators upstream of MAPK, which ultimately modulate gene expression Brunton L, Chabner B, Knollman B, eds. Goodman & Gilman’s The Pharmacological Basis of Therapeutics, 12th ed. McGraw-Hill, 1110; Pg. 1242, Figure 38–5.

See: The Growth Hormone Receptor_A Tutorial

157


Growth Hormone (rhGH) Adverse effects Adverse effects  In about 2% of patients, anti-GH antibodies develop  edema  muscle and joint pain  benign intracranial hypertension  hair loss  hypothyroidism  hypoglycemia or hyperglycemia, and  risk of cancer  Administration of GH is contraindicated in  

obese patients patients with closed epiphyses who do not have GH deficiency, patients with neoplastic disease Marc Imhotep Cray, MD

158


Question A 38-year-old man presents complaining of gradually enlarging hands and feet over the past several years. In comparison with a photo from 15 years ago, his facial features have become obviously coarsened. Laboratory evaluation shows mildly elevated plasma glucose, and MRI of the brain reveals an enlarged mass in the sella turcica. Given the suspected diagnosis, specialized testing is performed in which GH levels are measured following administration of an oral glucose load; no measurable decrease is seen. What is the diagnosis? Note: One good way to diagnose this disorder is to look at an old picture of the patient and compare it with the patient’s current appearance. Because the physical changes take place over decades, family members and friends often do not recognize them. Marc Imhotep Cray, MD

159


A. A 26-year-old attractive woman prior to acromegaly changes. B. Facial changes 20 years later in the same woman. Note the coarse facial features with large nose, lips, and chin. Protrusion of lower jaw is visible.

Marc Imhotep Cray, MD

Usatine RP etal. (Eds.) The Color Atlas of Family Medicine. McGraw-Hill, 2013

160


Features of acromegaly /gigantism  A 22-year-old man with gigantism due to excess growth hormone is shown to left of his identical twin  increased height and prognathism (A) and enlarged hand (B) and foot (C) of affected twin are apparent  Their clinical features began to diverge at age of approximately 13 years


GH Excess (Acromegaly) and Treatment Acromegaly is a disfiguring hormonal disorder caused by excessive GH secretion from a pituitary tumor  a rare condition, most common cause is a benign GH (hyper)secreting pituitary adenoma

Signs of acromegaly include  coarse facial features and  enlarged hands, feet, tongue, and internal organs (which lead to heart disease, hypertension, diabetes, arthralgias)  Treatments includes  surgical removal of tumor and (or)  radiation, or  subcutaneous use of octreotide a GH inhibitor, analogs are available in a long-acting depot form or  pegvisomant (prototype growth hormone receptor antagonist) Marc Imhotep Cray, MD

162


GH Excess (Acromegaly) and Treatment (2) Octreotide is a synthetic analog of somatostatin having a longer duration of action (t½ 1.5 h) Note: Octreotide is 45 times more potent than SST in inhibiting GH release, but only twice as potent in reducing insulin secretion.

MOA

 inhibition of GH and IGF-1 levels  suppression of response of LH to GnRH  also inhibits secretion of thyrotropin, serotonin  By normalizing levels of GH and IGF-1 (both markers for acromegaly) octreotide controls clinical signs and symptoms

Uses  Uses include acromegaly, carcinoid (serotonin-secreting) tumors and other rare tumors of GI tract (VIPomas) and bleeding esophageal varices  See Vasoactive Intestinal Peptide tumor and Multiple Endocrine Neoplasia (Tutorial 2) Marc Imhotep Cray, MD

163


Octreotide

 Common adverse effects of octreotide are  gastrointestinal more serious effects include  cardiac arrhythmias  hypoglycemia or hyperglycemia  suppression of thyrotropin  pancreatitis and  biliary tract abnormalities

 It is admin. SQ two or three times daily  

a depot formulation is available for deep intramuscular injection Lanreotide is much longer acting than octreotide admin. IM twice a month

Marc Imhotep Cray, MD

164


Octreotide cont.  Octreotide also inhibits TSH secretion and is treatment of choice for TSH-secreting adenoma in patients who are not candidates of surgery  Lanreotide is another somatostatin analog that can be given i.m. in slow release formulation (longer acting)  Vapreotide and seglitide are other somatostatin analogs  Pasireotide is new somatostatin analog approved for treatment of Cushing‘s disease Marc Imhotep Cray, MD

165


Pegvisomant (GH receptor antagonist)  GH is a protein substitution of one amino acid converts endogenous GH into an antagonist  Commercial GH antagonists have multiple substitutions to enhance binding affinity

MOA  Pegvisomant block action of GH at GH receptor in liver prevents GH from activating GH signaling pathways Pharmacokinetics  Addition of polyethylene glycol 500 (PEG-500) to GH  GH antagonists, increases its half-life from 30 minutes to about 2 days  As a protein, it cannot be taken orally; it is injected subcutaneously Uses  Acromegaly  For patients with persistently elevated IGF-1 despite other therapy  Possibly as monotherapy (more research required) Marc Imhotep Cray, MD

166


Pegvisomant cont. Contraindication  Severe liver disease (because of risk of liver damage) Adverse Effects  Elevated liver enzymes (transaminases) are seen in 25% of patients, indicating liver damage. Liver enzymes must be routinely monitored Important Notes  Surgery, dopamine agonists, and somatostatin are standard first-line treatments for acromegaly  DA agonists and somatostatin are inhibitors of GH secretion but are not direct antagonists of GH therefore their MOA is slightly different, and they act at a different location in biochemical pathway Marc Imhotep Cray, MD

167


Pegvisomant cont.

Marc Imhotep Cray, MD

Bardal SK et.al. Applied Pharmacology. St. Louis: Saunders, 2011.

168


Posterior pituitary (neurohypophysis)

Posterior pituitary bright spot. Sagittal T1-MRI image showing hyperintensity (arrow) in the posterior aspect of the sella turcica. Young WF. The Netter Collection of Medical Illustrations Vol 2- The Endocrine System, 2nd Edn. Saunders, 2011 Marc Imhotep Cray, MD

169


Hormones of the Posterior Pituitary  Oxytocin  Used in obstetrics to stimulate uterine contraction and induce labor  Oxytocin also causes milk ejection  Vasopressin (antidiuretic hormone, ADH) is structurally related to oxytocin  Has both antidiuretic and vasopressor effects  In kidney, it binds to V2 receptor to increase water permeability and reabsorption in collecting tubules Thus, major use of vasopressin is to treat diabetes insipidus  Other effects of vasopressin are mediated by V1 receptor, which is found in liver, vascular smooth muscle (where it causes constriction), and other tissues  It also finds use in management of cardiac arrest and in controlling bleeding due to esophageal varices Marc Imhotep Cray, MD

170


Oxytocin Effects originates primarily in paraventricular nuclei of hypothalamus causes ejection of milk from breast when stimulated by suckling

Regulation of oxytocin secretion (1) Suckling  major stimulus for oxytocin secretion o afferent fibers carry impulses from nipple to spinal cord relays in hypothalamus trigger release of oxytocin from posterior pituitary o sight or sound of infant may stimulate hypothalamic neurons to secrete oxytocin, even in absence of suckling

(2) Dilation of cervix and orgasm  increases secretion of oxytocin Marc Imhotep Cray, MD

171


Oxytocin

Marc Imhotep Cray, MD

Mulroney SE & Myers AK. Netter's Essential Physiology 2nd Ed. Philadelphia: Elsevier, 2016.

172


Actions of oxytocin Contraction of myoepithelial cells in breast  Milk is forced from mammary alveoli into ducts and delivered to infant Contraction of uterus  During pregnancy, oxytocin receptors in uterus are upregulated as parturition approaches, although role of oxytocin in normal labor is uncertain Uses  Oxytocin can be used to induce labor and reduce postpartum bleeding Marc Imhotep Cray, MD

173


ADH (or) Vasopressin  originates primarily in supraoptic nuclei of hypothalamus  regulates serum osmolarity by increasing H2O permeability of late distal tubules and collecting ducts Regulation of ADH secretion: Factors that Increase ADH Secretion      

↑Serum osmolarity Volume contraction Pain Nausea (powerful stimulant) Hypoglycemia Nicotine, opiates, antineoplastic drugs

Marc Imhotep Cray, MD

Factors that Decrease ADH Secretion    

↓ Serum osmolarity Ethanol α-Agonists ANP

174


Actions of ADH (↑ H2O permeability (aquaporin 2, AQP2) of principal cells of late distal tubule and collecting duct (via a V2 receptor and an adenylate cyclase–cAMP mechanism) Constriction of vascular smooth muscle (via a V1 receptor and an IP3/Ca2+ mechanism)

Marc Imhotep Cray, MD

175


Antidiuretic hormone (ADH)

Marc Imhotep Cray, MD

Mulroney SE & Myers AK. Netter's Essential Physiology 2nd Ed. Philadelphia: Elsevier, 2016.

176


Desmopressin (DDAVP)  Desmopressin an analog of vasopressin, has minimal activity at V1 receptor, making it largely free of pressor effects

Clinical Uses:  longer acting than vasopressin and is preferred for treatment of diabetes insipidus and nocturnal enuresis  desmopressin may be administered intranasally or orally  Local irritation may occur with nasal spray  Nasal spray should not be used for enuresis due to reports of seizures in children using this formulation Marc Imhotep Cray, MD

177


Summary table of posterior pituitary hormones & their Effects Hormone

Synthesized by

Antidiuretic hormone (ADH)

Supraoptic vasopressinergic neurons

Raised osmolarity; Lower low blood volume osmolarity

Kidney

Increases permeability of collecting duct to reabsorb water

Oxytocin

Paraventricular oxytocinergic neurons

Stretch receptors in the nipple and cervix, estrogen

Uterus and mammary glands

Smooth muscle contraction leading to birth or milk ejection

Marc Imhotep Cray, MD

Stimulated by

Inhibited by

Stress

Target

Organ Effect

178


Drugs that mimic or block effects of hypothalamic and pituitary hormones

Marc Imhotep Cray, MD

Katzung & Trevors Pharmacology Examination and Board Review, 11th Ed. McGraw-Hill, 2015.


Case 39 Answers: Drugs Active on Hypothalamus and Pituitary Gland Summary: A 67-year-old man with metastatic prostate cancer is to receive depot leuprolide acetate. • Leuprolide acetate is an analog of which hypothalamic hormone: Gonadotropin releasing hormone (GnRH). • Mechanism of action of leuprolide acetate: Chronic administration of GnRH analog results in the reduction of the number of GnRH receptors in the pituitary (downregulation), with resultant decreases in pituitary gonadotropin production. • Pituitary hormones affected: Luteinizing hormone (LH) and follicle-stimulating hormone (FSH) production is reduced. Marc Imhotep Cray, MD

180


Case 39 Answers: Drugs Active on Hypothalamus and Pituitary Gland cont. Gonadotropin Suppression  Stable potent derivatives of GnRH include leuprolide and goserelin  Because these agonists are long acting, they suppress gonadotropin production after an initial stimulation  In some uses, initial stimulation of gonadotropin is undesirable (prostatic cancer) o a newer GnRH antagonist, ganirelix inhibits gonadotropin production without stimulation and may ultimately replace longacting agonists


Case 39 Answers: Drugs Active on Hypothalamus and Pituitary cont. CLINICAL CORRELATION

The hypothalamic-pituitary-gonadal axis is a classic example of a hormonal stimulation- negative feedback system. The hypothalamus produces GnRH, which binds to specific receptors on pituitary gonadotropic cells. These cells then produce LH and FSH, which act on the gonads. LH and FSH regulate the female menstrual cycle by their effects on the ovarian follicles and the ovarian production of estrogen and progesterone. In males, LH and FSH regulate spermatogenesis and the production of testosterone in the testes. Estrogen, progesterone, and testosterone then function as feedback signals for the hypothalamic production of GnRH. Leuprolide acetate is a synthetic 9-amino acid analog of GnRH. Marc Imhotep Cray, MD

182


Case 39 Answers: Drugs Active on Hypothalamus and Pituitary cont. When initially administered, leuprolide acetate results in increases in LH, FSH, and gonadal steroid production because of its action as a GnRH agonist. However, with chronic administration, there is a reduction in the number of GnRH receptors in the pituitary gonadotropic cells. This causes a reduction in FSH or LH production and a resultant reduction in gonadal hormone production. In women this effect may be beneficial in conditions such as endometriosis, where estrogen stimulates the growth and activity of the ectopic endometrial tissue, which causes symptoms.

Marc Imhotep Cray, MD

183


Case 39 Answers: Drugs Active on Hypothalamus and Pituitary cont. The effect in men is to lower the production of testosterone to near castrate levels. Because prostate cancer is often testosterone dependent, leuprolide acetate can be used as a treatment for prostate cancer in those who are not surgical candidates, do not desire surgery, or have metastatic disease. Leuprolide acetate must be administered parenterally, and it has a depot form which is active for up to 3 months. It commonly causes “menopausal� side effects, such as hot flashes, as a result of the reduction in gonadal hormone production. Other antiandrogenic drugs such as abiraterone, which blocks conversion of pregnenolone to androgens by inhibiting CYP17, can be used in combination with leuprolide or as sole therapeutic agents. Marc Imhotep Cray, MD

184


Lect. 3 of 6

Thyroid Disorders Thyroid secretes 2 types of hormones: iodine-containing amino acids (thyroxine and triiodothyronine) and a peptide (calcitonin). Thyroxine and triiodothyronine have broad effects on growth, development, and metabolism. Calcitonin is important in calcium metabolism and is discussed in lecture 4. This presentation focuses on the drugs used in the treatment of hypothyroidism and hyperthyroidism.

Marc Imhotep Cray, MD

185


Learning Objectives Thyroid and Anti-thyroid Drugs 1. The concept that thyroid hormone plays a major role in regulating development as well as metabolism and calorigenesis. 2. The steps in synthesis of tetraiodothyronine (T4) and triiodothyronine (T3) 3. The endocrine regulation of T3 and T4 production and feedback loops 4. The physiological roles of T3 and T4 and, therefore, the changes associated with hypothyroidism and hyperthyroidism. 5. The mechanism of action, adverse effects and contraindications of drugs used to treat hyperthyroidism. Marc Imhotep Cray, MD

186


Some hormones and drugs affecting the Thyroid Gland DRUGS AFFECTING THE THYROID Iodine and potassium iodide Liothyronine Levothyroxine Methimazole Propylthiouracil (PTU) Radioactive iodine (131I) Beta blockers (propranolol)

Marc Imhotep Cray, MD

187


Thyroid Disorders

Marc Imhotep Cray, MD

Baron SJ and Lee CI. Lange Pathology Flash Cards. New York: McGraw-Hill, 2009

188


Case 42 Thyroid Hormones A 44-year-old woman presents to the office because of fatigue. She has felt sluggish for months and thinks she may be anemic. She has started taking iron pills but isn’t feeling any better. She has been sleeping well and doesn’t feel depressed. She has noticed some thinning of her hair and feels as if her skin is dry. She takes a multivitamin and iron supplement, otherwise no medications. She has smoked a pack of cigarettes a day for approximately 20 years, occasionally drinks alcohol, and doesn’t exercise. Her mother takes some kind of thyroid pill and has diabetes. On examination, her blood pressure and pulse are normal. Her hair is thinned but there are no focal patches of alopecia or scarring of the scalp. Her skin is diffusely dry. Her thyroid gland feels diffusely enlarged, is nontender, and has no nodules. The remainder of her examination is unremarkable. Lab tests show a normal complete blood count (CBC), glucose, and electrolytes. Her thyroid-stimulating hormone (TSH) level is elevated, and T4 level is reduced. You diagnose her with hypothyroidism and start her on oral levothyroxine sodium. _ What is levothyroxine sodium? _ How is triiodothyronine (T3) produced in the body? _ What is the mechanism of action of thyroid hormones? Marc Imhotep Cray, MD

189


Thyroid Gland Structure

Widmaier EP, Raff H & Strang KT. Vander’s Human Physiology : The Mechanisms of Body Function, 11th ed. New York, NY: McGraw-Hill, 2008.

Mulroney SE & Myers AK. Netter's Essential Physiology 2nd Ed. Philadelphia: Elsevier, 2016.


Normal thyroid in situ, gross

Klatt EC. Robbins and Cotran Atlas of Pathology, 3rd Ed. Philadelphia: Saunders, 2015. 191


Normal thyroid, microscopic

Klatt EC. Robbins and Cotran Atlas of Pathology, 3rd Ed. Philadelphia: Saunders, 2015.

192


Thyroid Hormones (TH)  Triiodothyronine (T3) and thyroxine (T4) are produced by thyroid gland in response to hormones released by pituitary and hypothalamus  Hypothalamic thyrotropin-releasing hormone (TRH) stimulates release of thyrotropin (i.e., thyroid-stimulating hormone [TSH]) from pituitary in response to low circulating levels of TH  TSH in turn promotes hormone synthesis and release by increasing thyroid activity

 When sufficient synthesis has occurred, high circulating TH levels block further production by inhibiting TSH release (negative feedback)  As serum concentrations of thyroid hormone decrease, hypothalamic-pituitary centers again become responsive by releasing TRH and TSH

Alldredge BK et al.(Eds.). Koda-Kimble and Young’s Applied Therapeutics : The Clinical Use of Drugs,10th ed. Lippincott Williams &Wilkins, 2013. 193


Thyroid Hormones Thyroid gland is responsible for regulating normal growth and development by maintaining a level of metabolism in body tissues that is optimal for normal function Thyroid synthesizes, stores, and releases 2 major*, metabolically active hormones: triiodothyronine (T3) and thyroxine (T4)  T3, active form of thyroid hormone, is 4 times more potent than T4 but its serum concentration is lower N.B. *Thyroid gland secretes three hormones essential for regulation of metabolism: Follicular cells T3, T4 Parafollicular cells Calcitonin Marc Imhotep Cray, MD

194


Thyroid Hormones (2) Approximately 80% of gland’s total daily production of T3 results from conversion of T4 to T3 through deiodination of T4

 T3 and T4 exist in either free (active) or protein-bound (inactive) forms  More than 99% of circulating T4 is bound to plasma proteins, so only a small fraction exists in free form o As a result, T4 is metabolized very slowly and has a long half-life (7 days) o T3 is less bound to plasma proteins and thus undergoes faster metabolism and has a shorter half-life (1.5 days) Marc Imhotep Cray, MD

195


Thyroid Hormones (3) Pharmacokinetics  Both T4 and T3 are absorbed after oral administration  Food, calcium preparations, and aluminum-containing antacids can decrease absorption of T4 but not of T3  T4 is converted to T3 by one of two distinct deiodinases, depending on the tissue  T4 is a prohormone to the active T3  Hormones are metabolized through microsomal P450 system  Drugs that induce P450 enzymes, such as phenytoin, rifampin, and phenobarbital (enzyme inducers) accelerate metabolism of thyroid hormones o enzyme induction can increase metabolism of thyroid hormones, making levels subtherapeutic while treating hypothyroidism Marc Imhotep Cray, MD

196


Physiological effects of thyroid hormones  Principal effects of TH are:  stimulation of metabolism – raised basal metabolic rate;  promotion of normal growth and maturation, particularly  of t central nervous system and skeleton  sensitization to the effects of catecholamines

Marc Imhotep Cray, MD

          

Other effects: ↑Body temperature ↑Cardiac rate and contractility ↑Peripheral vasodilatation ↑Red cell mass ↑Circulatory volume ↑Respiratory drive ↑Peripheral nerves (reflexes) ↑Hepatic metabolic enzymes ↑Bone turnover Skin and soft tissue effects 197


THs: Synthesis, Release, and Regulation  TSH binds to receptors on thyroid follicular cells and activates adenylyl cyclase, which stimulates iodine trapping, iodothyronine synthesis and release of thyroid hormones  Thyroid hormones are synthesized and stored as amino acid residues of thyroglobulin  Major steps in synthesis and release of T3/T4 include: thyroid uptake of iodide oxidation of iodide and iodination of tyrosyl groups of thyroglobulin coupling iodotyrosine residues to produce iodothyronines proteolysis of thyroglobulin release of T4 and T3 into blood conversion of T4 to T3 in peripheral tissues and thyroid Marc Imhotep Cray, MD

198


Steps involved in T3 and T4 formation

Marc Imhotep Cray, MD

Widmaier EP, Raff H & Strang KT. Vander’s Human Physiology : The Mechanisms of Body Function, 11th ed. New York, NY: McGraw-Hill, 2008.

199


THs: Synthesis, Release, and Regulation (3) Thyroid hormone synthesis and release are controlled by negative feedback mechanisms 1. at level of thyroid 

Circulating T3/T4 halt TRH and TSH secretion

2. at level of hypothalamic-pituitary axis 

Low circulating hormone levels trigger hypothalamic release of TRH which induces pituitary secretion of TSH o Increasing TSH levels stimulate thyroid iodide uptake and hormone synthesis

3. autoregulation of iodide uptake  Thyroid also regulates its own iodine uptake to protect against excess hormone production if extra iodide is ingested (Wolff–Chaikoff effect) Marc Imhotep Cray, MD

200


Synthesis of thyroid hormones (8 Steps) Each step in synthesis is stimulated by TSH Thyroglobulin is synthesized from tyrosine in thyroid follicular cells, packaged in secretory vesicles, and extruded into follicular lumen (step 1) The iodide (I-) pump (Na+–I- cotransport) is present in thyroid follicular epithelial cells  actively transports I− into thyroid follicular cells for subsequent incorporation into thyroid hormones (step 2) o inhibited by thiocyanate and perchlorate anions Marc Imhotep Cray, MD

201


Synthesis of thyroid hormones cont. Oxidation of I- to I2  catalyzed by a peroxidase enzyme (thyroid peroxidase) in follicular cell membrane (step 3) I2 is reactive form, which will be “organified” by combination with tyrosine on thyroglobulin  Thyroid Peroxidase enzyme is inhibited by propylthiouracil, which is used therapeutically to reduce TH synthesis for Tx of hyperthyroidism

o same peroxidase enzyme catalyzes remaining organification and coupling reactions involved in synthesis of thyroid hormones

Marc Imhotep Cray, MD

202


Synthesis of thyroid hormones cont. Organification of I2  At junction of follicular cells and follicular lumen, tyrosine residues of thyroglobulin react with I2 to form monoiodotyrosine (MIT) and diiodotyrosine (DIT) (step 4)  High levels of I− inhibit organification and, therefore, inhibit synthesis of thyroid hormone =Wolff–Chaikoff effect

Marc Imhotep Cray, MD

203


Synthesis of thyroid hormones cont. Coupling of MIT and DIT  While MIT and DIT are attached to thyroglobulin, two coupling reactions occur (step 5) i. When two molecules of DIT combine, thyroxine (T4) is formed ii. When one molecule of DIT combines with one molecule of MIT, triiodothyronine (T3) is formed  More T4 than T3 is synthesized, although T3 is more active  Iodinated thyroglobulin is stored in follicular lumen until thyroid gland is stimulated by TSH to secrete thyroid hormones Marc Imhotep Cray, MD

204


Synthesis of thyroid hormones cont. Stimulation of thyroid cells by TSH    When thyroid cells are stimulated, iodinated thyroglobulin is taken back into follicular cells by endocytosis (step 6)  Lysosomal enzymes then digest thyroglobulin releasing T4 and T3 into circulation (step 7)  Leftover MIT and DIT are deiodinated by thyroid deiodinase (step 8)  I2 released is reutilized to synthesize more thyroid hormones o Therefore, deficiency of thyroid deiodinase mimics I2 deficiency Marc Imhotep Cray, MD

205


Synthesis of thyroid hormones cont. Binding of T3 and T4  In circulation, most of T3 and T4 is bound to thyroxine-binding globulin (TBG) (also bound to albumin and pre-albumin to a lesser extent) o In hepatic failure, TBG levels decrease, leading to a decrease in total thyroid hormone levels, but normal levels of free hormone o In pregnancy, TBG levels increase, leading to an increase in total thyroid hormone levels, but normal levels of free hormone (i.e., clinically, euthyroid)

Conversion of T4 to T3 and reverse T3 (rT3)   In peripheral tissues, T4 is converted to T3 by 5’-iodinase (or to rT3)  T3 is more biologically active than T4   rT3 is inactive Marc Imhotep Cray, MD

206


8 Steps Synthesis of Thyroid Hormones Illust.

Marc Imhotep Cray, MD

Costanzo LS. Physiology. 5th ed. New York: Elsevier; 2009

207


Thyroid Hormone molecular-cellular mechanisms  Ligand-bound thyroid receptor may dimerize with itself or with retinoic acid receptor (RxR) before translocation to nucleus  Bound thyroid hormone receptors (TR) interact with specific thyroid response elements (TREs) of thyroid hormoneresponsive genes

 Bind with receptors in nuclei of target cells and alter synthesis rates of specific mRNAs increasing production of certain proteins, including Na+/K+ -ATPase

Golan DE et.al. Principles of Pharmacology: The Pathophysiologic Basis of Drug Therapy 3rd Ed. Lippincott Williams & Wilkins, 2012 208


Thyroid regulation, physiology, and pathophysiology

Brown TA. Rapid Review Physiology 2nd Ed. Philadelphia: Mosby, 2012.

209


Goitrous Hypothyroidism Epidemiology (Endemic goiter)  One form of hypothyroidism that exists around world is caused by iodine deficiency  Worldwide, goiter is most common endocrine disorder with rates of 4% to 15% in areas of adequate iodine intake and more than 90% where there is iodine deficiency  Endemic goiter is defined as goiter that affects more than 5% of population  Most goiters are not associated with thyroid dysfunction  FM-to-M ratio of goiter is 3:1, and 6:1 for goitrous hypothyroidism NB: Annual incidence of autoimmune hypothyroidism (U.S.) is 4 in 1000 women and 1 in 1000 men, with a mean age at Dx of 60 years Marc Imhotep Cray, MD

Usatine RP etal. (Eds.) The Color Atlas of Family Medicine. McGraw-Hill, 2013

210


Worldwide iodine nutrition

Marc Imhotep Cray, MD

See WHO and the International Council for the Control of Iodine Deficiency Disorders (http://indorgs.virginia.edu/iccidd/mi/cidds.html)


Goitrous Hypothyroidism Pathophysiology  With inadequate iodine consumption synthesis of TH is compromised leading to a decrease in plasma levels of T3 & T4

Goiter at an advanced stage

 This, in turn, releases negative feedback on hypothalamus and pituitary TRH levels become chronically elevated in portal circulation of anterior pituitary Plasma TSH concentration is also elevated due to increased TRH  Resulting overstimulation of thyroid can produce goiters that can achieve astounding sizes if untreated

Widmaier EP, Raff H & Strang KT. Vander’s Human Physiology : The Mechanisms of Body Function, 11th ed. New York, NY: McGraw-Hill, 2008.


Endemic goiter ď ą Massive goiter in an Ethiopian woman who lives in an endemic area for goiters

ď ą Many adults have large goiters in Ethiopia where there is little iodine in their diets Usatine RP etal. (Eds.) The Color Atlas of Family Medicine. McGraw-Hill, 2013 Marc Imhotep Cray, MD

213


Thyroid, goiter, microscopic

Klatt EC. Robbins and Cotran Atlas of Pathology, 3rd Ed. Philadelphia: Saunders, 2015.

214


Hypothyroidism Hypothyroidism, a syndrome that results from a deficiency of thyroid hormones caused by either primary (thyroid gland) or secondary (hypothalamic or pituitary) dysfunction Most common cause of primary hypothyroidism is Hashimoto thyroiditis  autoimmune disorder in which unsuppressed T lymphocytes produce excessive amounts of antibodies that destroy thyroid cells

Certain drugs, such as amiodarone, lithium, nitroprusside, iodides, and sulfonylureas, can also induce hypothyroidism Hypothyroidism is more prevalent in females and persons older than 60 years Marc Imhotep Cray, MD

215


Hashimoto thyroiditis, microscopic

Klatt EC. Robbins and Cotran Atlas of Pathology, 3rd Ed. Philadelphia: Saunders, 2015.

216


Hashimoto thyroiditis, gross

Klatt EC. Robbins and Cotran Atlas of Pathology, 3rd Ed. Philadelphia: Saunders, 2015.

217


Hypothyroidism cont.  Typically pts. presents with symptoms of “slowing down” including: weight gain, fatigue, sluggishness, cold intolerance, constipation, muscle aches and goiter may be present  Patients with end-stage hypothyroidism = myxedema coma may experience hypothermia, confusion, stupor or coma, carbon dioxide retention, hyponatremia, and ileus  Laboratory findings include increased TSH and low free T4 levels  Pts. with primary hypothyroidism have decreased T3 and T4 levels and elevated TSH  Pts. with pituitary (secondary) hypothyroidism and hypothalamic (tertiary) hypothyroidism have decreased T3, T4, and TSH Marc Imhotep Cray, MD

218


Myxedema Can be described as hypothyroidism of adult Causes  Hashimoto thyroiditis  Idiopathic causes  Iodine deficiency o A problem in geographic areas with poor nutrition o Deficiency in pregnant women can lead to cretinism in child  Paradoxically, high doses of iodine lead to a decrease in thyroid hormone production  Over-irradiation of thyroid using iodine-131 for treatment of hyperthyroidism NB: Myxedema coma is defined as severe hypothyroidism Marc Imhotep Cray, MD

leading to decreased mental status, hypothermia, and other symptoms related to slowing of function in multiple organs. It is a medical emergency with a high mortality rate

219


Marc Imhotep Cray, MD

Raff RB, Rawls SM, Beyzarov EP. Netter's Illustrated Pharmacology, Updated Edn. Saunders, 2014

220


Hypothyroidism: Treatment of Choice Principal treatment goal for hypothyroidism is to achieve a euthyroid state with thyroid replacement therapy Preparation of choice is levothyroxine, a synthetic T4 formulation with advantages including:  stability  uniform potency  relatively low cost  once-daily dosing and  lack of foreign proteins

Marc Imhotep Cray, MD

221


Hypothyroidism: Treatment of Choice cont.  Levothyroxine may have innate metabolic activity, but most of its activity is due to its conversion to T3  Patients should notice improvement in typical symptoms of hypothyroidism after 3 to 4 weeks of treatment  Toxicity is directly related to T4 levels and manifests as nervousness, tachycardia, heat intolerance, and weight loss

 Levothyroxine is available in various brands and generics, which may not be bioequivalent, so only 1 product should be used throughout treatment Marc Imhotep Cray, MD

222


Levothyroxine sodium Thyroxine is given as levothyroxine sodium (synthetic sodium salt of thyroxine / T4) in maintenance therapy Mechanism of action of thyroid hormones:  Levothyroxine (T4) is converted toT3 in vivo  Bind with receptors in nuclei of target cells and alter synthesis rates of specific messenger mRNAs increasing production of certain proteins including Na+/K+ -ATPase

Marc Imhotep Cray, MD

223


Levothyroxine cont. Pharmacokinetics—Half-life of 6 days and a peak onset of 9 days Route of administration—Oral Indications—Hypothyroidism Contraindications—Levothyroxine should not be given to people with thyrotoxicosis, and should be used with caution in those who have cardiovascular disease Adverse effects—Arrhythmias, tachycardia, anginal pain, cramps, headache, restlessness, sweating, weight loss Therapeutic regimen—Starting dose of levothyroxine sodium should be no greater than 100 mcg daily (reduce in elderly or those with cardiovascular disease) and increase by 25–50 mcg every 4 weeks until a dose of 100–200 mcg is reached Marc Imhotep Cray, MD

224


Thyroid Hormone (T4)-drug Interactions  Calcium and iron products chelate oral T4 preparations in GIT, diminishing their effectiveness  Soy flour, high-fiber diets, and some legumes inhibit absorption of T4  Warfarin doses should be decreased because T4 may increase degradation of vitamin K clotting factors (II, VII, IX, X)  Effects of β-agonists, stimulants, and decongestants should be monitored because T4 potentiate sympathetic effects on heart

 T4 may increase dose requirement for insulin and oral hypoglycemic drugs Marc Imhotep Cray, MD

225


Liothyronine and T3/T4 Combination  Liothyronine is a pure T3 preparation that is not recommended for routine thyroid replacement  After oral ingestion, T3 is absorbed more rapidly than T4, which may produce supraphysiologic plasma T3 levels can lead to thyrotoxicosis  Also, free T4 levels remain low during T3 administration and, if misinterpreted, could lead to incorrect use of more hormone T3 levels must be monitored  Other disadvantages are need for multiple doses, higher expense, and greater potential for cardiotoxicity  T3 is therefore not better than T4, which is converted to T3 anyway o However, T3 is recommended for acute severe myxedema Marc Imhotep Cray, MD

226


Liothyronine and T3/T4 Combination (2)  Liotrix (a stable synthetic) and desiccated thyroid contain T4

plus T3  Liotrix uses a physiologic ratio of 4 : 1 but has same problems as T3 and is more expensive 

Desiccated thyroid, derived mostly from pork, is not recommended product potency and composition vary and can result in toxic effects, including allergic reactions to animal protein

Marc Imhotep Cray, MD

227


Hyperthyroidism Hyperthyroidism, or thyrotoxicosis, is due to excessive thyroid hormone production and is characterized by increased metabolism in all body tissues Most common cause of hyperthyroidism is Graves disease an autoimmune disorder in which an abnormal thyroid immunoglobulin binds to TSH receptor and causes uncontrolled thyroid hormone production  In older patients, most common cause of hyperthyroidism is multinodular toxic goiter

Drugs such as amiodarone, iodides, and lithium can also cause hyperthyroidism  Like hypothyroidism, hyperthyroidism more often in FM than in M

Marc Imhotep Cray, MD

228


Graves disease, microscopic

Klatt EC. Robbins and Cotran Atlas of Pathology, 3rd Ed. Philadelphia: Saunders, 2015.

229


Graves disease, microscopic (2)

Klatt EC. Robbins and Cotran Atlas of Pathology, 3rd Ed. Philadelphia: Saunders, 2015.

230


Hyperthyroidism cont.  Symptoms of hyperthyroidism include goiter, exophthalmos, nervousness, heat intolerance, palpitations, weight loss, insomnia, and new or worsening cardiac findings (atrial fibrillation, angina) Untreated hyperthyroidism can progress to thyroid storm, a possibly fatal state with acute onset of high fever, exaggerated thyrotoxicosis symptoms, cardiovascular collapse, and shock  Laboratory findings include high serum levels of free T4, undetectable TSH levels, or both Marc Imhotep Cray, MD

231


Graves disease Exophthalmos

Inc. osmotic muscle swelling, muscle inflammation, and adipocyte countďƒ exophthalmos Le T and Bhushan V. First Aid for the USMLE Step 1 2015 (McGraw-Hill 2015)

Marc Imhotep Cray, MD

Raff RB, Rawls SM, Beyzarov EP. Netter's Illustrated Pharmacology, Updated Edn. Saunders, 2014

232


Hyperthyroidism cont. Of pharmacologic options  Thionamides [propylthiouracil (PTU) and methimazole] are preferred agents for children, pregnant women (PTU) , and young adults with uncomplicated Graves disease  These agents can be used as long-term therapy or as short-term therapy to reduce thyroid hormone levels before RAI or surgery  Treatment of choice is radioactive iodine  Surgery (subtotal or total thyroidectomy) is considered treatment of choice in cases of suspected malignancy, esophageal obstruction, respiratory difficulties, presence of large goiter Marc Imhotep Cray, MD

233


Question A 42-year-old woman with a history of pernicious anemia comes to the physician complaining of increased anxiety, heart palpitations, heat intolerance, unexplained weight loss, and multiple daily bowel movements. She has not had a period in 4 months. On physical examination, the patient is found to have a goiter, a thyroid bruit, and mild exophthalmos. Laboratory studies show elevated triiodothyronine and free thyroxine levels, and an undetectable thyroidstimulating hormone. Which of the following is the most likely etiology of this patient’s disease? (A) Autoimmune stimulation of thyroid-stimulating hormone receptors (B) Idiopathic replacement of thyroid tissue with fibrous tissue (C) Thyroid adenoma (D) Thyroid hormone-producing ovarian teratoma (E) Viral infection leading to destruction of thyroid tissue Marc Imhotep Cray, MD

234


The correct answer is A This patient presents as a classic case of Graves’ disease. In Graves’ disease, thyroid-stimulating IgG antibodies bind to TSH receptors and lead to thyroid hormone production. This causes glandular hyperplasia and enlargement characteristic of the goiter associated with Graves’ disease. Graves’ disease is the most common cause of thyrotoxicosis. Patients with this condition may have other autoimmune diseases, such as pernicious anemia or type 1 diabetes mellitus Symptom, they frequently present with anxiety, irritability, tremor, heat intolerance with sweaty skin, tachycardia and cardiac palpitations, weight loss, increased appetite, fine hair, diarrhea, and amenorrhea or oligomenorrhea. Signs include diffuse goiter, proptosis, periorbital edema, and thickened skin on the lower extremities. Laboratory values reveal increased thyroid hormone levels and decreased TSH levels. Marc Imhotep Cray, MD

235


Thyroid storm

 Uncommon but serious complication that occurs when hyperthyroidism is incompletely treated/untreated and then significantly worsens in setting of acute stress such as infection, trauma, surgery  Presents with agitation, delirium, substantial elevation in BMR and extreme fever, diarrhea, coma, and tachyarrhythmia (cause of death)  Treat with 4 P’s: NB: Treatment of thyroid storm is β-blockers (e.g., Propranolol), same as that for hyperthyroidism, Propylthiouracil, except that drugs are given in higher corticosteroids (e.g., Prednisolone), doses and more frequently Intravenous administration of Potassium iodide (Lugol iodine) medication is most efficacious

Marc Imhotep Cray, MD

236


Thyroid storm cont. Why is propylthiouracil (PTU) used for this condition instead of methimazole? Both PTU and methimazole inhibit TH synthesis, but because thyroid has an abundant store of thyroid hormone it may take weeks for this effect to manifest  However, because at higher doses PTU also inhibits conversion of T4 to T3 in peripheral tissues, and T3 is more active form of thyroid hormone  PTU can have a fairly rapid effect

Marc Imhotep Cray, MD

237


Thioamides (or Thionamides) MOA inhibit formation of thyroid hormones  substrates for thyroid peroxidase block oxidation of iodide in thyroid and prevent subsequent synthesis of T3 and T4  interfering with incorporation of iodine into tyrosyl residues of thyroglobulin  inhibiting coupling of iodotyrosyl residues to form iodothyronines  PTU, but not methimazole, inhibits peripheral deiodination of T4 to T3, which causes a more rapid decline in T3 levels in patients with thyroid storm Marc Imhotep Cray, MD

238


Thionamides MOA cont. Two antithyroid U.S. are PTU and methimazole; both belong to thionamide class  Carbimazole (used in UK) is a prodrug and acts after conversion to methimazole They differ in that PTU also decreases peripheral conversion of T4 to T3, has a shorter half-life, and is drug of choice in pregnancy, as methimazole is teratogenic Marc Imhotep Cray, MD

Raff RB, Rawls SM, Beyzarov EP. Netter's Illustrated Pharmacology, Updated Edn. Saunders, 2014

239


Thionamides cont.  Methimazole is 10 times more potent than PTU, but both drugs are equally effective if given in equipotent dosages 

preferable to (PTU) except during first trimester of pregnancy and during thyroid storm

 Methimazole can be given once daily (longer t1/2), whereas PTU must be given every 6 to 8 hours (short duration)  PTU is preferred for pregnant women (methimazole teratogenic)  A clinical response usually seen after 6 to 8 weeks of therapy 

inhibit thyroid hormone synthesis but do not affect existing thyroid hormone stores (last approximately 30 days)

 The duration of therapy is usually 12 to 18 months to years Marc Imhotep Cray, MD

240


Thionamides: Adverse Effects A pruritic maculopapular rash, without other systemic symptoms, is most common adverse effect of thioamides  In mild cases, rash resolves despite therapy, or another thioamide can be used (minimal cross-sensitivity exists)  If systemic symptoms (eg, fever, arthralgias) occur (hypersensitivity rxn), thioamide therapy should be stopped Hepatotoxicity involves hepatocellular damage (with PTU) and obstructive jaundice (with methimazole)  Liver function test (LFT) results should be watched if a history of liver disease or risk for hepatitis exists Marc Imhotep Cray, MD

241


Thionamides: Adverse Effects cont. Agranulocytosis (leukopenia with much lower polymorphonuclear leukocyte numbers) is most serious adverse effect  Onset of symptoms (fever, malaise, sore throat) is quite sudden high methimazole doses may lead to greater risk o If this disorder is diagnosed, thioamide administration should be stopped, CBC w Diff. drawn and patient should be monitored for infection

Hypersensitivity reactions occur suddenly, often within first 3 months of therapy include thrombocytopenia, pancytopenia, aplastic anemia, and agranulocytosis  Thus white blood cell counts should be closely monitored  Pts. should be counseled to see physicians if they experience any flulike symptoms Marc Imhotep Cray, MD

242


Thionamides: Adverse Effects cont. Other serious effects include:  peripheral neuritis  neuropathy  taste disorders  nephrotoxicity  myopathy  arthritis  SLE

Marc Imhotep Cray, MD

Drug-Drug Interactions Drugs that induce cytochrome P450 enzymes, such as phenytoin, rifampin, and phenobarbital, accelerate metabolism of thyroid hormone and may decrease effectiveness

243


Thionamides: Special Cautions  During hyperthyroid state, other drugs metabolized by liver or eliminated renally may need to be adjusted because metabolism may be increased  Patients using drugs with a narrow therapeutic index, such as  Digoxin  Warfarin  Phenytoin should be monitored carefully b/c dosing adjustments will be necessary as hyperthyroidism or hypermetabolic state resolves Marc Imhotep Cray, MD


Question A 22-year-old white FM presents to your office with a recent onset of fever and throat pain. Her past medical history is significant for hyperthyroidism controlled with medical therapy. Her blood pressure is 110/70 mmHg and heart rate is 90/min. Physical examination is insignificant. Which of the following is the best next step in the management of this patient? A. Empirical oral antibiotics B. WBC count with differential C. ESR and anti-neutrophil antibodies D.TSH and T4/T3 levels E. Ibuprofen and oral hydration Marc Imhotep Cray, MD

245


Answer Answer is B. WBC count with differential Agranulocytosis describes an absolute neutrophil count of less than 500 /Ml. Although agranulocytosis only occurs in approximately 0.5% of cases treated with antithyroid medications, it is a serious complication because neutrophils are vital in mounting an immune response to many pathogens Patients typically present with fever and a sore throat. If thionamide-associated agranulocytosis is suspected, the drug is immediately discontinued and a white blood cell count with differential is drawn Marc Imhotep Cray, MD

246


Radioactive iodine (131I) Radioactive iodine is used for post-adolescent patients, patients with Graves ophthalmopathy or history of thyroid surgery, poor surgical candidates, and those who do not respond to thionamides  It is treatment of choice in older patients with heart disease and those with toxic multinodular goiter Maximal effects of RAI do not occur for 3 to 4 months MOA  131I, used most often, is rapidly trapped by thyroid β particles act mostly on parenchymal thyroid cells (cytotoxic), with minimal damage to adjacent tissues  Effects of radiation depend on dosage, with larger doses causing cytotoxicity  Proper RAI doses can destroy gland without injuring nearby tissues Marc Imhotep Cray, MD

247


Radioactive lodine: Adverse Effects  Major adverse effect of RAI is hypothyroidism Also, Post-RAI hyperthyroidism caused by hormones leaking from damaged thyroid can occur but is minimized by use of thioamides or β blockers before RAI (depletes gland of hormones)  Immediate adverse effects include  mild thyroid pain and  hair thinning Long-term effects include carcinogenesis and genetic damage Contraindications children and pregnant or breast-feeding women, and can induce or worsen ophthalmopathy Antidote for overdose large doses of Na+ or K+ iodate given to compete with radioiodine for thyroid uptake and to hasten excretion by increasing iodide turnover (increased fluid intake and a diuretic are adjuvants) Marc Imhotep Cray, MD

248


Iodide  Iodide (ie, Lugol solution: 5% iodine and 10% potassium iodide) is oldest known remedy for symptomatic relief of hyperthyroidism, and, before advent of pharmacologic therapy, it was sole treatment available  Today, iodide therapy has been mostly replaced by thioamides and β blockers MOA Iodides act by  blocking organification of iodine  inhibiting release of thyroid hormones, and  decreasing gland size and vascularity USE  Iodides act rapidly and produce symptomatic relief after 2 to 7 days  They are thus useful in patients with thyroid storm and  those awaiting relief from thioamide therapy (6 to 8 weeks)

Marc Imhotep Cray, MD

Wolff-Chaikoff effect: In a normal individual, giving large amounts of iodine at once will lead to transient hypothyroidism, which is counterintuitive.

249


Iodide cont. Iodides are also routinely given, preferably with thioamides, 10 to 14 days before surgery to facilitate removal of gland by reducing its size and vascularity  Iodide cannot be given before RAI because it can block retention of RAI by the gland Major adverse effects of iodide include  hypersensitivity reactions and  risk of hypothyroidism or worsening of hyperthyroidism

Marc Imhotep Cray, MD

250


Adrenergic Antagonists  Many signs and symptoms of hyperthyroidism are mediated through sympathetic nervous system logical to use adrenergic antagonists for symptomatic relief because these agents block effects of thyroid hormones on catecholamines  Adrenergic antagonists do not affect underlying disease process, so they are not used as primary therapy  they are quite useful in providing rapid symptomatic relief before thioamides, RAI, or surgery can take effect  They can also be used as adjuncts to thioamides and RAI for  neonatal thyrotoxicosis  thyrotoxicosis in pregnancy, and  thyroid storm Marc Imhotep Cray, MD

251


Adrenergic Antagonists cont. The β blocker propranolol, which reduces conversion of T4 to T3, is most widely used adrenergic antagonist  it relieves o Palpitations o tachycardia o anxiety o sweating o tremor o neuromuscular manifestations of hyperthyroidism calcium channel blocker diltiazem may be useful when propranolol should be avoided (eg, pts with asthma, CHF, diabetes) Marc Imhotep Cray, MD

252


Summary of drugs used in thyroid disease

Katzung & Trevors Pharmacology Examination and Board Review, 11th Ed. McGraw-Hill, 2015.

Marc Imhotep Cray, MD


Case 42 Answers Thyroid Hormones Summary: A 44-year-old woman is diagnosed with hypothyroidism and prescribed levothyroxine. • Levothyroxine sodium: Synthetic sodium salt of thyroxine (T4). • Derivation of T3 in the body: Approximately 75 percent from the deiodination of T4; also produced by the coupling of monoiodotyrosine (MIT) and diiodotyrosine (DIT). • Mechanism of action of thyroid hormones: Bind with receptors in nuclei of target cells and alter synthesis rates of specific messenger ribonucleoprotein acids (mRNAs), increasing production of certain proteins including Na + , K+ -ATPase. Marc Imhotep Cray, MD

254


Case 42 Answers ,Thyroid Hormones cont. CLINICAL CORRELATION

Thyroid hormones have wide-ranging effects of tissues throughout the body. They are involved primarily in the regulation of metabolism. The hypothalamicpituitary-thyroid axis regulates release of active hormone from the thyroid via a feedback loop. Thyrotropin-releasing hormone (TRH) is produced in the hypothalamus and stimulates the release of TSH from the anterior pituitary. TSH binds to membrane receptors in the thyroid and stimulates the production and release of T 4 and T 3 via a cyclic adenosine monophosphate (cAMP)-mediated system. Synthesis of T 4 exceeds T3 by approximately fourfold; most circulating T 3 comes from peripheral deiodination of T 4. T 4 and T 3 are almost entirely protein bound, mostly to thyroxine binding globulin (TBG) and albumin. Unbound thyroid hormone binds to receptors located in the nuclei of target cells. This alters transcription of specific mRNAs, which lead to the increased production of proteins, including Na +, K + -ATPase. This results in a net increase in ATP and oxygen consumption, raising the metabolic rate. Marc Imhotep Cray, MD

255


Case 42 Answers Thyroid Hormones cont. Hypothyroidism occurs when there is inadequate thyroid hormone production and release to meet the body’s metabolic demands. In primary hypothyroidism the thyroid gland is unable to synthesize adequate amounts of thyroid hormone. The pituitary releases increasing amounts of TSH to try to stimulate production, leading to the characteristic laboratory findings of low circulating levels of thyroid hormones with an elevated TSH. Conversely, primary hyperthyroidism is diagnosed by the presence of elevated thyroid hormone levels and a suppressed level of TSH. Hypothyroidism is most often treated by the oral administration of synthetic T 4 in the form of levothyroxine sodium. This replaces both T 4 and, by deiodination, T 3. Marc Imhotep Cray, MD

256


Lect. 4 of 6

Parathyroid Gland Disorders & Agents Affecting Calcium Homeostasis

Marc Imhotep Cray, MD

257


Learning Objectives Parathyroid Gland Disorders 1. Describe the location and structure of parathyroid glands 2. explain the mechanism of parathyroid hormone action and its effects 3. Describe the causes, signs, symptoms and treatment of hypoparathyroidism and hyperparathyroidism. Pharmacology of Calcium Metabolism 1. The role of key organs involved in regulation of plasma calcium concentration 2. The endocrine regulation of calcium homeostasis and mechanisms involved 3. The principles underlying the treatment of both hyper- and hypocalcemia. 4. The indications, mechanism of action, adverse effects and contraindications of the drugs used in therapy of hypo- and hypercalcemia. Marc Imhotep Cray, MD

258


Drugs That Affect Calcium Levels DRUGS THAT INCREASE CALCIUM LEVELS Calcium citrate (many other salts) Calcium gluconate Teriparatide Vitamin D Analogues Calcitriol Cholecalciferol Dihydrotachysterol Doxercalciferol Ergocalciferol Paricalcitol Marc Imhotep Cray, MD

DRUGS THAT DECREASE CALCIUM LEVELS Bisphosphonates Alendronate Ibandronate Pamidronate Etidronate Risedronate Tiludronate Zoledronic acid

Calcimimetics Cinacalcet Clodronate Estrogens and raloxifene Gallium nitrate Phosphate

Calcitonin

259


Marc Imhotep Cray, MD

Baron SJ and Lee CI. Lange Pathology Flash Cards. New York: McGraw-Hill, 2009

260


Case 44 Agents Affecting Calcium Homeostasis A 66-year-old woman presents for an annual health maintenance visit. She is generally feeling well and has no specific complaints. She takes hydrochlorothiazide for hypertension, levothyroxine sodium for hypothyroidism, and a multivitamin. She went through menopause at age 48 and never took hormone replacement therapy. She is a former cigarette smoker, having a 30 pack-year history and having quit 20 years ago. She occasionally has a glass of wine with dinner and walks three or four times a week for exercise. On examination you note that her height is 1 inch less than it was 3 years ago. Her vital signs are normal. She has a prominent kyphoscoliosis of the spine. Her examination is otherwise unremarkable. Blood work reveals normal electrolytes, renal function, blood count, calcium, and thyroidstimulating hormone (TSH) levels. You order a bone density test, which shows a significant reduction of density in the spine and hips. You diagnose her with osteoporosis and start her on alendronate sodium. _ What is the mechanism of action of PTH on the bone and in the kidney? _ What is the mechanism of action of alendronate sodium? 261


Hypoparathyroidism Definition • Low PTH levels, usually due to destruction of parathyroid glands (acquired) Etiology • Common causes: - Surgery - Infiltration and destruction of parathyroid glands (Wilson disease, hemachromatosis, and radiation) - PTH production may be suppressed in hypomagnesemia (magnesium important for PTH homeostasis) Marc Imhotep Cray, MD

262


Hypoparathyroidism (2) Clinical Presentation Laboratory • Decreased serum PTH • Hypocalcemia • Hyperphosphatemia • Normal 25-hydroxyvitamin D level • Decreased 1,25- dihydroxyvitamin D levels

Marc Imhotep Cray, MD

263


Hypoparathyroidism (3) Diagnosis • Increased urine: calcium to creatinine ratio and hypophosphaturia • ECG: prolonged Q-T interval (hypocalcemia)

Treatment  Supplementation with calcium and 1,25-dihydroxyvitamin D  Caution with intravenous calcium administration

Marc Imhotep Cray, MD

264


Hypoparathyroidism (4)  Symptoms (most due to hypocalcemia) • Seizures • Constipation • Muscle cramps • Hyperreflexia • Tetany • Abdominal pain • Lethargy • Cardiac dysrhythmia • Chvostek’s sign (facial twitching when the zygomatic arch is tapped) • Trousseau’s sign (forearm spasms induced by inflating BP cuff on upper arm)

Marc Imhotep Cray, MD

265


Hyperparathyroidism Definition • High levels of PTH levels, usually due to excessive release Types of HPT • Primary Hyperparathyroidism • Secondary Hyperparathyroidism • Tertiary Hyperparathyroidism

Marc Imhotep Cray, MD

266


Primary Hyperparathyroidism • Parathyroid adenoma is the most common cause • (85% of all hyperparathyroid cases)

• Hyperplasia of parathyroid glands • Parathyroid carcinoma (rare)

Secondary Hyperparathyroidism • Feedback response to hypocalcemia stimulates parathyroid glands leading to hyperplasia and excessive PTH production • Causes of hypocalcemia: - Renal failure is most common cause - Vitamin D deficiency - Malabsorption of intestinal calcium Marc Imhotep Cray, MD

267


Rubin R , Strayer DS Eds. Rubin’s Pathology: Clinicopathologic Foundations of Medicine, 6th Ed. Baltimore: Lippincott Williams & Wilkins, 2012. 268


Tertiary Hyperparathyroidism • Constant stimulation of parathyroids in secondary hyperparathyroidism causes autonomous secretion of PTH by gland • End result is hypercalcemia because feedback response is functional • Correction of hypercalcemia associated with tertiary HPT requires surgical resection of most of four parathyroid glands

Marc Imhotep Cray, MD

269


Clinical Presentation of HPT Laboratory • Elevated serum PTH levels • Elevated 1,25-dihydroxyvitamin D levels • Hypercalcemia • Hypophosphatemia

Marc Imhotep Cray, MD

Symptoms (most due to hypercalcemia) “Stones, groans, and psychic moans”

• Kidney stones • Abdominal pain • Bone pain • Depression • Nausea & Vomiting • Weakness • Lethargy • Hypertension 270


Diagnosis & Treatment of HPT Dx • Urine: decreased calcium to creatinine ratio and hyperphosphaturia • ECG: short Q-T interval (hypercalcemia) Treatment  Calcium binding agents  Treat underlying etiology

Marc Imhotep Cray, MD

271


Drugs That Affect Bone Mineral Homeostasis  Calcium and phosphorus, the 2 major elements of bone, are crucial not only for mechanical strength of the skeleton but also for normal function of many other cells in body  A complex regulatory mechanism has evolved to tightly regulate calcium and phosphate homeostasis

 Parathyroid hormone (PTH), vitamin D, and fibroblast growth factor 23 (FGF23) are primary regulators  calcitonin, glucocorticoids, and estrogens play secondary roles  These hormones, or drugs that mimic or suppress their actions, as well as several nonhormonal agents, are used in treatment of bone mineral disorders, including:  Osteoporosis  Rickets  Osteomalacia  Paget’s disease Marc Imhotep Cray, MD


Calcium Metabolism (Parathyroid Hormone, Vitamin D, Calcitonin) Overall Ca2+ homeostasis  40% of total Ca2+ in blood is bound to plasma proteins  60% of total Ca2+ in blood is not bound to proteins and is ultrafilterable  Ultrafilterable Ca2+ includes Ca2+ that is complexed to anions such as phosphate and free, ionized Ca2+  Free, ionized Ca2+ is biologically active  Serum [Ca2+] is determined by interplay of intestinal absorption, renal excretion, and bone remodeling (bone resorption and formation)  Each component is hormonally regulated  To maintain Ca2+ balance, net intestinal absorption must be balanced by Marc urinary Imhotep Cray,excretion MD

273


Calcium Metabolism (Parathyroid Hormone, Vitamin D, Calcitonin) cont.  Positive Ca2+ balance  is seen in growing children  Intestinal Ca2+ absorption exceeds urinary excretion, and excess is deposited in growing bones  Negative Ca2+ balance  is seen in women during pregnancy or lactation  Intestinal Ca2+ absorption is less than Ca2+ excretion, and deficit comes from maternal bones

Marc Imhotep Cray, MD

274


What are the three hormones that regulate calcium levels in blood and tissues and their origin of secretion? 1. PTH is secreted from chief cells in parathyroid glands  PTH is released in response to low serum calcium its purpose is to raise serum calcium level 2. Vitamin D is produced from diet as well as from synthesis through cholesterol with help of ultraviolet (UV) light  Its purpose is to raise serum calcium level 3. Calcitonin comes from parafollicular cells in thyroid gland  Calcitonin is secreted in response to high serum calcium and will lower serum calcium level Marc Imhotep Cray, MD

275


Parathyroid Hormone (PTH) Biosynthesis  PTH secreted by chief cells of parathyroid gland Mode of Action  PTH binds PTH receptor → activation of guanyl nucleotide regulatory protein → activation of adenylate cyclase → increased cAMP production Regulation  Increased serum calcium and increased 1,25-(OH) 2D3 levels → decreased PTH secretion PTH secretion  Decreased serum calcium → increased PTH secretion Marc Imhotep Cray, MD

276


Parathyroid Hormone (PTH) cont. Function: Overall effect is to increase serum calcium and decrease serum phosphate levels Effects on bone:  Promotes osteoclastic activity  Increases rate of skeletal remodeling Effects on kidney:  Promotes calcium reabsorption in distal tubule of nephron  Increases phosphate excretion  Increases formation of 1,25-(OH) 2D3 (activated vitamin D) Effects on intestine:  Increased 1,25-(OH) 2D3 results in increased intestinal calcium and phosphate absorption Marc Imhotep Cray, MD

277


Calcium homeostasis: PTH & Vit D

Marc Imhotep Cray, MD

McInnis M., Mehta S. Step-up to USMLE Step 1 2015 Edition. Wolters Kluwer, 2015

278


Calcium homeostasis: PTH & Vit D (2)

Marc Imhotep Cray, MD

Le T and Bhushan V. First Aid for the USMLE Step 1 2015 (McGraw-Hill 2015)

279


Calcium Regulation by Parathyroid Hormone and Vitamin D Summary Table

Miksad RA, Meyer GK & DeLaMora PA. Last Minute Internal Medicine. New York: McGraw-Hill, 2008

Marc Imhotep Cray, MD

280


Vitamin D  Vitamin D3 (cholecalciferol) is absorbed by small intestine as part of diet (e.g. dairy food) or is synthesized from cholesterol in skin  VitaminD3 synthesis requires ultraviolet B (UVB) light sun it is then converted into calcitriol Calcitriol is biologically active form of vitamin D and is a major determinant of intestinal calcium and phosphate reabsorption.  Activation  Human vitamin D is an inactive steroid called cholecalciferol (or vitamin D3) o It is a fat-soluble steroid is stored in adipose tissue  Two reactions must take place in different organs to activate vitamin D o Activated vitamin D (1,25-dihydroxycholecalciferol) Marc Imhotep Cray, MD

281


Activation of Vitamin D

Actions of vitamin D on GIT, bone (PTH, parathyroid hormone) and kidney

Horton-Szar D. Crash Course: Endocrinology, 4th Ed. Elsevier, 2012 Marc Imhotep Cray, MD

282


Calcium-relate Diseases & Disorders Osteoporosis, Paget disease, and osteomalacia are disorders of the bone  Osteoporosis is characterized by progressive loss of bone mass and skeletal fragility  Patients with osteoporosis have an increased risk of fractures, which can cause significant morbidity  Osteoporosis occurs in older men and women but is most pronounced in postmenopausal women  Paget disease is a disorder of bone remodeling that results in disorganized bone formation and enlarged or misshapen bones  Unlike osteoporosis, Paget disease is usually limited to one or a few bones  Patients may experience bone pain, bone deformities, or fractures  Osteomalacia is softening of bones that is most often attributed to vitamin D deficiency Osteomalacia in children is referred to as rickets Marc Imhotep Cray, MD

283


Changes in bone morphology in osteoporosis

Marc Imhotep Cray, MD

Whalen K. Lippincott Illustrated Reviews: Pharmacology 6th Ed. Wolters Kluwer, 2015

284


Treatment of Osteoporosis  Nondrug strategies to reduce bone loss in postmenopausal women include  adequate dietary intake of calcium and vitamin D  weight-bearing exercise, and  smoking cessation  In addition, patients at risk for osteoporosis should avoid drugs that increase bone loss such as glucocorticoids Note: Use of glucocorticoids (for example, prednisone 5 mg/day or equivalent) for 3 months or more is a significant risk factor for osteoporosis.

Marc Imhotep Cray, MD

Drugs that can contribute to bone loss or increased fracture risk. • Aluminum antacids • Anticonvulsants (e.g., phenytoin) • Aromatase inhibitors • Furosemide • Glucocorticoids • Heparin • Medroxyprogesterone acetate • Proton pump inhibitors • SSRIs • Thiazolidinediones • Thyroid (excessive replacement)

285


Treatment of Osteoporosis cont.  Pharmacologic therapy for osteoporosis is warranted in 1. postmenopausal women and men aged 50 years or over who have a previous osteoporotic fracture 2. a bone mineral density that is 2.5 standard deviations or more below that of a young adult, or 3. a low bone mass with a high probability of future fractures

Common agents used include:  Bisphosphonates: o Alendronate, Ibandronate, Risedronate, Zoledronic acid  Selective estrogen receptor modulators  Calcitonin  Denosumab  Teriparatide Marc Imhotep Cray, MD

286


Bisphosphonates* Mechanism of action  Bind to hydroxyapatite in bone, inhibiting osteoclast activity Uses  Postmenopausal bone loss 1. Alendronate (oral; once a week) 2. Risedronate (oral; once a week)  Osteoporosis and compression fractures 1. Alendronate (oral; once a week) 2. Risedronate (oral; once a week) 3. Ibandronate (oral; once a month) 4. Zoledronic acid (IV; once a year) Marc Imhotep Cray, MD

Uses cont.  Hypercalcemia due to malignancy 1. Clodronate 2. Etidronate 3. Tiludronate 4. Zoledronic acid  Paget’s disease 1. Clodronate 2. Etidronate 3. Tiludronate 4. Zoledronic acid *Note common ending -dronate for all bisphosphonates.

287


Bisphosphonates cont. Adverse effects  Reflux esophagitis (gastroesophageal reflux disease; GERD) when taken orally; avoid this by: 1. Taking these drugs on an empty stomach, with at least 8 oz. water, immediately upon awakening 2. Remaining in an upright position for at least 30 minutes after taking drug 3. Avoiding food or drink for 30 minutes after taking drug

   

Musculoskeletal pain Hypocalcemia Hypophosphatemia Osteonecrosis (jaw)

Marc Imhotep Cray, MD

288


Bisphosphonates cont. Pharmacokinetics Food and other medications decrease absorption of bisphosphonates, which are already poorly absorbed (less than 1%) after oral administration

Bisphosphonates are cleared from plasma by binding to bone and being cleared by kidney  not metabolized by CYP450 system Elimination half-life may be years Marc Imhotep Cray, MD

289


Selective estrogen receptor modulators (SERMs) Raloxifene MOA  Agonist in bone  Antagonist in breast  Antagonist in uterus Uses  Prevention and treatment of osteoporosis in postmenopausal women  Risk reduction for invasive breast cancer in postmenopausal women with osteoporosis  Risk reduction in postmenopausal women with high risk for invasive breast cancer Marc Imhotep Cray, MD

Adverse effects  Thromboembolism  Peripheral edema  Hot flashes  Headache  Depression  Vaginal bleeding

290


Denosumab MOA  Denosumab is a monoclonal antibody that targets receptor activator of nuclear factor kappa-B ligand and inhibits osteoclast formation and function Use  Denosumab is approved for treatment of postmenopausal osteoporosis in women at high risk of fracture  It is administered via subcutaneous injection every 6 months Adverse Effects  increased risk of infections  dermatological reactions  hypocalcemia  osteonecrosis of the jaw  atypical fractures  Reserved for women at high risk of fracture and those who are intolerant of or unresponsive to other osteoporosis therapies Marc Imhotep Cray, MD

291


Calcitonin Uses  Administered parenterally to treat hypercalcemia  Paget’s disease of bone  Postmenopausal osteoporosis (intranasal) Adverse effects a. Rhinitis b. Flushing c. Back pain

Marc Imhotep Cray, MD

Teriparatide (a PTH analogue) A recombinant segment of human parathyroid hormone administered subcutaneously for Tx of osteoporosis MOA Causes dissolution of bone but can more commonly cause bone formation Uses  Osteoporosis in postmenopausal women at high risk of fracture  Primary or hypogonadal osteoporosis in men at high risk of fracture Adverse effects  Hypercalcemia  Hyperuricemia  Arthralgia  Respiratory effects

292


Vitamin D analogues Uses  Treatment of vitamin D deficiency  Prophylaxis against vitamin D deficiency  Rickets prevention

Examples  Calcitriol  Cholecalciferol  Dihydrotachysterol o Given with calcium to supplement diet of infants  Doxercalciferol  Hypoparathyroidism (with calcium supplements)  Ergocalciferol  Osteoporosis  Paricalcitol o Prevention and treatment

 Chronic renal disease 1. Calcitriol 2. Paricalcitol (Oral and IV)

Marc Imhotep Cray, MD

293


Question OP is a 65-year-old female who has been diagnosed with postmenopausal osteoporosis. She has no history of fractures and no other pertinent medical conditions. Which of the following would be most appropriate for management of her osteoporosis? A. Alendronate B. Calcitonin C. Denosumab D. Raloxifene E. Teriparatide

Marc Imhotep Cray, MD

294


Correct answer = A Bisphosphonates are first-line therapy for osteoporosis in postmenopausal women without contraindications. Calcitonin and raloxifene are alternatives but may be less efficacious (especially for nonvertebral fractures). Teriparatide and denosumab should be reserved for patients at high risk or those who fail other therapies.

Marc Imhotep Cray, MD

295


Estrogen or hormonal replacement therapy (HRT) Evidence-based medicine (EBM) states that overall health risks from HRT in postmenopausal women appear to exceed possible benefits Mechanism of action  Reduces bone resorption Uses  Postmenopausal osteoporosis (reduces bone loss)  Cannot restore bone Adverse effects  Similar to oral contraceptives but to a lesser extent because of lower estrogen content  The Women’s Health Initiative (WHI) Trial reported an increase in incidence of strokes in both estrogen-alone and the estrogen-progestin subgroups as compared with placebo groups.  Thromboembolism Marc Imhotep Cray, MD

296


Case 44 Answers Agents Affecting Calcium Homeostasis Summary: A 66-year-old woman with osteoporosis is prescribed alendronate. • Mechanism of action of PTH on the bone: Pulsatile administration, the normal physiologic mode, enhances bone formation. Continuous delivery, for example, as a consequence of a parathyroid tumor, results in bone resorption. • Mechanism of action of PTH in the kidney: Increases reabsorption of Ca 2+ and Mg2+ and increases production of vitamin D and the active metabolite calcitriol and decreases reabsorption of phosphate, bicarbonate, amino acids, sulfate, sodium, and chloride. • Mechanism of action of alendronate sodium: Inhibition of osteoclastic activity in bone, which reduces bone reabsorption. Marc Imhotep Cray, MD

297


Case 44 Answers Agents Affecting Calcium Homeostasis cont. CLINICAL CORRELATION PTH has multiple actions on bone. Chronic elevations in PTH, for example, from a tumor, stimulate the resorption of bone via its stimulation of the number and activity of osteoclasts. This is mediated by specific PTH receptors in the bone, coupled to an increase in cyclic adenosine monophosphate (cAMP). Intermittent administration of PTH stimulates bone growth. Estrogen is an indirect inhibitor of PTH activity in the bone. This effect allows premenopausal women to maintain higher levels of bone density. Following menopause, with the resultant decrease in circulating estrogen levels, there is a relative increase in osteoclastic activity and resorption of bone, with a net loss of bone mineral density. Marc Imhotep Cray, MD

298


Case 44 Answers Agents Affecting Calcium Homeostasis cont. Alendronate sodium is an analog of pyrophosphate that directly binds to bone. It inhibits osteoclastic activity, reducing the resorption of bone. This retards the progression of bone density loss and may allow for increases in density, because osteoblastic activity is not affected. It is administered orally, and its most common adverse effects are gastrointestinal (GI). It may produce esophagitis, and even esophageal perforation, if the pill were to get caught in the esophagus while swallowing. For that reason, patients taking alendronate are instructed to take it on an empty stomach with a full glass of water and to remain upright for at least 30 minutes after ingesting the medication. Marc Imhotep Cray, MD

299


Lect. 5 of 6

Corticosteroids & Adrenocortical Dysfunction Corticosteroids are steroid hormones produced by adrenal cortex. They consist of 2 major physiologic and pharmacologic groups are: (1) glucocorticoids, which have important effects on intermediary metabolism, catabolism, immune responses, and inflammation; and (2) mineralocorticoids, which regulate sodium and potassium reabsorption in collecting tubules of kidney. This lecture focuses on the most common disease states that result from over- or underproduction of corticosteroids and the pharmacology of glucocorticoid and mineralocorticoid agonists and antagonists. Marc Imhotep Cray, MD

300


Learning Objectives Adrenocorticosteroids 1. The role of ACTH and the HPA axis in regulation of corticosteroid synthesis. 2. The principal physiological responses to both glucocorticoids and mineralocorticoids, especially the role of cortisol and exogenous glucocorticoids in the negative feedback suppression of the HPA axis. 3. The use of synthetic glucocorticoids and mineralocorticoids drugs in the treatment of adrenal deficiency diseases such as adrenal insufficiency and congenital adrenal hyperplasia 4. The mechanism of action of glucocorticoid drugs and their pharmacological use in the treatment of non-endocrine diseases e.g. rheumatoid arthritis, asthma, inflammation and cancer.

Marc Imhotep Cray, MD

301


Learning Objectives Adrenocorticosteroids cont. 5. The major adverse effects associated with the clinical use of glucocorticoids. 6. The concept that abrupt withdrawal of chronic glucocorticoid therapy can lead to acute adrenal crisis due to atrophy of the adrenal cortex and subsequent deficiency in endogenous cortisol production. 7.The use of cortical synthesis inhibitors such as ketoconazole, metyrapone, aminoglutethimide and mitotane in the treatment of Cushing’s disease.

Marc Imhotep Cray, MD

302


Common Adrenal Corticosteroids & Inhibitors CORTICOSTEROIDS Betamethasone Cortisone/Hydrocortisone Dexamethasone Fludrocortisone Methylprednisolone Prednisolone Prednisone Triamcinolone

Marc Imhotep Cray, MD

INHIBITORS OF ADRENOCORTICOID BIOSYNTHESIS OR FUNCTION Eplerenone Spironolactone Ketoconazole Metyrapone Aminoglutethimide Mitotane Mifepristone

303


Marc Imhotep Cray, MD

Baron SJ and Lee CI. Lange Pathology Flash Cards. New York: McGraw-Hill, 2009

304


Case 41 Adrenal Cortex A 45-year-old man presents for the evaluation of weight gain. He has noticed a 20-lb weight gain in the past few months without any change in his diet or activity level. He has started developing “stretch marks” on his abdomen as well. His wife has noted that even his face seems to be “growing fatter.” Review of systems is significant for complaints of fatigue, multiple recent upper respiratory infections, and the development of facial acne. He has no significant medical history and takes no medications. There is a family history of diabetes and hypertension. On examination, his blood pressure is elevated at 165/95 mm Hg, but his other vital signs are normal. His face is plethoric, and he has a small fatty hump developing on his upper back. His abdomen is obese but soft and nontender without masses or fluid. Skin examination is notable for moderate facial acne and multiple violaceous striae on the abdomen. Blood tests show an elevated glucose level of 150 mg/dL, normal electrolytes, and renal function. His thyroid function tests are normal. You suspect idiopathic Cushing disease and order a dexamethasone suppression test to assist with confirming the diagnosis. _ Which pituitary hormone stimulates the release of adrenocortical steroids? _ What is the major glucocorticoid produced in the adrenal glands? _ What is the major mineralocorticoid produced in the adrenal glands? _ What is the major effect of mineralocorticoids? Marc Imhotep Cray, MD

305


Adrenal corticosteroids (Prototype drug, hydrocortisone)

Note: Cortisol is used when describing endogenous hormone secretion, and hydrocortisone when describing exogenous drug administration.

 Adrenal corticosteroids are available for physiological replacement therapy, in  Primary Adrenal Insufficiency (Addison’s disease)  Secondary Adrenal Insufficiency and  Congenital Adrenal Hyperplasia however,  Their chief use in medicine is for their anti-inflammatory and immunosuppressive effects (pharmacotherapy)  Supraphysiological doses are required to achieve these pharmacological effects  Chronic use of supraphysiological doses has many adverse Marc Imhotepeffects Cray, MD

306


Hormones of adrenal gland (cortex)  Principal hormone is glucocorticoid cortisol secreted from largest zone, fasciculata; mineralocorticoid aldosterone is secreted by glomerulosa, and a number of androgens and estrogens are secreted by zona reticularis

Zones of adrenal gland and hormones they secrete.

 Hypothalamic–pituitary system, through corticotropin releasing factor (CRF) and ACTH, controls cortisol and, to a lesser extent, aldosterone secretion  synthesis and secretion of aldosterone is regulated mainly by renin–angiotensin system (ATII) , and by variation in plasma K + (minor) levels Bennett PN, Brown MJ, Sharma P. Clinical Pharmacology11th Edn. Elsevier, 2012. Marc Imhotep Cray, MD

307


Hormones of adrenal gland cont.  Adrenal cortex (derived from mesoderm) & Adrenal medulla (derived from neural crest)  Adrenal cortex think GFR:  Glomerulosa (Na+)  Fasciculata (glucocorticoids)  Reticularis (androgens) Mnemonic To remember hormones produced by each layer “the deeper you go, the sweeter it gets”: • Mineralocorticoid (salt hormone) o aldosterone • Glucocorticoid (sugar hormone) o hydrocortisone • Androgen (sex hormone) McInnis M., Mehta S. Step-up to USMLE Step 1 2015 Edition. Wolters308 Kluwer, 2015 o dehydroepiandrosterone


Normal adrenal gland, gross

Klatt EC. Robbins and Cotran Atlas of Pathology, 3rd Ed. Philadelphia: Saunders, 2015.

309


Normal adrenal gland, microscopic

Klatt EC. Robbins and Cotran Atlas of Pathology, 3rd Ed. Philadelphia: Saunders, 2015.

310


Comparison of atrophic, normal, and hyperplastic adrenal glands

Klatt EC. Robbins and Cotran Atlas of Pathology, 3rd Ed. Philadelphia: Saunders, 2015.

311


Question How would we expect plasma aldosterone levels to be affected in a patient with secondary (pituitary) adrenal insufficiency?

Marc Imhotep Cray, MD

Young FW. The Netter Collection of Medical Illustrations Vol 2. The Endocrine System. Philadelphia: Saunders, 2011.

312


Answer Aldosterone secretion is primarily stimulated by angiotensin II and increase serum potassium levels. ACTH has little influence on aldosterone secretion, so this hormone will continue to be secreted at normal levels.

Marc Imhotep Cray, MD

Young FW. The Netter Collection of Medical Illustrations Vol 2. The Endocrine System. Philadelphia: Saunders, 2011.

313


Adrenocorticotropic hormone (ACTH) or Corticotropin  ACTH is a peptide hormone synthesized and secreted by anterior pituitary  Released in pulses with diurnal rhythm (highest amount released at around 6 am and lowest in evening)  Its secretion is stimulated by stress, whereas cortisol acts via negative feedback loop to suppress its release  Its major endocrine function is to stimulate the synthesis and release of cortisol from adrenal cortex Marc Imhotep Cray, MD

314


ACTH (Corticotropin) cont.  Synthetic corticotropin-derivative is used clinically to assess adrenocortical status  

Synthetic human ACTH1-24 is called cosyntropin In adrenocortical insufficiency, adrenocortical response to ACTH administration is reduced

 ACTH is a single polypeptide of 39-amino acid 

Amino acids 1 to 24 is required for full biological activity

 Amino terminal sequence (1–13) of ACTH is identical to alphamelanocyte-stimulating hormone (α-MSH) 

Thus, excess secretion of ACTH from pituitary causes hyperpigmentation due to its α-MSH activity

Marc Imhotep Cray, MD

315


ACTH Synthesis [A pro-opiomelanocortin(POMC)-derived melanocortins]

Marc Imhotep Cray, MD

316


Corticotropin cont.  Pharmacokinetics  Porcine and synthetic corticotropin is well absorbed by intramuscular administration  Its half-life is less than 20 minutes and extensively gets metabolized  Pharmacodynamics  Corticotropin stimulates adrenal cortex to produce glucocorticoid, mineralocorticoid and androgen and promotes adrenal hypertrophy and hyperplasia  Corticotropin may cause increased skin pigmentation  Mechanism of Action  ACTH binds to specific receptors on cell surface of adrenal cortex this activates G-protein-coupled processes to increase cAMP which in turn stimulates rate-limiting step (cholesterol to pregnenolone catalyzed by desmolase) in synthesis of adrenocorticosteroids and adrenal androgens Marc Imhotep Cray, MD

317


Corticotropin (ACTH) cont.  Clinical Uses  Mainly used for diagnosis of adrenal-insufficiency cosyntropin (synthetic human ACTH) may be used to rule out adrenal-insufficiency o In normal individuals following cosyntropin administration, plasma cortisol should exceed 18 μg/dL o A subnormal response is indicative of primary or secondary adrenocortical insufficiency • In primary adrenocortical insufficiency (Addison’s disease associated with adrenal atrophy) endogenous plasma ACTH level is elevated • In secondary adrenocortical insufficiency endogenous plasma ACTH level is decreased

 Adverse Effects  Adverse effects of ACTH use are similar to those of glucocorticoids Marc Imhotep Cray, MD

318


Steroid metabolism (steroidogenesis) Biosynthesis of steroids

P450scc

Enzymes with prefix CYP represent mitochondrial cytochrome P450 mixed function oxidases, and numbers indicate site of steroid hydroxylation. Steroids indicated in bold are primary secreted steroids.

Marc Imhotep Cray, Lynn MD Wecker et.al. Brody’s Human Pharmacology: Molecular to Clinical, 5th Ed. Philadelphia: Mosby, 2010

319


ACTH effects on adrenal steroidogenesis  Target organ of ACTH is adrenal cortex, where it binds to specific receptors on cell surfaces  Occupied receptors activate G protein– coupled processes to increase cyclic adenosine monophosphate (cAMP), which in turn stimulates rate-limiting step in adrenocorticosteroid synthetic pathway (cholesterol to pregnenolone) ACTH, adrenocorticotropic hormone; ATP, adenosine triphosphate; CRE, cAMP response element; CREB, cAMP response element binding; MRAP, MC2R-accessory protein; StAR, steroidogenic acute regulatory [protein]


Question Regarding the release of ACTH from the anterior pituitary in a 24year-old man who is a marathon runner, which of the following is the next step in the process? (A) Activation of protein S (B) Increase of cGMP (C) Stimulation of conversion of cholesterol to pregnenolone (D) Release of adrenocorticosteroids (E) Synthesis of pulmonary ACTH

Marc Imhotep Cray, MD

321


The answer is C: Stimulation of conversion of cholesterol to pregnenolone. The target organ of ACTH is the adrenal cortex, where it binds to specific receptors on the cell surfaces. The occupied receptors activate G protein窶田oupled processes to increase cyclic adenosine monophosphate (cAMP), which in turn stimulates the rate-limiting step in the adrenocorticosteroid synthetic pathway (cholesterol to pregnenolone). This pathway ends with the synthesis and release of the adrenocorticosteroids and the adrenal androgens. Incorrect answers rational (A) The occupied receptors activate G protein窶田oupled processes. (B) The activated G protein complex increases cyclic AMP. (D) Release of adrenocorticosteroids occurs after they are synthesized. (E) Pulmonary ACTH is produced in disease states such as oat cell carcinoma. Marc Imhotep Cray, MD

322


Adrenal steroids and congenital adrenal hyperplasia

Marc Imhotep Cray, MD

Le T and Bhushan V. First Aid for the USMLE Step 1 2015 (McGraw-Hill 2015)

323


Adrenal steroids and congenital adrenal hyperplasia (2)

Le T and Bhushan V. First Aid for the USMLE Step 1 2015 (McGraw-Hill 2015) Marc Imhotep Cray, MD

324


Congenital adrenal hyperplasia

Rubin R , Strayer DS Eds. Rubin’s Pathology: Clinicopathologic Foundations of Medicine, 6th Ed. Baltimore: Lippincott Williams & Wilkins, 2012. 325


Physiologic, Biochemical & Immunologic Effects of Cortisol ↑Blood pressure:  Upregulates α1-receptors on arterioles↑sensitivity to norepinephrine and epinephrine  At high concentrations, can bind to mineralocorticoid (aldosterone) receptors

↑Insulin resistance (diabetogenic) ↑Gluconeogenesis, lipolysis, and proteolysis “Cortisol is a BIG FIB” ↓Fibroblast activity (causes striae) ↓Inflammatory and Immune responses:  Inhibits production of leukotrienes and prostaglandins  Inhibits WBC adhesion neutrophilia (neutrophil demargination)  Blocks histamine release from mast cells  Reduces eosinophils N.B Exogenous corticosteroids can cause  Blocks IL-2 production reactivation of TB and candidiasis (blocks IL-2 ↓Bone formation (osteoblast activity) production), as IL-2 Stimulates growth of helper, Marc Imhotep Cray, MD

cytotoxic, and regulatory T cells, and NK cells.

326


Regulation of Adrenal Hormones The 2 adrenal glands in human body are responsible for producing  mineralocorticoids (eg, aldosterone) regulate fluid and electrolyte balance  glucocorticoids (eg, cortisol) which are essential for carbohydrate metabolism  Much successful effort has gone into separating glucocorticoid from mineralocorticoid effects

 Aldosterone production is mediated primarily by reninangiotensin system and serum potassium levels Cortisol production is regulated by a feedback mechanism involving hypothalamic-pituitary-adrenal (HPA) axis Marc Imhotep Cray, MD

327


Mineralocorticoids and Glucocorticoids  Mineralocorticoids enhance reabsorption of sodium and water from distal tubule of kidney and increase urinary potassium and hydrogen ion excretion

 Principal function of glucocorticoids involves regulation of carbohydrate metabolism  glucocorticoids are also involved in other physiologic actions, including: o gluconeogenesis o glucose utilization o lipid and bone metabolism o fluid and electrolyte homeostasis o alteration of levels of various immune cells o alleviation of inflammatory response, and o participation in neuropsychiatric functions Marc Imhotep Cray, MD

328


Glucocorticoid functions cont. As a result of these functions—most notably immunosuppressive and antiinflammatory actions (a direct result of immunosuppressive effects)— glucocorticoids are widely used in treatment of  cancer N.B. Supraphysiological doses are  autoimmune disorders generally required to achieve  inflammatory disorders such as these pharmacological effects.  asthma Chronic use of supraphysiological  inflammatory bowel disease doses has many adverse effects.  arthritis, and  allergies Marc Imhotep Cray, MD

329


Glucocorticoid functions cont. Target Tissue Effect  Increases blood glucose Liver Increases hepatic glycogen Liver Protein catabolism Muscle Reduces inflammation and suppresses Immune responses Macrophages and lymphocytes Stimulates HCL secretions Stomach Stimulates gluconeogenesis Liver Stimulates lipolysis Adipose

Marc Imhotep Cray, MD

Raff RB, Rawls SM, Beyzarov EP. Netter's Illustrated Pharmacology, Updated Edn. Saunders, 2014

330


Corticosteroids Block Inflammatory Mediators

Marc Imhotep Cray, MD

Le T and Bhushan V. First Aid for the USMLE Step 1 2015 (McGraw-Hill 2015)

331


Corticosteroids  Therapeutic corticosteroids (eg, hydrocortisone, prednisone, dexamethasone), with different mineralocorticoid and glucocorticoid activities*, are antiinflammatory and immunosuppressive via inhibiting immune cells  reduces formation, release, and activity of inflammation mediators (eg, cytokines, histamine, prostaglandins, leukotrienes)

 Short-term therapy adverse effects include *Relative potencies for glucocorticoid  Insomnia and mineralocorticoid (Na+-retaining)  euphoria, and increased appetite effects are central to choice of agent  Long term therapy adverse effects include in relation to clinical indication.  Osteoporosis  Muscle wasting  Hypertension  Growth suppression  Edema  Cataracts  Hyperglycemia  Peptic ulcer disease  Cushing-like syndrome  Hypokalemia  Increased risk of infections 332


Corticosteroids cont.  Most important differences between these drugs consist of duration of action and degree of inherent mineralocorticoid activity causes sodium and fluid retention  Both cortisone and hydrocortisone have glucocorticoid and mineralocorticoid properties  Their synthetic analogs prednisone, prednisolone, and methylprednisolone have both effects as well, although glucocorticoid effects predominate  By contrast, triamcinolone, dexamethasone, and betamethasone have exclusively glucocorticoid antiinflammatory activity  Mineralocorticoid activity is needed in adrenal insufficiency but not in severe inflammation Marc Imhotep Cray, MD

333


Relative potencies of adrenal steroids

Whalen K, Finkel R & Panavelil TA. Lippincott Illustrated Reviews-Pharmacology 6th Ed. : Wolters Kluwer, 2015 Marc Imhotep Cray, MD

334


Corticosteroids cont.  Long-term drug use can suppress the HPA axis and abrupt stopping of therapy can cause possibly fatal acute adrenal insufficiency syndrome “adrenal crisis”  Slow dosage tapering allows HPA axis to begin functioning and thus prevention of adrenal crisis  To reduce systemic absorption and side effects, drugs can be given topically or by inhalation or nasal spray, intraarticular injection, or rectal suppository

 Alternate-day and lowest effective dosing may limit side effects and adrenal atrophy Marc Imhotep Cray, MD

335


Glucocorticoids Mechanisms of Action Glucocorticoids stimulate cell through both a classical cytosolic receptor that, on binding with agonist, translocates to nucleus, and an unidentified membrane-bound receptor The classical receptor is responsible for so-called genomic effects through either activation or repression of DNA transcription Up-regulation of gene transcription occurs when receptor dimerizes on specific DNA glucocorticoid response elements (GREs) with consequent recruitment of coactivator proteins  Many of undesired effects of glucocorticoid occur through this pathway Marc Imhotep Cray, MD

336


MOA of Glucocorticoids cont.  Repression of DNA transcription occurs at slightly lower [cortisol] than required for transactivation  Through protein–protein interaction, glucocorticoid–receptor complex inactivates pro-inflammatory transcription factors such as nuclear factor (NF)-kB and activator protein 1 (AP-1) preventing their stimulation of inflammatory mediators:  prostaglandins  leukotrienes  cytokines and  platelet-activating factor  These mediators would normally contribute to increased vascular permeability and subsequent changes including edema, leucocyte migration and fibrin deposition Marc Imhotep Cray, MD

337


Molecular-cellular MOA of Glucocorticoids Intracellular MOA of GC Receptor  Figure shows molecular pathway by which cortisol (labeled S) enters cells and interacts with glucocorticoid receptor (GR) to o change GR conformation (indicated by change in shape of the GR) o induce GR nuclear translocation o activate transcription of target genes

Marc Imhotep Cray, MD

Brunton LL, Chabner BA , Knollmann BC (Eds.). Goodman and Gilman’s The Pharmacological Basis of Therapeutics. 12th ed. McGraw-Hill, 2011

338


Three MOA of GCs and GR in inhibition of inflammation: 1. Non-genomic activation 2. DNA-dependent regulation, and 3. Protein interference mechanisms (e.g. NF-kB elements) Red arrows denote activation, blue arrow repression, and red X lack of product (i.e. no mRNA).

-----HSP, heat-shock protein; mRNA, messenger RNA; NF, nuclear factor; P, phosphate; TNF, tumor necrosis factor.

See: Rhen T, Cidlowski J A 2005 Antiinflammatory action of glucocorticoids – new mechanisms for old drugs. New England Journal of Medicine 353(16):1711–1723.) Marc Imhotep Cray, MD

Bennett PN, Brown MJ, Sharma P. Clinical Pharmacology11th Edn. Elsevier, 2012.

339


Question A 37-year-old kidney transplant recipient presents to her primary care physician for follow-up. Among other immunosuppressant drugs, she has been taking daily prednisone for the past 2 months since her transplant. With only a few doses of prednisone left, she gets snowed into her house and cannot refill her prescription (but she has enough of the other medications to last a few more weeks). If she runs out of prednisone and cannot get it refilled, what is she most at risk for developing? (A)Cardiovascular collapse (B) Osteoporosis (C) Increased risk of infection (D) Insomnia (E)Imhotep Nausea/vomiting Marc Cray, MD

340


The correct answer is A: Cardiovascular collapse (adrenal crisis) Chronic use of glucocorticoids (such as prednisone) will lead to adrenal atrophy because exogenous steroid suppresses hypothalamic-pituitaryadrenal (HPA) axis. If exogenous steroid is abruptly withdrawn, atrophied adrenal gland is unable to compensate by producing endogenous steroids quickly enough. Sudden loss of adrenal steroids is termed “adrenal crisis”  can result in cardiovascular collapse and death. Incorrect answers (B) Osteoporosis is a possible result of continued chronic glucocorticoid therapy, not abrupt cessation. (C) Increased risk of infection is a result of continued chronic glucocorticoid therapy, not abrupt cessation. (D) Insomnia is a possible side effect from short-term oral or parenteral glucocorticoid therapy. (E) Nausea/vomiting are possible side effects from short-term oral or parenteral glucocorticoid therapy. Marc Imhotep Cray, MD

341


Question  What adrenal disease should be suspected in a young patient with bacterial meningitis due to Neisseria meningitidis who also becomes acutely hypotensive?  Sn & Sx:  septicemia, hypotension  disseminated intravascular coagulation (DIC)  adrenal hemorrhage, and  petechial rash Marc Imhotep Cray, MD

Raff RB, Rawls SM, Beyzarov EP. Netter's Illustrated Pharmacology, Updated Edn. Saunders, 2014


Waterhouse-Friderichsen syndrome Ques. What adrenal disease should be suspected in a young patient with bacterial meningitis due to Neisseria meningitidis who also becomes acutely hypotensive? Ans. Waterhouse-Friderichsen syndrome typically causes bilateral adrenal hemorrhage, which can be rapidly fatal. Responsible bacterium is Neisseria meningitidis Sn & Sx:  septicemia, hypotension  disseminated intravascular coagulation (DIC)  adrenal hemorrhage, and  petechial rash Marc Imhotep Cray, MD


Waterhouse-Friderichsen syndrome, gross

Klatt EC. Robbins and Cotran Atlas of Pathology, 3rd Ed. Philadelphia: Saunders, 2015.

344


Waterhouse-Friderichsen syndrome, CT

Klatt EC. Robbins and Cotran Atlas of Pathology, 3rd Ed. Philadelphia: Saunders, 2015.

345


Cushing Syndrome Cushing syndrome is a group of clinical symptoms that result from prolonged exposure to excess glucocorticoids  May be caused by exogenous factors, such as long-term corticosteroid use, or  May be of endogenous origin due to either o excess ACTH secretion (ACTH dependent) = Cushing Disease or o autonomous cortisol hypersecretion (ACTH independent)  Conditions such as adrenocortical adenomas and carcinomas as well as ectopic ACTH and CRH syndromes are responsible for endogenous syndrome Marc Imhotep Cray, MD

346


Etiology Cushing Syndrome Increase cortisol due to a variety of causes: 1. Exogenous corticosteroids—result in ACTH, bilateral adrenal atrophy (most common cause) 2. Primary adrenal adenoma, hyperplasia, or carcinoma—result in ACTH, atrophy of uninvolved adrenal gland 3. ACTH-secreting pituitary adenoma (Cushing disease) or paraneoplastic ACTH secretion (eg, small cell lung cancer, bronchial carcinoids)—result in ACTH, bilateral adrenal hyperplasia

Marc Imhotep Cray, MD

Note: Cushing disease is responsible for majority of endogenous cases of Cushing syndrome.

347


Hypothalamic-pituitary-adrenal axis in health and disease A. In normal hypothalamic-pituitary–adrenal axis, hypothalamic secretion of CRH stimulates release of ACTH ACTH, in turn, stimulates synthesis and secretion  of cortisol by adrenal cortex  Cortisol inhibits further release of CRH and ACTH B. A primary adrenal tumor causes Cushing’s syndrome by autonomously producing cortisol ( thick line ), independent of regulation by ACTH  excessive cortisol production suppresses ACTH production ( dashed line )

Golan DE et.al. Principles of Pharmacology: The Pathophysiologic 348 2012. Basis of Drug Therapy 3rd Ed. Lippincott Williams & Wilkins,


Hypothalamic-pituitary-adrenal axis in health and disease (2) C. An ACTH-producing pituitary adenoma causes Cushing’s disease by autonomously secreting excessive levels of ACTH ( thick line ), which stimulate adrenal gland to produce increased levels of cortisol ( thick line )  ACTH secretion by tumor has a blunted sensitivity to feedback inhibition by cortisol D. An ectopic ACTH-secreting tumor (small cell carcinoma of lung) also stimulates adrenal gland to produce increased levels of cortisol, which suppress pituitary ACTH production However, circulating ACTH levels remain elevated due to ectopic-source production of hormone

Golan DE et.al. Principles of Pharmacology: The Pathophysiologic 349 2012. Basis of Drug Therapy 3rd Ed. Lippincott Williams & Wilkins,


Adrenal adenoma, gross

Klatt EC. Robbins and Cotran Atlas of Pathology, 3rd Ed. Philadelphia: Saunders, 2015.

350


Findings in Cushing Syndrome  Clinical manifestations affect multiple organ systems and depend on degree and duration of hypercortisolism  Most common sign is progressive obesity seen in face, neck, trunk, and abdomen  Facial fat accumulation produces a moon-face appearance, enlarged dorsocervical fat pad produces a buffalo hump  Other symptoms include: o weight gain o weakness o muscle wasting (reduced arm muscle mass) o osteoporosis o cardiovascular (hypertension) o hyperglycemia (insulin resistance) o amenorrhea o immunosuppression

351


A young patient from original series of Harvey Cushing Left: before onset of disease. Right: after development of Cushing’s Syndrome.

Marc Imhotep Cray, MD

Widmaier EP, Raff H & Strang KT. Vander’s Human Physiology : The Mechanisms of Body Function, 11th ed. New York, NY: McGraw-Hill, 2008.


Clinical features of Cushing's syndrome A Note central obesity and broad, purple stretch marks B close-up C Note thin and brittle skin in an elderly pt. with Cushing's. D Hyperpigmentation of knuckles in a pt. with ectopic ACTH excess


Diagnosis of Cushing Syndrome  Screening tests include:  increase free cortisol on 24-hr urinalysis  increase midnight salivary cortisol, and no suppression with overnight low-dose dexamethasone test

 Measure serum ACTH  

If decrease suspect adrenal tumor or exogenous glucocorticoids If increase distinguish between Cushing disease and ectopic ACTH secretion with a high-dose (8 mg) dexamethasone suppression test and CRH stimulation test o Ectopic secretion will not decrease with dexamethasone because source is resistant to negative feedback o Ectopic secretion will not increase with CRH because pituitary ACTH is suppressed

Marc Imhotep Cray, MD

354


Site of Action of Cushing Syndrome (hypercortisolism)

Agents Aminoglutethimide Metyrapone Ketoconazole Mitotane= an adrenolytic

Kester M & Vrana KE. Elsevier's Integrated Review: Pharmacology, 2nd Edn. Philadelphia: Saunders, 355 2012.


Ketoconazole  Therapy for exogenous Cushing syndrome consists of minimizing exposure to glucocorticoids or ACTH  For the endogenous syndrome, therapy aims to reduce cortisol production in preparing patients for surgery or to maintain normal plasma cortisol levels until full effects of surgery or radiation are felt  Ketoconazole (an antifungal agent) is used to treat paraneoplastic Cushing syndrome secondary to ectopic ACTH production MOA highly effective in decreasing cortisol by inhibiting adrenocortical cytochrome P-450–dependent enzymes  At much higher doses than normally used for antifungal activity, ketoconazole, inhibits both 11- and 17-hydroxylases o These enzymes normally catalyze formation of cortisol precursors such as pregnenolone as well as metabolizing drugs Marc ImhotepoCray,(See MD NIP Figure 5-21 and steroidogenesis schematic online)

356


Ketoconazole cont.  Drug-Drug Interactions Because ketoconazole is a cytochrome P-450–enzyme inhibitor  can increase levels of many hepatically metabolized agents, such as    

cyclosporine warfarin digoxin, and phenytoin

 Side effects of ketoconazole include       

blood dyscrasias headache dizziness fatigue gynecomastia GI symptoms, and rash

 Patients respond to therapy after 4 to 6 weeks Marc Imhotep Cray, MD

357


Metyrapone  Metyrapone is used to treat Cushing syndrome when dose limiting side effects occur with ketoconazole  and it can be used in combination with other agents

 Metyrapone is also used as a test for adrenal function  MOA It reduces cortisol production by inhibiting 11-β-hydroxylation final step in glucocorticoid synthesis process leads to accumulation of adrenal androgens and 11-deoxycorticosterone (the potent mineralocorticoid)  Resultant adverse effects include    

Water retention GI disturbances, and Dizziness Nausea & Vomiting

 Hirsutism and acne (enhanced production of androgens resulting from cortisol blockage)  Hypertension (because of accumulation of 11-deoxycortisol)

o Dose reduction can limit these adverse effects  Metyrapone may take up to 4 months to produce a response Marc Imhotep Cray, MD

358


Aminoglutethimide Aminoglutethimide is used primarily for  Cushing syndrome secondary to adrenal hyperplasia  ectopic ACTH production, or  adrenal carcinoma o most useful when given after pituitary irradiation or in combination with metyrapone MOA inhibits cholesterol desmolase, first and rate-limiting step responsible for converting cholesterol to pregnenolone  Also blocks conversion of androstenedione to estrone and estradiol in peripheral tissues Marc Imhotep Cray, MD

359


Aminoglutethimide cont.  Inhibition interrupts production of cortisol, aldosterone, and estrogens  A reflex increase in ACTH results, which partly or completely overcomes blockade this reflex can be prevented by replacement amounts of hydrocortisone (but not dexamethasone) given concomitantly

 Adverse effects include         

headache sedation dizziness nausea anorexia rash blood dyscrasias tachycardia, and hypertension

 This drug may take up to 4 months to produce a response Marc Imhotep Cray, MD

360


Mitotane  MOA Adrenolytics (drugs that cause atrophy of adrenal gland)  Mitotane therapy typically requires hospitalization so plasma and urinary cortisol levels can be closely monitored  Steroid replacement therapy may be required in these patients Adverse Effects  Lethargy, somnolence, and other adverse central nervous system effects are experienced by most patients

Marc Imhotep Cray, MD

361


Corticosteroid Antagonists Receptor Antagonists ď ą Spironolactone and eplerenone, antagonists of aldosterone at its receptor (discussed in renal pharmacology module in connection with diuretics) ď ą Mifepristone (RU-486) is a competitive inhibitor of glucocorticoid receptors as well as progesterone receptors (discussed in reproductive module) and has been used in treatment of Cushing’s syndrome

Marc Imhotep Cray, MD


Adrenocortical insufficiency 1. Primary adrenocortical insufficiency  is most commonly caused by autoimmune destruction of adrenal cortex can be chronic (Addison disease) or causes acute adrenal crisis Characterized by following Sn & Sx:  ↓ adrenal glucocorticoid, androgen, and mineralocorticoid  ↑ ACTH (Low cortisol stimulate ACTH secretion by negative feedback)  Hypoglycemia (caused by cortisol deficiency)  Weight loss, weakness, nausea, and vomiting  Hyperpigmentation (Low cortisol stimulate ACTH secretion; ACTH contains MSH fragment)  ↓ pubic & axillary hair in FM (caused by deficiency of adrenal androgens)  ECF volume contraction (hyponatremic), hypotension, hyperkalemia, and metabolic acidosis (caused by aldosterone deficiency) Marc Imhotep Cray, MD

363


Addison Disease (Primary Adrenal Insufficiency) Symptoms, caused by reduced production of  glucocorticoids  mineralocorticoids, and  sex hormones range from vague feelings of illness to acute syncope and mental status changes

Biochemical abnormalities (eg, hyponatremia, hyperkalemia) usually exist

Marc Imhotep Cray, MD

364


Primary Adrenal Insufficiency (Addison Disease) Caused by  autoimmune-mediated destruction of adrenal cortex  mycobacterial infection  adrenal metastases  use of certain drugs

Marc Imhotep Cray, MD

Raff RB, Rawls SM, Beyzarov EP. Netter's Illustrated Pharmacology, Updated Edn. Saunders, 2014

365


Adrenocortical insufficiency cont. 2. Secondary adrenocortical insufficiency  Caused by primary deficiency of ACTH  o does not exhibit hyperpigmentation (b/c deficiency of ACTH) o does not exhibit volume contraction, hyperkalemia, or metabolic acidosis (b/c aldosterone levels normal)  Symptoms are otherwise similar to those of Addison disease 3. Tertiary adrenal insufficiency  Seen in patients with chronic exogenous steroid use precipitated by abrupt withdrawal adrenal crisis o Aldosterone synthesis unaffected Marc Imhotep Cray, MD

366


Adrenal Insufficiency cont.  life-threatening acute adrenal crisis occurs in cases of undiagnosed adrenal insufficiency & untreated stress, mimics septic shock presents with  severe anorexia  dehydration, and  Hypotension cardiovascular collapse  Treatment: IV fluids and high-dose IV glucocorticoids  Chronic disease is managed with a glucocorticoid (hydrocortisone) plus a mineralocorticoid (fludrocortisone) with dosage tailored to avoid Cushing syndrome or inadequate therapy  Patients should be monitored for fludrocortisone side effects o electrolyte changes o hypertension o edema, and o hyperglycemia Marc Imhotep Cray, MD

367


Pathophysiology of acute adrenal crisis  Can occur in patients with undiagnosed ACTH deficiency, in patients receiving corticosteroids and who are not given increased steroid dosage during periods of stress Precipitants include: o Infection o Trauma o Surgery, and o Dehydration o Gastrointestinal infections  particularly challenging because of associated inability to ingest or absorb oral hydrocortisone replacement  can lead to adrenal crisis despite other treatments  If unrecognized and untreated coma, severe hypotension, or shock unresponsive to vasopressors may rapidly lead to death Ref: Hammer GD, McPhee SJ, eds. Pathophysiology of Disease: An Introduction to Clinical Medicine, 7th Edn. New York: McGraw-Hill, Marc Imhotep Cray, MD 2014; pgs. 616-16

368


Pathophysiology of acute adrenal crisis (2)  Patients with acute adrenal crisis have symptoms of fever, weakness, apathy, and confusion  Anorexia, nausea, and vomiting may lead to volume depletion and dehydration  Abdominal pain may mimic that of an acute abdominal process  Evidence suggests that Sx of acute GC deficiency are mediated by significantly elevated plasma levels of cytokines, particularly IL-6 and, to a lesser extent, IL-1 and TNF  Hyponatremia, hyperkalemia, lymphocytosis, eosinophilia, and hypoglycemia occur frequently Ref: Hammer GD, McPhee SJ, eds. Pathophysiology of Disease: An Introduction Marc Imhotep Cray, MD to Clinical Medicine, 7th Edn. New York: McGraw-Hill, 2014; pgs. 616-16

“Hot T-bone stEAK”: IL-1: fever (hot). IL-2: stimulates T cells. IL-3: stimulates bone marrow. IL-4: stimulates IgE production. IL-5: stimulates IgA production. IL-6: stimulates aKute-phase protein production.


Hyperaldosteronism  Increased secretion of aldosterone from adrenal gland  Clinical features include hypertension, ↓ K+, metabolic alkalosis 

No edema due to aldosterone escape mechanism

 Primary hyperaldosteronism  Seen with adrenal adenoma (Conn syndrome) or idiopathic adrenal hyperplasia ↑aldosterone, ↓renin

 Secondary hyperaldosteronism  Seen in pts. with renovascular hypertension & juxtaglomerular cell tumor (due to independent activation of renin-angiotensin-aldosterone system) ↑aldosterone, ↑renin Marc Imhotep Cray, MD

370


Corticosteroid Agonists & Antagonists

Katzung & Trevors Pharmacology Examination and Board Review, 11th Ed. McGraw-Hill, 2015.

Marc Imhotep Cray, MD

371


Case 41 Answers Adrenal Cortex Summary: A 45-year-old man has Cushing disease. • Pituitary hormonal stimulus of adrenocortical steroid production: ACTH. • Primary adrenal glucocorticoid: Cortisol. • Primary adrenal mineralocorticoid: Aldosterone. • Major mineralocorticoid effects: Regulation of salt and water balance in the kidney, promote sodium retention, and potassium loss. CLINICAL CORRELATION Cushing disease is caused by ACTH-secreting tumors in the pituitary gland. The continuous production of ACTH disrupts the normal circadian production of ACTH and overrides the feedback of adrenal steroids on the hypothalamus and pituitary, resulting in excessive adrenocortical steroid production. Glucocorticoids affect most organs and tissues in the body. Marc Imhotep Cray, MD

372


Case 41 Answers Adrenal Cortex cont. Their effects are mediated by specific intracellular glucocorticoid receptors that modulate the transcription rates of specific genes and results in increases or decreases of specific proteins. The major glucocorticoid produced in the adrenal glands is cortisol (hydrocortisone). Glucocorticoids have numerous physiologic effects, including the stimulation of gluconeogenesis, increasing lipolysis, decreasing glucose uptake into fat cells, and redistributing body fat. These effects cause some of the symptoms and signs of Cushing disease, which include glucose intolerance or overt diabetes, weight gain, and increasing truncal obesity. Glucocorticoids also have anti-immune effects, which include decreasing circulating lymphocytes, monocytes, eosinophils, and basophils, increases in circulating neutrophils and atrophy of lymphoid tissue. The excess production of glucocorticoids can therefore lead to immune system suppression and recurrent infections. Marc Imhotep Cray, MD

373


Case 41 Answers Adrenal Cortex cont. Under normal physiologic conditions, adrenocortical steroids will exert a negative feedback of ACTH release from the pituitary gland. ACTH release, and subsequent cortisol production, can be suppressed even more by the administration of synthetic steroids such as dexamethasone. ACTH, which is continuously produced by a tumor, will not be suppressed by this feedback mechanism. This formulates the basis for the dexamethasone suppression test, in which a dose of dexamethasone is administered and subsequent cortisol production is measured. Normally dexamethasone administration would cause a reduction of circulating cortisol. In Cushing disease the measurement of cortisol will remain at normal, or even elevated, levels. Marc Imhotep Cray, MD

374


Lect. 6 of 6

Diabetes Mellitus (DM) The most common pancreatic disease requiring pharmacologic therapy is diabetes mellitus, a deficiency of insulin production or effect. Diabetes is treated with several parenteral formulations of insulin and oral or parenteral noninsulin antidiabetic agents.

Marc Imhotep Cray, MD

375


Learning Objectives cont. Drugs to Treat Diabetes 1. The fundamental differences between type 1 and type 2 diabetes 2. Recognition of diagnostic criteria and therapeutic goals for treatment of diabetes. 3. The pharmacological differences between the various insulin formulations used in the treatment of diabetes especially their duration of action. Specifically, which insulin types are used for the control of post-prandial glucose levels versus those used for the control of fasting glucose levels. 4. The use and clinical benefits of an intensive insulin therapy regimen in the treatment of type 1 diabetes 5. The important role of diet and exercise in the treatment of type-2 diabetes

Marc Imhotep Cray, MD

376


Learning Objectives cont. Drugs to Treat Diabetes cont. 6. The indications, mechanism of action, clinical effects, adverse effects and contraindications of drugs commonly used in Tx of type 2 diabetes including: a) Biguanides-metformin-reduces hepatic glucose production‌ and more b) Sulfonylureas-stimulating insulin secretion c) Meglitinides-stimulating insulin secretion d) Thiazolidinediones-increasing insulin sensitivity in peripheral tissues e) ι-glucosidase inhibitors-decreases amount of dietary CHOs absorbed f) Modulators of incretin -increases glucose-dependent insulin secretion g) Insulin h) Inhibition of glucagon-pramlintide 7. An understanding of which drugs used for treatment of type-2 diabetes primarily affects either post-prandial or fasting glucose levels Marc Imhotep Cray, MD

377


Learning Objectives cont. Drugs to Treat Diabetes cont. 8.The concept that effective treatment of type-2 diabetes will likely require combination therapy with one or more oral anti-diabetic agents, as well as potentially the use of insulin therapy. 9. An understanding of the current treatment algorithm approved by the American Diabetes Association for the treatment of type-2 diabetes. 10. The effectiveness of tight glycemic control in the prevention of the macroand microvascular complications of diabetes

Marc Imhotep Cray, MD

378


Drugs Used in the Treatment of DM Insulins Rapid-acting Insulin aspart Insulin glulisine Insulin inhalation (limited availability) Insulin lispro Short-acting Regular insulin Intermediate-acting Neutral protamine Hagedorn (NPH) insulin Combination insulin Insulin aspart protamine and insulin aspart Insulin lispro protamine and insulin lispro Insulin NPH and insulin regular Long-acting Insulin glargine Insulin detemir Marc Imhotep Cray, MD

Oral Hypoglycemics

Oral Agents cont.

Sulfonylureas First generation Chlorpropamide Tolazamide Tolbutamide Second generation Gliclazide Glimepiride Glipizide Glyburide Meglitinides Nateglinide Repaglinide Biguanide Metformin

Îą-Glucosidase inhibitors Acarbose Miglitol Thiazolidinediones Pioglitazone Rosiglitazone Amylinomimetic Pramlintide Incretin-based Exenatide Sitagliptin

379


Endocrine pancreas cell types  Islets of Langerhans are collections of α, β, and δ endocrine cells Islets arise from pancreatic buds  α = glucagon (peripheral)  β = insulin (central)  δ = somatostatin (interspersed)  Preproinsulin (synthesized in RER)  cleavage of “presignal ”to proinsulin (stored in secretory granules)  cleavage of proinsulin  exocytosis of insulin and C- peptide equally  Insulin and C-peptide increased in insulinoma and sulfonylurea use, whereas  Exogenous insulin lacks C-peptide Marc Imhotep Cray, MD

380


Insulin and C- peptide  Insulin is secreted together with inactive C-peptide  C-peptide provides a useful index of insulin secretion:  

plasma concentration is low or absent in patients with T1DM very high in patients with insulinoma (an uncommon tumor which causes hypoglycemia by secreting insulin)

 C-peptide concentration is not elevated in patients with hypoglycemia caused by injection of insulin

Marc Imhotep Cray, MD

NB: C-peptide should not be confused with “C-reactive peptide” (CRP) which is an acute phase protein synthesized by liver and used as a nonspecific index of inflammation.


Human proinsulin and its conversion to insulin

Marc Imhotep Cray, MD

Brunton LL & Parker KL. Goodman and Gilman Manual of Pharmacology and Therapeutics. New York: McGraw-Hill, 2008


Case 43 Pancreas and Glucose Homeostasis A 12-year-old boy is brought to the office by his parents because of abdominal pain for the past day. Prior to this, the parents noted that he was drinking a lot of water and going to the bathroom frequently. He said that his mouth was very dry and he was very thirsty. Until the past day or two he was eating more than usual but was losing weight. He has no significant medical history, and the family history is unremarkable. On examination, he appears moderately ill, and his blood pressure is normal, but he is tachycardic. His mucous membranes are dry. His abdomen is diffusely tender but without rebound or guarding. A urine dipstick test in the office reveals the presence of large ketones and glucose. A glucose measurement from a drop of blood obtained by finger stick is markedly elevated at 550 mg/dL. You immediately admit the patient to the hospital for newly diagnosed type I diabetes mellitus in ketoacidosis and start an infusion of IV fluids and regular insulin. _ What is the structure of natural human insulin? _ What effect does insulin have on potassium? _ What is the effect of ι-adrenergic stimulation on insulin secretion? _ What is the effect of β-adrenergic stimulation on insulin secretion? Marc Imhotep Cray, MD

383


Control of Blood Glucose  Glucose occupies a central position in metabolism as predominant substrate for energy production  Cells receive their supply of glucose from blood, and control mechanisms ensure that blood glucose concentration remains within narrow limits  Glucose enters blood by absorption from gut and from breakdown of stored glycogen (in muscle & liver) or gluconeogenesis (in liver)  At physiological concentrations , glucose is transferred from blood into cells almost entirely by facilitated diffusion  In most tissues this transfer is dependent on action of polypeptide hormone insulin Marc Imhotep Cray, MD

384


Control of Blood Glucose cont.  Glucose is primary stimulus of insulin secretion Mechanism of Glu-driven insulin secretion: 1. Glucose entry into β cells is facilitated by GLUT-2 transporter 2. glucose in β cell is phosphorylated to glucose-6-phosphate by glucokinase 3. Glucose-6-phosphate is metabolized by β cells,  Increasing intracellular ATP/ADP ratio and  closing an ATP-sensitive K+ channel

4. results in depolarization of β-cell membrane which opens voltage-gated Ca2+ channels Increased intracellular Ca2+ levels activate exocytosis of insulin containing secretory vesicles Marc Imhotep Cray, MD


Regulation of insulin secretion from β cells by  Nutrients (glucose, amino acids, FFAs) and  Hormones/NTs o glucagon-like peptide-1 (GLP1) o epinephrine, NE o acetylcholine (ACh)

Marc Imhotep Cray, MD

White BA & Porterfield SP. Endocrine and Reproductive Physiology, 4th ed. (Mosby physiology monograph series). Mosby, 2013.


Control of Blood Glucose cont. Presence of nutrients in small intestine stimulates release from gut endocrine cells of peptide hormones called incretins which promote insulin secretion Principal incretins:  glucose-dependent insulinotropic peptide (GIP) secreted by upper gut,  glucagon-like peptide-1 (GLP-1), released from distal gut

Marc Imhotep Cray, MD

387


Control of Blood Glucose cont.  GLP-1 has several actions that regulate glucose homeostasis:     

enhanced glucose-dependent insulin secretion incretins are responsible for about 60% of insulin that is secreted in response to a meal inhibition of glucagon release via GPCR binding inhibits acid secretion and gastric emptying, promotes insulin sensitivity promotion of satiety by action on hypothalamus increases beta cells mass and insulin expression, PTM (posttranslational modification), and secretion

 Actions of GLP-1 are brief  very short plasma half-life of 1–2 min due to rapid degradation by dipeptidyl peptidase-4 (DPP-4) 

Class of Oral hypoglycemic=Incretin mimetics o DDP-IV Inhibitors block digestion of incretins

Marc Imhotep Cray, MD


Control of insulin release from pancreatic islets of Langerhans β-cells

Modified from: Waller DG. Medical Pharmacology and Therapeutics, 4E. Saunders, 2014 Marc Imhotep Cray, MD

389


Main processes determining blood glucose concentrations

Dale MM & Haylett DG. Rang & Dales’ ‘Pharmacology Flashcards, Updated Ed. Churchill Livingstone, 2014.

Marc Imhotep Cray, MD

390


Insulin Dale MM & Haylett DG. Rang & Dales’ ‘Pharmacology Flashcards, Updated Ed. Churchill Livingstone, 2014.

Actions: Promotes tissue uptake and storage of glucose, amino acids and fats. Acutely lowers blood glucose. Inhibits hepatic glycogenolysis and gluconeogenesis. Increases glycogen synthesis in muscle/liver. Inhibits lipolysis. Stimulates protein synthesis. Longer-term effect on growth and gene expression. Marc Imhotep Cray, MD

391


Pancreas and Insulin Production  Pancreas is principal organ involved in production and secretion of hormones that maintain normal blood glucose levels (or euglycemia)  Pancreatic β cells of islets of Langerhans produce, store, and secrete insulin  As discussed earlier, pancreas first produces a parent protein called preproinsulin which is then cleaved to form the smaller compound proinsulin Proinsulin is then cleaved to form insulin and peptide C Marc Imhotep Cray, MD

392


Pancreas and Insulin Production cont.  Pancreas also produces  Glucagon (α), a hormone that increases blood glucose levels, and  Somatostatin (δ cells), a hormone that inhibits both insulin and glucagon secretion o physiologic role in pancreatic islet function remains unclear  Ingestion of carbohydrates prompts an increase in release of insulin and concomitant decrease in plasma glucagon levels  Glucagon is released in response to low blood glucose levels and protein ingestion 

It stimulates insulin secretion which in turn inhibits glucagon release in a negative feedback loop

Marc Imhotep Cray, MD

393


Insulin Secretion Insulin secretion is a highly regulated process that varies throughout day

In a postprandial setting (after a meal), a burst of insulin secretion normally occurs in response to a transient increase in plasma glucose level In a post-absorptive period, pancreas reduces insulin secretion, which maintains low basal levels of circulating insulin

Insulin is key to body’s use of glucose it promotes  uptake of glucose, fatty acids, and amino acids, and  it facilitates their conversion to forms used for storage in most tissues Marc Imhotep Cray, MD

394


Biphasic response of insulin secretion to Glu infusion  Serum insulin levels normally begin to rise within 10 minutes after food ingestion and reach a peak in 30 to 45 minutes  Higher serum insulin level rapidly lowers blood glucose to baseline values  When insulin secretion is stimulated, insulin is released rapidly (within minutes)  called early phase of insulin secretion  If stimulus is maintained, insulin secretion falls within 10 minutes , then slowly rises over a period of 1 hour  Second phase is called late phase of insulin release Marc Imhotep Cray, MD

White BA & Porterfield SP. Endocrine and Reproductive Physiology, 4th ed. (Mosby physiology monograph series). Mosby, 2013.


Insulin Secretion cont. Cellular Actions of Insulin Key insulin target tissues for regulation of glucose homeostasis are liver, muscle, and fat:  liver, where glycogen (main carbohydrate reserve, which is easily converted to glucose) is synthesized, stored, and broken down  skeletal muscle, where glucose oxidation produces energy  adipose tissue, where glucose is converted to fatty acids, glycerol phosphate, and triglycerides

Marc Imhotep Cray, MD

396


Insulin Secretion

Marc Imhotep Cray, MD

Raff RB, Rawls SM, Beyzarov EP. Netter's Illustrated Pharmacology, Updated Edn. Saunders, 2014

397


Insulin Secretion cont.  Increase in oxidizable substrates (e.g., glucose and G-6-P) enhances ATP production, thereby inhibiting an ATP-sensitive K+ channel  Decrease in K+ conductance causes Em to rise, opening a voltage-sensitive Ca2+ channel; Ca2+ then acts as the insulin secretagogue  ATP-sensitive K+ channel in β cells is an octamer composed of four Kir 6.2 and four SUR1 subunits  both subunits contain nucleotide-binding domains  Kir 6.2 mediates inhibitory response to ATP  SUR1 binds ADP, the channel activator diazoxide, and the channel inhibitors (and promoters of insulin secretion) sulfonylureas and meglitinide

Wecker L. etal. , Brody’s Human Pharmacology: Molecular to Clinical, 5th Ed. Philadelphia: Mosby, 2010.


Actions of Insulin  Insulin is considered a “fuel storageâ€? hormone o promotes storage of glucose (as glycogen) and fatty acids (as triglycerides in adipose tissue o stimulates uptake of glucose into cells via glucose transporter 4 (GLUT4) transporters, and glucose is used or stored as glycogen o major glycogen stores are in muscle and liver o Insulin also stimulates fat synthesis and inhibits lipolysis in adipose tissue, which maintains stores of triglycerides and reduces keto acid production o Lastly, insulin stimulates uptake of amino acids into skeletal muscle and storage as protein o The overall result is that insulin decreases plasma glucose, fatty acids, and keto acids NB: Unlike glucose, insulin does not cross placenta. Marc Imhotep Cray, MD

Mulroney SE & Myers AK. Netter's Essential Physiology 2nd Ed. Philadelphia: Elsevier, 2016.

399


Actions of Glucagon

In opposition to insulin, glucagon promotes use of cellular energy stores through release of glucose into blood

 Glucagon is considered the “fuel mobilizationâ€? hormone because o it breaks down glycogen, protein, and lipids, thus releasing glucose, amino acids, fatty acids, and keto acids into blood to serve metabolic demand o Glucagon is stimulated by low blood glucose levels and promotes glycogenolysis and gluconeogenesis in liver, thus increasing blood glucose levels o Glucagon also stimulates lipolysis and release of fatty acids from adipose tissue, which are oxidized to keto acids in liver o In muscle, glucagon inhibits protein synthesis, thus providing amino acids for conversion to glucose via gluconeogenesis in liver

Marc Imhotep Cray, MD

Mulroney SE & Myers AK. Netter's Essential Physiology 2nd Ed. Philadelphia: Elsevier, 2016.

400


Actions of Glucagon (2)  In cardiac tissue glucagon binds a glucagon receptor and raises cAMP resulting in a positive inotropic (contractility) and chronotropic (heart rate) effect on heart  MOA by which glucagon is therapeutic in β-blocker and calcium channel blocker overdoses  importantly, its action is independent of β receptors or calcium channels Marc Imhotep Cray, MD

Bardal SK et.al. Applied Pharmacology. St. Louis: Saunders, 2011.


Plasma glucose and insulin levels after an oral aqueous glucose (75 gm) challenge after an 8-hour fast. Subjects were grouped based on a diagnosis of normal, (A); type 1 diabetes mellitus, (B); type 2 diabetes mellitus, (C); and impaired glucose tolerance, (D). (next 4 slides describe)

Marc Imhotep Cray, MD

Wecker L, et al. Brody’s Human Pharmacology: Molecular to Clinical, 5th Ed. Philadelphia, PA: Mosby, 2010

402


Plasma Glu and insulin levels after an oral aqueous glucose (75 gm) challenge after an 8-hour fast (2) (A) Nondiabetic individuals display a peak glucose concentration of 10 to 12 mM by 30 minutes and normoglycemia (5 mM) in 90 to 120 minutes; the insulin concentration follows a similar response Wecker L, et al. Brody’s Human Pharmacology: Molecular to Clinical, 5th Ed. Philadelphia, PA: Mosby, 2010 Marc Imhotep Cray, MD


Plasma Glu and insulin levels after an oral aqueous glucose (75 gm) challenge after an 8-hour fast (3) (B) Type 1 Diabetes Mellitus at all time points, patients with T1DM exhibit greatly elevated plasma glucose levels compared with normal, and plasma insulin levels were essentially undetectable Wecker L, et al. Brody’s Human Pharmacology: Molecular to Clinical, 5th Ed. Philadelphia, PA: Mosby, 2010

Note: In clinical practice patients with suspected type 1 diabetes mellitus are not usually subjected to oral glucose tolerance tests. Diagnosis of insulinopenia, which is accompanied by elevated blood glucose levels, is Marc Imhotep Cray, MD adequate information to initiate insulin treatment.


Plasma Glu and insulin levels after an oral aqueous glucose (75 gm) challenge after an 8-hour fast (4) (C) Type 2 Diabetes Mellitus hyperglycemia at all time points is also characteristic of T2DM. However, plasma insulin levels exhibit an extended delay, although nearly normal levels can be observed. In type 2 diabetes mellitus with insulin resistance, plasma glucose concentrations can remain elevated in spite of nearly normal levels of insulin. Marc Imhotep Cray, MD


Plasma Glu and insulin levels after an oral aqueous glucose (75 gm) challenge after an 8-hour fast (5) For pts with impaired glucose tolerance, Bld Glu at 30 to 60 minutes is higher than normal and may remain elevated at 120 minutes. Because of β cell compensation for impaired glucose, plasma insulin conc. are higher than normal, both in fasting state and after glucose is administered. For many pts this is a temporary situation, and depending on cause, outcome can return to normal, remain unchanged, or mimic T2DM Marc Imhotep Cray, MD


Question A 23-year-old healthy man receives an infusion of 100 mL of glucose solution. Which of the following curves would represent his plasma insulin concentration at 2 min after infusion?

(A) Letter A (B) Letter B (C) Letter C (D) Cannot be determined

Marc Imhotep Cray, MD

407


Glucose transporters  Insulin-dependent glucose transporters:  GLUT-4: adipose tissue, striated muscle (exercise can also increase GLUT-4 expression)  Insulin-independent transporters:  GLUT-1: RBCs, brain, cornea, placenta  GLUT-2 (bidirectional): β islet cells, liver, kidney, small intestine  GLUT-3: brain, placenta  GLUT-5 (fructose): spermatocytes, GI tract  Brain utilizes glucose for metabolism normally and ketone bodies during starvation  RBCs always utilize glucose because they lack mitochondria for aerobic metabolism BRICK L (insulin-independent glucose uptake): Brain, RBCs, Intestine, Cornea, Kidney, Liver

Marc Imhotep Cray, MD

408


Insulin recruitment of glucose transporters (molecular-cellular MOA) 1. Insulin binds to its receptor in cell membrane. 2. Activated receptor promotes recruitment of glucose transporters from intracellular pool to cell membrane. 3. Glucose transporters increase insulin-mediated uptake of glucose into cell. 4 When insulin levels decrease, glucose transporters move from cell membrane to intracellular storage pool, where they can be recycled. 5. Vesicles fuse to form an organelle called the endosome. McInnis M., Mehta S. Step-up to USMLE Step 1 2015 Edition. Wolters Kluwer, 2015 409


Insulin MOA (molecular-cellular level) See: The Insulin Receptor: A Brief Tutorial

Marc Imhotep Cray, MD

Brunton L, Chabner B, Knollman B, eds. Goodman & Gilman’s The Pharmacological Basis of Therapeutics, 12th ed. McGraw-Hill, 2011; Pg. 1242, Figure 43–4.

410


Insulin MOA (tissue-organ level)  Insulin controls uptake, use, and storage of cellular nutrients:  promote uptake of glucose by cells • does this by mobilizing glucose transporters (GLUT-4) on surface of muscle and adipose tissue  In liver and in muscle, insulin leads to increased glycogen deposition by increasing its synthesis and inhibiting its degradation  In liver, insulin inhibits synthesis of glucose (gluconeogenesis)  In muscle, insulin facilitates uptake of amino acids, promoting protein synthesis  In adipose tissue, insulin promotes synthesis of triglycerides and inhibits lipolysis

Bardal SK et.al. Applied Pharmacology. St. Louis: Saunders, 2011. 411


Lack of lnsulin  Without insulin, glucose is not transported across cell membranes leads to a cascade of metabolic events

 Body reacts by inducing gluconeogenesis (liver converts glycogen to glucose)  To produce energy skeletal muscle converts its structural proteins to amino acids which are carried to liver where they are converted to glucose  Resultant excess glucose, still not being used by cells, leads to hyperglycemia  Insulin deficiency increases fat catabolism: free fatty acids are broken down into keto acids to increase energy sources Marc Imhotep Cray, MD

412


Lack of lnsulin cont.  Kidneys eliminate keto acids produces ketonuria and ketonemia  Keto acids also reduce blood Ph can result in ketoacidoses, coma, and death  Diabetes is caused by a relative or absolute lack of insulin with hyperglycemia being hallmark medical finding  Once thought of as 1 disease, diabetes is now believed to be a chronic heterogeneous group of disorders that result from pathologic processes that depend on diabetes type Marc Imhotep Cray, MD

413


Lack of Insulin

Marc Imhotep Cray, MD

Raff RB, Rawls SM, Beyzarov EP. Netter's Illustrated Pharmacology, Updated Edn. Saunders, 2014

414


Diabetes Mellitus Diabetes mellitus is inability to properly manage glucose metabolism (glucose intolerance) because of either  inability to produce and secrete insulin as a result of autoimmune destruction of pancreatic β-cells (type 1) or  inability of peripheral tissues to respond to circulating insulin as a result of diminished or impaired insulin receptor signaling (type 2)  Negative consequences of DM occur because of elevated blood sugar levels  Short-term diabetic ketoacidosis (DKA) (type 1), hyperosmolar nonketotic syndrome (HNKA) ,(type 2)  Long-term results in microvascular and macrovascular complications

Pharmacotherapy of diabetes, therefore, relies on keeping sugar levels within the normal range 415


Type 1 Diabetes Mellitus  In type 1 DM, insulin-producing β cells of pancreas are destroyed by either intrinsic genetic factors or extrinsic factors such as viruses or chemical toxins  In one theory that involves an autoimmune-mediated mechanism, predisposed patients react abnormally to environmental triggers by producing antibodies that are directed against β cells  Insulin secretion is impaired early in disease and eventually stops  Type 1 DM usually develops abruptly during childhood or adolescence and usually presents with polydipsia, polyuria, and polyphagia

 Ketoacidosis is more likely to occur in type 1 DM than in type 2 DM  Signs (Sn) and Symptoms (Sx) of diabetic ketoacidosis: o Kussmaul respirations; fruity breath; abdominal pain; nausea; vomiting; polyuria; polydipsia; dehydration; fatigue Marc Imhotep Cray, MD

416


Type 1 Diabetes Mellitus cont.  Patients require lifelong treatment with exogenous insulin to control bld glucose levels and prevent short-term and longterm macrovascular and microvascular complications, such as  nephropathy  neuropathy  retinopathy, and  cardiovascular disease (CAD, PVD, Stroke)

Oral hypoglycemic agents are ineffective in patients with type 1 DM because functioning β cells are required Marc Imhotep Cray, MD

417


Type 2 Diabetes Mellitus  Central defects in type 2 DM are decreased insulin secretion and insulin resistance  Before diabetes is diagnosed, patients, often obese have hyperinsulinemia caused by excess dietary carbohydrates  Pancreas malfunctions and fails to supply high insulin demands also, impaired secretion is complicated by insulin resistance:  insulin resistance means insulin cannot decrease plasma glucose levels through suppression of hepatic glucose production and stimulation of glucose use in skeletal muscle and adipose tissue  Resistance develops in several possible ways, eg, o chronic hyperinsulinemia causes insulin receptor downregulation leads to defects in insulin binding and post-receptor insulin signaling pathways

Marc Imhotep Cray, MD

418


Type 2 Diabetes Mellitus cont.  Unlike type 1 DM, type 2 DM  has a more gradual onset  may not present with classic symptoms, and  usually occurs in overweight patients older than 35 years  Oral hypoglycemic agents  decrease plasma glucose levels  improve insulin resistance, and  reduce long-term complications  Many patients need insulin therapy as well Marc Imhotep Cray, MD

419


Type 2 Diabetes Mellitus

Marc Imhotep Cray, MD

Raff RB, Rawls SM, Beyzarov EP. Netter's Illustrated Pharmacology, Updated Edn. Saunders, 2014

420


Oral Agents for Type 2 Diabetes Patients with type 2 diabetes are managed pharmacologically by following actions:  Increasing pancreatic insulin secretion with sulfonylureas or meglitinides (known as insulin secretogogues)  Decreasing hepatic gluconeogenesis with biguanides

 Improving insulin sensitivity in peripheral tissues with thiazolidinediones  Preventing breakdown of complex carbohydrates into simple sugars with α-glucosidase inhibitors Marc Imhotep Cray, MD

421


DM Acute Manifestations and Diagnosis Acute Manifestations Polydipsia, polyuria, polyphagia, weight loss, DKA (type 1), hyperosmolar nonketotic syndrome (HNKA,(type 2) Rarely, can be caused by unopposed secretion of GH and epinephrine  Also seen in pts on glucocorticoid therapy (steroid diabetes) Diagnosis TEST HbA1c

DIAGNOSTIC CUTOFF ≥ 6.5%

FPG

≥ 126 mg/dL

2-hour OGTT

≥ 200 mg/dL

Marc Imhotep Cray, MD

NOTES Reflects average blood glucose over prior 3 months

Fasting for > 8 hours 2 hours after consumption of 75 g of glucose in water 422


DM Chronic Complications Nonenzymatic glycation:  Small vessel disease (diffuse thickening of basement membrane) retinopathy (hemorrhage, exudates, microaneurysms, vessel proliferation), glaucoma, neuropathy, nephropathy (nodular glomerulosclerosis, aka Kimmelstiel-Wilson nodules progressive proteinuria [initially microalbuminuria; ACE inhibitors are renoprotective] and arteriolosclerosis hypertension; both lead to chronic renal failure).  Large vessel atherosclerosis: CAD, peripheral vascular occlusive disease, gangrene limb loss, cerebrovascular disease MI most common cause of death  Osmotic damage (sorbitol accumulation in organs with aldose reductase and decrease or absent sorbitol dehydrogenase):  Neuropathy (motor, sensory [glove and stocking distribution], and autonomic degeneration impotency)  Cataracts Marc Imhotep Cray, MD

423


DM Microvascular and Macrovascular Complications

Marc Imhotep Cray, MD

Raff RB, Rawls SM, Beyzarov EP. Netter's Illustrated Pharmacology, Updated Edn. Saunders, 2014

424


Insulin Therapy  Insulin is sole therapy for type 1 DM  Currently used insulin preparations are all human insulin produced by recombinant deoxyribonucleic acid (DNA) techniques  There are rapid, short, intermediate, and long-acting insulin preparations available

 MOA Exogenous insulin stimulates carbohydrate metabolism and helps with transfer of glucose into cardiac and skeletal muscle and adipose tissue 

Insulin also:  aids in conversion of glucose to glycogen  stimulates lipogenesis and protein synthesis, and  reduces serum potassium and magnesium levels

Marc Imhotep Cray, MD

Note: Insulin is also used (combination therapy or monotherapy) in type 2 DM poorly controlled with diet and oral agents.

.425


Insulin Therapy cont.  Insulin, a protein, is degraded in GI system if used orally so it is given subcutaneously, or, in emergencies, intravenously  Absorption of an insulin product may vary in a patient from one injection to next, absorption being affected by site of injection, temperature, physical activity, and dose  Insulin preparations differ in dose, onset, duration, and sources of origin, including  biosynthetic and semisynthetic human (therapeutically equal)  human insulin (least antigenic and most soluble), and  beef and pork (replaced by human due to not infrequent antigenic rxns) Marc Imhotep Cray, MD

426


Commonly Used Insulin Preparations Insulin Preparation

Onset of Activity

Time of Peak Activity

Duration of Action

Insulin lispro

5–15 min

1–2 h

3–4 h

Insulin aspart

5–15 min

1–2 h

3–4 h

Insulin glulisine

5–15 min

1–2 h

3–4 h

Regular insulin

30 min

2–4 h

6–8 h

NPH (neutral protamine Hagedorn) insulin

1–2 h

4–12 h

18–24 h

Insulin detemir

3–4 h

3–9 h

6–24 h

Insulin glargine

3–4 h

No peak

24 h or longer

Marc Imhotep Cray, MD

427


Onset and duration of action of human insulin and insulin analogs

Whalen K, Finkel R & Panavelil TA. Lippincott Illustrated Reviews-Pharmacology 6th Ed. Philadelphia, PA: Wolters Kluwer, 2015

428


Examples of three regimens that provide both prandial and basal insulin replacement

B = breakfast, L = lunch , S = supper Whalen K, Finkel R & Panavelil TA. Lippincott Illustrated ReviewsPharmacology 6th Ed. Philadelphia, PA: Wolters Kluwer, 2015 Marc Imhotep Cray, MD

429


Reactions to Insulin: Hypoglycemia and Adipose Tissue Changes  Hypoglycemia is most common and serious adverse reaction to insulin predisposing factors include  inadequate food intake  poor timing of injections  exercise, and  use of hypoglycemic drugs  Symptoms of hypoglycemia are autonomic (eg, sweating, trembling, feeling of warmth) or neuroglycopenic (eg, confusion, weakness, drowsiness)  Hunger, tachycardia, blurred vision, and loss of consciousness also occur  Elderly patients with neuropathy, patients with long standing diabetes (>10 years), and patients taking β blockers can have blunted symptoms Marc Imhotep Cray, MD

430


Acute Hypoglycemia

Marc Imhotep Cray, MD

Raff RB, Rawls SM, Beyzarov EP. Netter's Illustrated Pharmacology, Updated Edn. Saunders, 2014

431


Reactions to Insulin: Hypoglycemia and Adipose Tissue Changes cont. Antidote Use of sugar packets, candy, or pure glucose products can help with hypoglycemia  Unconscious patients must be injected with glucagon or IV glucose or dextrose Adipose Tissue Changes  Insulin injection may also cause lipohypertrophy, which occurs in patients who use only 1 site rather than rotating sites  Rotating sites solves problem

 Lipoatrophy, an immunologic reaction to insulin, is treated by Marcchanging Imhotep Cray, MD to human insulin and injecting it into affected area

432


Standard treatment versus Intensive Tx Standard insulin therapy involves twice-daily injections Intensive treatment utilizes three or more injections daily with frequent monitoring of blood glucose levels ADA recommends a target mean blood glucose level of 154 mg/dL or less (HbA1c ≤ 7%) intensive treatment is more likely to achieve this goal 

Normal mean blood glucose is approx. 115 mg/dL or less (HbA1c < 5.7%)

NB: Glycated HbA, also known as HbA 1c: In diabetics, elevated glucose leads to increased glycation of HbA within red blood cells. Because red blood cells circulate for 120 days, measurement of HbA 1c in diabetic patients serves as an index of glycemic control over the preceding months. Marc Imhotep Cray, MD

433


Standard treatment versus Intensive Tx cont.  Frequency of hypoglycemic episodes, coma, and seizures is higher with intensive insulin regimens However,  Patients on intensive therapy show a significant reduction in microvascular complications of diabetes such as retinopathy, nephropathy, and neuropathy compared to patients receiving standard care  Intensive therapy should not be recommended for patients with longstanding diabetes, significant microvascular complications, advanced age, and those with hypoglycemic unawareness  Intensive therapy has not been shown to significantly reduce macrovascular complications of diabetes Marc Imhotep Cray, MD

434


Standard treatment versus Intensive Tx cont.

Whalen K, Finkel R & Panavelil TA. Lippincott Illustrated Reviews-Pharmacology 6th Ed. Philadelphia, PA: Wolters Kluwer, 2015

435


Oral Hypoglycemic Agents Pharmacology Sulfonylureas First generation Chlorpropamide Tolazamide Tolbutamide Second generation Gliclazide Glimepiride Glipizide Glyburide

Biguanides Metformin

α-Glucosidase inhibitors Acarbose Miglitol

Thiazolidinediones (Glitazones) Pioglitazone Rosiglitazone

Incretin-based

Meglitinides

Exenatide (Incretin mimetics) Sitagliptin (DPP-IV inhibitors/“Gliptins”)

Nateglinide Repaglinide

Amylinomimetic

Marc Imhotep Cray, MD

Pramlintide 436


Sulfonylureas (Secretagogues)  Sulfonylureas, historical mainstay of therapy in type 2 DM, used as monotherapy or with insulin or other oral agents  MOA act mainly by stimulating insulin secretion from pancreatic β cells, enhancing β-cell sensitivity to glucose, and reducing glucagon release  They work only if β cells are functioning  Older drugs (eg, chlorpropamide, tolbutamide) have been replaced by new agents (eg, glimepiride, glipizide, glyburide), with  greater potency  fewer drug interactions, and  better pharmacokinetic profiles Marc Imhotep Cray, MD

437


Sulfonylureas cont.  If glucose control fails with long-term sulfonylurea use, other agents added instead of increasing sulfonylurea doses  Sulfonylureas are best for  patients diagnosed after age of 40 years or  when disease duration is less than 5 years  body weight is nearly ideal, and  fasting glucose levels are less than 180 mg/dL  Main adverse effects are hypoglycemia and weight gain Others  GI-related effects  allergic reactions  hepatotoxicity  hypothyroidism, and  disulfiram reaction (chlorpropamide) Marc Imhotep Cray, MD

438


Sulfonylureas MOA

Marc Imhotep Cray, MD

Bardal SK et.al. Applied Pharmacology. St. Louis: Saunders, 2011.

439


Drugs that reduce and potentiate effects of sulfonylureas Drugs that may reduce effects of sulfonylureas, leading to loss of glucose control: • Atypical antipsychotics • Corticosteroids • Diuretics • Niacin • Phenothiazines • Sympathomimetics

Marc Imhotep Cray, MD

Drugs that may potentiate effects of sulfonylureas, leading to hypoglycemia: • Azole antifungals • Beta-blockers • Chloramphenicol • Clarithromycin • Monoamine oxidase inhibitors • Probenecid • Salicylates • Sulfonamides

440


Meglitinides or Glinides (Secretagogues)  Meglitinides (repaglinide and nateglinide) are approved as monotherapy or in combination with metformin or TZDs in patients with type 2 DM  MOA Similar to sulfonylureas, meglitinides cause an increase in insulin secretion from pancreatic β cells  Unlike sulfonylureas, meglitinides have a rapid onset and a shorter duration necessitates dosing within 30 minutes of each meal  Especially useful for patients who have difficulty controlling postprandial hyperglycemia  ideal for pts with postprandial hyperglycemia taken just before meals to reduce postprandial hyperglycemia o if a meal is skipped, the drugs should be skipped as well Marc Imhotep Cray, MD

441


Meglitinides cont. Efficacy of meglitinides in producing reductions in glycosylated hemoglobin concentration (HbA1c) and fasting plasma glucose (FPG) level is comparable to that of sulfonylureas and metformin (reduces HbA1c by 1.5-2% and FPG level by 50-70 mg/dL) Adverse effects include mild hypoglycemia (particularly if administration is not followed with food) and weight gain TEST HbA1c

DIAGNOSTIC CUTOFF ≼ 6.5%

Marc Imhotep Cray, MD

NOTES Reflects average blood glucose over prior 3 months

442


Meglitinides MOA

Marc Imhotep Cray, MD

Bardal SK et.al. Applied Pharmacology. St. Louis: Saunders, 2011.

443


Sulfonylureas vs Meglitinides  The main difference between sulfonylureas and meglitinides is that meglitinides have a rapid onset of action as well as a short duration of action  This makes meglitinides ideal for patients with postprandial hyperglycemia thus, these drugs are taken just before meals to reduce postprandial hyperglycemia o Note: if a meal is skipped, drug should be skipped as well

Marc Imhotep Cray, MD

444


Biguanides Metformin, only biguanide available in U.S., is used as  initial monotherapy or  with insulin or other oral drugs in patients with type 2 DM who have secondary failure to sulfonylurea monotherapy (initial response but then failed glucose control with longterm use) MOA Metformin decreases blood glucose levels by reducing hepatic glucose production and glycogen metabolism and improving insulin resistance via enhancing insulin-mediated glucose uptake Marc Imhotep Cray, MD

445


Biguanides MOA

Marc Imhotep Cray, MD

Bardal SK et.al. Applied Pharmacology. St. Louis: Saunders, 2011.

446


Biguanides cont.  Metformin decreases triglyceride and total cholesterol levels, increases HDL levels, and causes weight loss and is ideal for overweight hyperlipidemic patients  Hypoglycemia occurs only when metformin is used with insulin or hypoglycemic drugs  Adverse effects  GI related  of greatest concern, rare lactic acidosis, caused by inhibited conversion of lactate to glucose and greater lactate production o mostly affects patients with renal, hepatic, or cardiovascular disorders Marc Imhotep Cray, MD

447


α-Glucosidase Inhibitors  α-Glucosidase inhibitors (acarbose, miglitol) can be used singly or with insulin or other oral drugs for type 2 DM  MOA inhibit glucosidases in small intestine brush border that break down (hydrolyze) complex polysaccharides and sucrose into absorbable monosaccharides  Rate of carbohydrate digestion and glucose absorption is thus delayed leads to lower postprandial glucose spikes (by 25-50 mg/dL)  These drugs work best in patients with postprandial hyperglycemia and when taken with a meal containing complex carbohydrates

Marc Imhotep Cray, MD

448


α-Glucosidase Inhibitors cont. α-Glucosidase Inhibitors decrease FPG slightly (20-30 mg/dL) and HbA1c levels by 0.5% to 1.0% Adverse effects are GI related (flatulence, diarrhea, abdominal pain) which result from fermentation of unabsorbed carbohydrates in small intestine and are lessened by slow dose titration

 Used with insulin or other oral drugs, they can cause hypoglycemia Hepatic transaminase levels can increase (acarbose), so LFT results must be watched Marc Imhotep Cray, MD

449


Îą-Glucosidase Inhibitors MOA

Bardal SK et.al. Applied Pharmacology. St. Louis: Saunders, 2011. Marc Imhotep Cray, MD

450


Thiazolidinediones (TZDs) or Glitazones  Thiazolidinediones (rosiglitazone and pioglitazone) are a relatively new class of antihyperglycemic agents that can be used as monotherapy or in combination with insulin or other oral agents in patients with type 2 DM  MOA TZDs reduce hyperglycemia and hyperinsulinemia by decreasing insulin resistance (via enhancement of insulin-mediated glucose uptake) at peripheral sites and in liver results in increased insulin-dependent glucose disposal and decreased hepatic glucose output  These effects are accomplished by selective binding at peroxisome PPAR-γ receptor (PPARG*) which is found in adipose tissue, skeletal muscle, and liver  Receptor activation modulates transcription of several insulin responsive genes that control glucose and lipid metabolism Marc Imhotep Cray, MD

451


TZD: Clinical Rationale and Adverse Effects pharmacology is based on suggestions patients with type 2 DM already have too much insulin  Liver, however, is resistant to that insulin and therefore continues to produce large amounts of glucose  Instead of stimulating pancreas to produce more insulin, sensitivity to existing insulin should be increased to slow hepatic glucose production

 TZD effects on HbA1c and FPG fall between those of acarbose and sulfonylureas and metformin  TZDs plus insulin enhance glycemic control and decrease insulin needs Marc Imhotep Cray, MD

452


TZD: Clinical Rationale & Adverse Effects cont.  TZDs also reduce triglyceride levels and increase HDL, but they also increase LDL levels  First TZD (troglitazone) was withdrawn after causing hepatotoxicity  2 drugs now used (rosiglitazone and pioglitazone) have not shown hepatotoxic effects but LFTs should be checked before and during TZD therapy  Adverse Effects TZDs cause  hematologic effects (reduced hemoglobin, hematocrit, neutrophils)  hypoglycemia (when used with other drugs), and  edema (thus should be used with care in congestive heart failure) Marc Imhotep Cray, MD

453


Glitazones MOA

Bardal SK et.al. Applied Pharmacology. St. Louis: Saunders, 2011.

Marc Imhotep Cray, MD

454


Incretin mimetics Endogenous human incretins, such as glucagon-like peptide-1 (GLP-1), are released from gut and enhance insulin secretion (insulin secretagogue) Agents Exenatide is a synthetic GLP-1 analog Liraglutide is longer acting and more resistant to metabolism MOA Incretins decrease glucagon secretion, slow gastric emptying, reduce food intake, and promote β-cell proliferation USE SC injection of incretins may improve glycemic control in patients with type 2 diabetes mellitus who have not achieved adequate glycemic control on metformin, or a combination of metformin and a sulfonylurea Adverse Effect Pancreatitis has been observed as a serious adverse effect Marc Imhotep Cray, MD

455


GLP-1, an insulin secretagogue  GLP-1 (glucagon-like peptide-1 ) possesses several physiological properties that make it (and its functional analogs) a subject of intensive investigation as a potential treatment of T2DM  Known peripheral functions of GLP-1 include:       

increases insulin secretion from pancreas in a glucose-dependent manner increases insulin-sensitivity in both alpha cells and beta cells increases beta cells mass and insulin expression, PTM (posttranslational modification), and secretion inhibits acid secretion and gastric emptying in the stomach promotes insulin sensitivity decreases glucagon secretion from pancreas via GPCR binding

Marc Imhotep Cray, MD

456


Dipeptidyl peptidase IV inhibitors “Gliptins” Agents Sitagliptin, saxagliptin, and linagliptin are members of a class of antidiabetic agents MOA  Act by inhibiting dipeptidyl peptidase 4, a serine protease  Dipeptidyl peptidase IV is normally responsible for proteolysis (“deactivation”) of incretins including GLP-1 and glucose-dependent insulinotropic peptide (aka Gastric inhibitory polypeptide /GIP )  DPP-IV inhibitors may also improve β-cell function USE Admin. P.O. as monotherapy or in combination with metformin DPP-IV inhibitors decrease fasting and postprandial plasma glucose concentrations and plasma HbA1c concentration Adverse effects most common are headache and nausea Marc Imhotep Cray, MD

457


Incretin mimetics & DPP-IV inhibitors MOA

Bardal SK et.al. Applied Pharmacology. St. Louis: Saunders, 2011. Marc Imhotep Cray, MD

458


Amylin analogs Pramlintide is a synthetic amylin analog  Amylin is a polypeptide stored and secreted by β-cells of pancreas it acts in concert with insulin to reduce blood sugar MOA  Pramlintide acts to slow gastric emptying, decrease glucagon secretion, and decreases appetite USE  Administered SC, typically with insulin 

indicated only in patients with diabetes taking insulin with meals

Adverse effects  Most common are hypoglycemia and nausea  When pramlintide is initiated, dose of mealtime insulin should be decreased by 50% to avoid a risk of severe hypoglycemia Marc Imhotep Cray, MD

459


Summary of some adverse effects observed with oral hypoglycemic agents

Modified from: Whalen K, Finkel R & Panavelil TA. Lippincott Illustrated Reviews-Pharmacology 6th Ed. Philadelphia, PA: Wolters Kluwer, 2015

460


Question A 37-year-old woman with diabetes is brought to the emergency department unresponsive by her husband. She recently started taking a new medication to control her blood sugar, but her husband could not remember the name of it. Her blood sugar is 45 mg/dL. Which of the following diabetes medications is most likely for her condition? (A) Acarbose (B) Glipizide (C) Metformin (D) Pramlintide (E) Sitagliptin Marc Imhotep Cray, MD


Answer The answer is B: Glipizide. The negative consequences of diabetes occur due to long-term elevated blood sugar levels. Pharmacotherapy of diabetes, therefore, relies on keeping sugar levels within the normal range. Because of their mechanism of action, some of these drugs can cause hypoglycemia (as in this patient’s case). Sulfonylureas such as glipizide are an example of drugs that can cause hypoglycemia. Incorrect answers rational (A) Acarbose blocks brush border glucosidases necessary for final stage of carbohydrate metabolism. It is not associated with hypoglycemia. (C) Metformin works by inhibiting gluconeogenesis and intestinal carbohydrate absorption. It is known to cause hypoglycemia but much less commonly than sulfonylureas. (D) Pramlintide is an amylin analog and works by slowing gastric emptying, impairs glucagon secretion, and suppresses the appetite. Alone, it is not known to cause hypoglycemia. (E) Sitagliptin mimics incretins, which increase insulin secretion following a meal. Sitagliptin does not increase insulin secretion when blood glucose drops below about 90 mg/dL and does not produce significantly more hypoglycemia than placebo. Marc Imhotep Cray, MD


Question A 43-year-old woman with Type-2 diabetes has been taking insulin with meals as well as metformin. Her blood glucose remains poorly controlled. Her doctor prescribes an additional drug, which is an analog of an endogenous peptide that inhibits glucagon secretion. What is the most likely medication this patient is taking? (A) Exenatide (B) Glipizide (C) Miglitol (D) Pramlintide (E) Rosiglitazone Marc Imhotep Cray, MD

463


The answer is D: Pramlintide. Glucagon is inhibited by two endogenous peptides: insulin and amylin. Pramlintide is an analog of amylin. Pramlintide also diminishes the postprandial blood glucose spike by slowing gastric emptying and suppressing the appetite. Currently, pramlintide is indicated only in patients with diabetes taking insulin with meals (A) Exenatide mimics incretin secretion, which increases glucose-dependent insulin secretion. It does not inhibit secretion. (B) Glipizide is a second-generation sulfonylurea. These drugs enhance insulin secretion but do not block glucagon secretion. (C) Miglitol inhibits intestinal brush border Îą-glucosidase. This enzyme is needed for the final steps in carbohydrate breakdown before absorption, so miglitol decreases the amount of dietary carbohydrate absorbed. (E) Rosiglitazone lowers blood glucose by increasing insulin sensitivity in peripheral tissues. It does not inhibit glucagon secretion. Marc Imhotep Cray, MD

464


Components of comprehensive diabetes care Management of diabetes

Glycemic control • Diet/lifestyle • Exercise • Medication

Marc Imhotep Cray, MD

Treat associated conditions • Dyslipidemia • Hypertension • Obesity • CV disease

Screen for/manage complications • Retinopathy • CV disease • Neuropathy • Nephropathy • Other complications

Brunton LL, Chabner BA & Knollmann BC (Eds.). Goodman and Gilman’s The Pharmacological Basis of Therapeutics. 12th ed. New York: McGraw-Hill, 2011

465


Goals of Therapy in Diabetes INDEX

GOAL

Glycemic control A1C

<7.0%

Preprandial capillary plasma glucose

3.9-7.2 mmol/L (70-130 mg/dL)

Peak Postprandial capillary plasma

glucose 10.0 mmol/L (<180 mg/dL)d

Blood pressure

<130/80

Lipids Low-density lipoprotein

<2.6 mmol/L (<100 mg/dL)

High-density lipoprotein

>1.1 mmol/L (>40 mg/dL)

Triglycerides

<1.7 mmol/L (<150 mg/dL)

Redrawn from: Brunton LL, Chabner BA & Knollmann BC (Eds.). Goodman and Gilman’s The Pharmacological Basis of Therapeutics. 12th ed. New York: McGraw-Hill, 2011

Marc Imhotep Cray, MD

466


Treatment guidelines for T2DM

Marc Imhotep Cray, MD

Whalen K, Finkel R & Panavelil TA. Lippincott Illustrated Reviews-Pharmacology 6th Ed. Philadelphia, PA: Wolters Kluwer, 2015

467


Drugs for diabetes mellitus

Katzung & Trevors Pharmacology Examination and Board Review, 11th Ed. McGraw-Hill, 2015.

Marc Imhotep Cray, MD


Case 43 Answers Pancreas and Glucose Homeostasis Summary: A 12-year-old with newly diagnosed type I diabetes mellitus has ketoacidosis. • Structure of human insulin: A 51-amino acid polypeptide that consists of two chains linked by two disulfide bridges. • Effect of insulin on potassium: Promotes cellular K + uptake. • Effect of α-adrenergic stimulation: Inhibition of insulin secretion. • Effect of β-adrenergic stimulation: Increased insulin secretion. CLINICAL CORRELATION Insulin is a 51-amino acid polypeptide that is produced in pancreatic β cells and stored as a complex with Zn 2+ . The primary stimulus for insulin release is glucose, but amino acids, fatty acids, and ketone bodies may stimulate its release. Glucagon and somatostatin also modulate its secretion. α-Adrenergic stimulation is a predominant inhibitory mechanism, whereas β-adrenergic Marc Imhotep Cray, MD stimulation increases its release.

469


Case 43 Answers Pancreas and Glucose Homeostasis cont. Insulin acts by binding to specific membrane receptors that have tyrosine kinase activity. Tyrosine in the receptor becomes phosphorylated and the phosphoreceptor in turn phosphorylates a number of intracellular substrates that lead to increased glucose uptake. In muscle and adipose tissue, glucose transport is mediated by the recruitment of hexose transport molecules (GLUT-4) into the plasma membrane. Among its many actions, insulin increases glucose transport, glycogen synthesis and deposition, lipogenesis, and protein synthesis. It decreases intracellular lipolysis and hepatic gluconeogenesis. Insulin also stimulates cellular potassium accumulation. Type I diabetes mellitus is a disease in which pancreatic β cells fail to produce adequate amounts of insulin. Marc Imhotep Cray, MD

470


Case 43 Answers Pancreas and Glucose Homeostasis cont. Insulin must then be supplemented. Currently used insulin preparations are all human insulin produced by recombinant deoxyribonucleic acid (DNA) techniques. There are short-, intermediate-, and long-acting insulin preparations available. The most widely used insulin products must be given by injection, usually requiring 1–4 subcutaneous injections a day or continuous subcutaneous infusion with an insulin pump. Regular insulin can also be given intravenously in the setting of diabetic ketoacidosis. An insulin product was available for inhalation use but it was removed from the market due to lack of demand, need for high doses, and need for recurrent lung function monitoring. Another inhaled insulin is going to be introduced shortly. Insulin injections are also used in type II diabetics, who cannot achieve adequate control with oral agents. The most significant risk of insulin therapy is the induction of hypoglycemia. Hypoglycemia may produce tachycardia, sweating, and confusion. In severe cases, hypoglycemia may progress to coma, seizures, or even death. Marc Imhotep Cray, MD

471


Question & Answer Review

Marc Imhotep Cray, MD

472


Practice Question 1 Which of the following best describes the action of somatostatin? A. Inhibition of growth hormone release B. Inhibition of prolactin release C. Stimulation of insulin release D. Stimulation of LH release

Marc Imhotep Cray, MD

473


Answer 1 A. Somatostatin is a major regulator of growth hormone, and its effect is inhibitory of growth hormone release.

Marc Imhotep Cray, MD

474


Practice Question 2 In the first 2 weeks following a single injection of leuprolide in a man, one would expect which of the following? A. Decreased LH production B. Decreased testosterone production C. Increased LH receptors D. Increased testosterone production

Marc Imhotep Cray, MD

475


Answer 2 D. Acute leuprolide will increase FSH/LH and sex steroid production and have little effect on receptor numbers.

Marc Imhotep Cray, MD

476


Practice Question 3 A 22-year-old woman has severe endometriosis with dysmenorrhea. She is treated with depot leuprolide acetate. One week after her first injection, she notes a marked increase in the dysmenorrhea. What is the explanation? A. Direct effect of leuprolide on the endometrial implants B. Likely flare with increased gonadotropin effect prior to downregulation of receptors C. Probable placebo effect D. Resistance of her

Marc Imhotep Cray, MD

477


Answer 3 B. The initial response to GnRH analog is an increase in FSH and estrogen, leading to an exacerbation of the endometriosis. Thereafter, there is a downregulation of GnRH receptors of the pituitary, leading to a decrease in FSH and estrogen.

Marc Imhotep Cray, MD

478


Question 4 Which of the following best describes appropriate protocols for withdrawal of glucocorticoids from a patient who has been taking large doses for 6 months? A. Maintain dose of glucocorticoids and add metyrapone. B. Maintain dose of glucocorticoids and add spironolactone. C. An alternate-day dosage regimen of glucocorticoids should be begun. D. Slow reduction of the glucocorticoid dose over 1–2 weeks.

Marc Imhotep Cray, MD

479


Answer 4 D. Long-term use of glucocorticoids results in adrenal suppression and atrophy. A slow “weaning� from the drug is necessary so that the adrenals can recover.

Marc Imhotep Cray, MD

480


Question 5 A patient with severe shoulder pain resulting from inflammation is not responding to treatment with naproxen. You elect to begin a course of treatment with oral dexamethasone. What is the basis that the glucocorticoid will be more effective as an anti inflammatory agent? A. Glucocorticoids inhibit both prostaglandin production and inflammatory cells. B. Glucocorticoids are more potent inhibitors of cyclooxygenase than naproxen. C. Glucocorticoids inhibit biosynthesis of both COX-1 and COX-2. D. Glucocorticoids will reduce the edema in the inflamed area. Marc Imhotep Cray, MD

481


Answer 5 A. Glucocorticoids reduce prostaglandin production like NSAIDs and they also inhibit most of the cells that are involved in the inflammatory process.

Marc Imhotep Cray, MD

482


Question 6 A 32-year-old woman is prescribed a pill for excessive hair on her face and arms. She notes that she has been going to the bathroom at night more often. What is the most likely explanation for the nocturia? A. Diabetes insipidus effect of the medication B. Osmotic load to the kidney from the medication delivery system C. Distal renal tubule effect of the medication D. Hyperglycemic effect from the medication

Marc Imhotep Cray, MD

483


Answer 6 C. The medication is probably spironolactone, which is a competitive inhibitor of androgens at the receptor level, and also an antimineralocorticoid effect at the distal tubule, inhibiting free water resorption. As such, it is a potassium sparing diuretic agent.

Marc Imhotep Cray, MD

484


Question 7 A woman enters your clinic with an enlarged thyroid and you suspect simple adenomatous goiter. The serum TSH is elevated. Which of the following would be the best treatment for this condition? A. IV infusion of TSH B. Levothyroxine C. Propylthiouracil D. Thyroid ablation with 131 I

Marc Imhotep Cray, MD

485


Answer 7 B. Hypothyroidism is an indication for thyroid hormone replacement.

Marc Imhotep Cray, MD

486


Question 8 The mechanism by which thiocyanate reduces synthesis of thyroid hormones is by inhibition of which of the following? A. Iodine oxidation B. Iodide transport C. TSH biosynthesis D. TRH

Marc Imhotep Cray, MD

487


Answer 8 B. Anions such as perchlorate and thiocyanate inhibit the transport of iodide into thyroid cells.

Marc Imhotep Cray, MD

488


Question 9 A 33-year-old man is noted to have tachycardia, heat intolerance, weight loss, and an enlarged thyroid gland. Which of the following is the probable ultimate treatment for this patient? A. Long-term corticosteroid therapy B. Propranolol therapy C. Radioactive iodine D. Surgical resection

Marc Imhotep Cray, MD

489


Answer 9 C. This patient likely has Graves disease, the most common cause of hyperthyroidism in the United States, typically presenting with a painless goiter and symptoms of hyperthyroidism. The treatment of choice is radioactive iodine. Propranolol will help with the symptoms of tachycardia but not the underlying disease process.

Marc Imhotep Cray, MD

490


Question 10 Which of the following is the goal of insulin therapy? A. Control serum glucose as tightly as possible B. Control triglyceride biosynthesis C. Maintain adequate hepatic glycogen stores D. Maintain serum K + homeostasis

Marc Imhotep Cray, MD

491


Answer 10 A. The goal in treating diabetes is tight control of serum glucose to avoid the complications of hyperglycemia.

Marc Imhotep Cray, MD

492


Question 11 The thiazolidinediones are useful in treating type II diabetes because they have which of the following effects? A. Decrease the degradation of insulin B. Increase insulin release C. Increase glucose utilization D. Increase glucose uptake in muscle cells

Marc Imhotep Cray, MD

493


Answer 11 D. The TZDs are insulin sensitizers; they do not alter insulin secretion or degradation but act to increase glucose uptake in adipose and muscle.

Marc Imhotep Cray, MD

494


Question 12 A 42-year-old man is diagnosed with diabetes mellitus. He has tried diet and exercise without success. A second-generation sulfonylurea agent is prescribed. Which of the following is the most likely side effect he will experience? A. Agranulocytosis B. Hypoglycemia C. Lactic acidosis D. Myositis

Marc Imhotep Cray, MD

495


Answer 12 B. In general, the most common adverse effect of the agents for diabetes is hypoglycemia.

Marc Imhotep Cray, MD

496


Question 13 A 45-year-old female with past history of type II diabetes mellitus presents for routine follow-up. Her fasting blood sugars are controlled in the 80–100 range. However, on your lab report the hemoglobin A1c is elevated at 8.5. You presume that she needs better mealtime glucose control. She is already on an oral biguanide and oral sulfonylurea. Which of the following agents would help with her prandial or mealtime glucose control without persisting in her system to cause later hypoglycemia? A. Insulin detemir B. NPH insulin C. Repaglinide D. Insulin aspart

Marc Imhotep Cray, MD

497


Answer 13 D. Insulin aspart is a rapid acting insulin ideal for mealtime glucose control as its onset is 5–15 minutes and duration is on average 2–3 hours. NPH and detemir insulin have longer durations of action and may combine with the oral sulfonylurea and insulin glargine to cause hypoglycemia later on. Repaglinide is a fair option, however, in general patients on sulfonylureas should not be on repaglinide or nateglinide as they have similar mechanisms of action.

Marc Imhotep Cray, MD

498


THE END

See next slide for further study. Marc Imhotep Cray, MD

499


Sources and further study: eLearning Endocrine cloud folder tools and resources MedPharm Guidebook: Unit 5 Drugs Used In Disorders of Endocrine System Endocrine and Reproductive System Pharmacology eNotes Clinical Pharmacology Cases 39 to 43 (Learning Triggers) Textbooks Brunton LL, Chabner BA , Knollmann BC (Eds.). Goodman and Gilman’s The Pharmacological Basis of Therapeutics. 12th ed. New York: McGraw-Hill, 2011 Katzung, Masters, Trevor. Basic and Clinical Pharmacology, 12th ed. New York: McGraw-Hill, 2012 Mulroney SE. and Myers AK. Netter's Essential Physiology. Philadelphia: Saunders, 2009 Raff RB, Rawls SM, Beyzarov EP. Netter's Illustrated Pharmacology, Updated Edition. Philadelphia: Sanders, 2014 Toy E C. et.al. Case Files-Pharmacology Lange 3rd ed. New York: McGraw-Hill 2014. Marc Imhotep Cray, MD

500


Turn static files into dynamic content formats.

Create a flipbook
Issuu converts static files into: digital portfolios, online yearbooks, online catalogs, digital photo albums and more. Sign up and create your flipbook.